Вы находитесь на странице: 1из 113

This Copy is for Dr.

Mohamed ElHodiby
Question 1 Trophoblast cells that invade the decidua and fuse to form multi-nucleated giant cells

Options for Questions 1-1

A Extra-villous cytotrophoblasts B Villous cytotrophoblasts


C Endovascular trophoblasts D Extravascular trophoblasts
E Interstitial trophoblasts

A(Correct answ er: E)

Explanation
DEVELOPMENT OF THE VILLOUS TREE
• Primary villous stems become infiltrated by cytotrophoblasts between days 13-21 post-ovulation
• Villous stems are subsequently infiltrated by extra-embryonic mysenchyme which differentiates into fetal blood
vessels
• The distal parts of the villous stems are not vascularised. Here, cytotrophoblasts proliferate and spread laterally to
form a cytotrophoblastic shell, splitting the syncytiotrophoblast into a definitive syncytiotrophoblast on the fetal side
and the peripheral syncytium on the decidual side which degenerates and is replaced by fibrinoid material (Nitabuch's
layer)
• Sprouts extend from primary villous stems, initially made up of syncytiotrophoblast and then infiltrated by
cytotrophoblast and mesenchyme - these are primary stem villi and the placenta is a true villous structure by day 21
of gestation. These villi grow and divide into secondary, tertiary and terminal villi
• The villi oriented towards the uterine cavity degenerate between day 21 and the 4th month to form the chorion laeve.
The overlying decidua degenerates and the chorion laeve comes in contact with the deciduas of the opposite uterine
wall
• The rest of the villi form the chorion frondosum which develops into the definitive placenta
• Division and modification of the villous tree continues until term. First trimester villi are larger, have a complete layer
of cytotrophoblasts and have a loose mysenchymal core which is vascularised towards the end of the first trimester
• At term, the villi are smaller, cytotrophoblasts are few in number, the syncytiotrophoblast is irregularly thinned. Fetal
vessels are sinusoidal and occupy most of the villous core and lie close to the syncytiotrophoblast, forming
vasculusyncytial membranes which maximise materno-fetal transfer.
• Sometimes, the syncytiotrophoblast nuclei appear in clusters called syncytial knots - more common in placentas from
IUGR / pre-eclamptic pregnancies
• Maternal blood is separated from fetal blood by the syncytiotrophoblast and the fetal capillary endothelium

Question 2 The second wave of trophoblast invasion occurs at this gestation

Options for Questions 2-2

A 4-6 weeks B 8-12 weeks


C 12-16 weeks D 16-18 weeks
E 22-24 weeks

A(Correct answ er: D)

Explanation

DEVELOPMENT OF THE VILLOUS TREE

This Copy is for Dr. Mohamed ElHodiby


• Primary villous stems become infiltrated by cytotrophoblasts between days 13-21 post-ovulation
• Villous stems are subsequently infiltrated by extra-embryonic mysenchyme which differentiates into fetal blood
vessels
• The distal parts of the villous stems are not vascularised. Here, cytotrophoblasts proliferate and spread laterally to
form a cytotrophoblastic shell, splitting the syncytiotrophoblast into a definitive syncytiotrophoblast on the fetal side
and the peripheral syncytium on the decidual side which degenerates and is replaced by fibrinoid material (Nitabuch's
layer)
• Sprouts extend from primary villous stems, initially made up of syncytiotrophoblast and then infiltrated by
cytotrophoblast and mesenchyme - these are primary stem villi and the placenta is a true villous structure by day 21
of gestation. These villi grow and divide into secondary, tertiary and terminal villi
• The villi oriented towards the uterine cavity degenerate between day 21 and the 4th month to form the chorion laeve.
The overlying decidua degenerates and the chorion laeve comes in contact with the deciduas of the opposite uterine
wall
• The rest of the villi form the chorion frondosum which develops into the definitive placenta
• Division and modification of the villous tree continues until term. First trimester villi are larger, have a complete layer
of cytotrophoblasts and have a loose mysenchymal core which is vascularised towards the end of the first trimester
• At term, the villi are smaller, cytotrophoblasts are few in number, the syncytiotrophoblast is irregularly thinned. Fetal
vessels are sinusoidal and occupy most of the villous core and lie close to the syncytiotrophoblast, forming
vasculusyncytial membranes which maximise materno-fetal transfer.
• Sometimes, the syncytiotrophoblast nuclei appear in clusters called syncytial knots - more common in placentas from
IUGR / pre-eclamptic pregnancies
• Maternal blood is separated from fetal blood by the syncytiotrophoblast and the fetal capillary endothelium

Question 3 Is used by the placenta to produce oestrogens

Options for Questions 3-3

A Cholesterol B Testosterone
C DHEA-sulphate D Progesterone
E Cortisol

A(Correct answ er: C)

Explanation

PLACENTAL OESTROGENS
Mainly oestriol, but also oestradiol and oestrone in smaller amounts. Oestriol is produces from DHES-sulphate from
fetal zone of the fetal adrenal gland and also from the maternal adrenals. Fetal DHEA-S is initially hydroxylated by the
fetal liver

Placental hormone which is produced by the syncytiotrophoblast but not by the cytotrophoblast
Question 4
in vivo
Options for Questions 4-4

A Human chorionic gonadotrophin B Human placental lactogen


C Inhibin D Human chorionic thyrotropin
E Oestriol

A(Correct answ er: A)

Explanation
HCG
Produced by the syncytiotrophoblast. Cytotrophoblasts produce HCG in-vitro
Human Placental Lactogen

This Copy is for Dr. Mohamed ElHodiby


Has growth-hormone -like effects and decreases insulin-sensitivity.
Human chorionic thyrotropin and virtually all of the hypothalamic releasing hormones

Question 5 Concentrations of this hormone peak in the first trimester and rise again at term
Options for Questions 5-5

A Inhibin A B Activin
C Human chorionic gonadotrophin D Human placental lactogen
E Progesterone

A(Correct answ er: A)

Explanation
INHIBIN & ACTIVIN
Produced by the feto-placental unit (mainly by the ovary in non-pregnant state)
Inhibin-A levels peak in early pregnancy and rise again at term and are increased in pre-eclampsia
Activin levels increase with gestation age and a marked increase occurs with the onset of labour and in pre-
eclampsia

Question 6 Which one of the above is not readily transferred across the placenta?

Options for Questions 6-6

A Amino acids B Glucose


C Thyroxine D IgG
E Ca2+

A(Correct answ er: C)

Explanation
Molecule Placental transfer

Testosterone Minimal transfer - androgens aromatised by placenta. Very high maternal androgen
concentration may virilise female fetus

Ca2+, Mg2+ Active transfer against concentration gradient

PTH, Calcitonin Not transferred

Vitamin D Good transfer

IgA Minimal passive transfer

IgG Good active and active transfer from 7 weeks gestation

IgM No transfer

Glucose Facilitated diffusion - excellent transfer

Amino acids Active transport - excellent transfer

Free fatty acids Very limited transfer - essential fatty acids only

This Copy is for Dr. Mohamed ElHodiby


Ketone bodies Excellent transfer - diffusion

Insulin,
No transfer
glucagon
Thyroid
Poor transfer - diffusion
hormone

TRH Excellent transfer

Iodine and
Excellent transfer
thioamides
Cortisol &
Excellent transfer
aldosterone

ACTH No transfer

Question 7 Which one of the above is readily transferred across the placenta?

Options for Questions 7-7

A Testosterone B Parathyroid hormone


C Vitamin D D IgM
E Calcitonin

A(Correct answ er: C)

Explanation
Molecule Placental transfer

Testosterone Minimal transfer - androgens aromatised by placenta. Very high maternal androgen
concentration may virilise female fetus
Ca2+, Mg2+ Active transfer against concentration gradient
PTH, Calcitonin Not transferred
Vitamin D Good transfer
IgA Minimal passive transfer
IgG Good active and active transfer from 7 weeks gestation
IgM No transfer

Glucose Facilitated diffusion - excellent transfer


Amino acids Active transport - excellent transfer
Free fatty acids Very limited transfer - essential fatty acids only
Ketone bodies Excellent transfer - diffusion
Insulin, glucagon No transfer
Thyroid hormone Poor transfer - diffusion
TRH Excellent transfer
Iodine and thioamides Excellent transfer
Cortisol & aldosterone Excellent transfer
ACTH No transfer

Question 8 Dizygotic twins

This Copy is for Dr. Mohamed ElHodiby


Options for Questions 8-8

Have no more resemblance than siblings of


A Are less common than monozygotic twins B
different ages
C Can share the same placenta D Can share the same chorionic sac
E Can be monochorionic

A(Correct answ er: B)

Explanation
MULTIPLE PREGNANCY?Twins
UK data 2007: ~ 1 in 65 (1.5%) pregnancies were twins
Prevalence varies world-wide, being lowest in Japan and highest in Nigeria
Incidence of monozygotic twins relatively constant world-wide at ~ 3.5 per 1,000 births
Incidence of dizygotic twins varies widely
Dizygotic twins
• Fertilisation of two oocytes by different sperm
• Dizygotic twins have no more resemblance than brothers / sisters of different ages
• Both zygotes implant independently in the uterus and there are two separate placentas, amniotic and chorionic sacs
• The placentas and chorionic sacs may come into close approximation and fuse.
• Dizygotic twins cannot be monochorionic and cannot be identical. They are always dichorionic and diamniotic
Monozygotic twins
• A single ovum is fertilised and splits into two at different stages of development
• Earliest separation occurs at the 2 cell stage producing two zygotes which enter the uterus independently. Each
embryo would have its own placenta and chorionic sac, forming dichorionic diamniotic twins
• In the majority of cases, splitting occurs in the early blastocyst stage. The inner cell mass splits into two, producing
two embryos with a common placenta and a common chorionic cavity but separate amniotic cavities. These form
monochorionic diamniotic twins
• Rarely, splitting occurs at the stage of the bilaminar germ disc, resulting in two embryos with a common placenta,
common chorionic cavity and common amniotic cavity. These form monochorionic monoamniotic twins
• Splitting at later stages of development may result in incomplete splitting of the axial area of the germ disc, resulting
in conjoint twins

Question 9 With respect to the anatomy of the rectum

Options for Questions 9-9

Lymphatic drainage of the upper third is to the Lymphatic drainage of the middle third is to the
A B
superior mesenteric nodes superior mesenteric nodes
Lymphatic drainage of the lower third is to the Nerve supply is from the superior mesenteric
C D
inferior mesenteric nodes plexus
Venous drainage forms part of the portal-systemic
E
anastomoses

A(Correct answ er: E)

Explanation
THE RECTUM
• ~13cm long
• Upper third - covered by peritoneum on its anterior and lateral surfaces, middle third covered by peritoneum on the
anterior surface only and lower third devoid of peritoneum

This Copy is for Dr. Mohamed ElHodiby


• The three teniae coli of sigmoid colon come together to form a broad band of longitudinal smooth muscle on its
anterior and posterior surfaces
• Three transverse folds, two on the left and one on the right
• Anterior relations: sigmoid colon and small intestine within pouch of Douglas (upper third); Posterior surface of vagina
(lower third)
• Posterior relations: sacrum, coccyx, piriformis and coccygeus muscles, lavatory ani, sacral plexus and pelvic
sympathetic trunk
Blood supply
• Superior rectal artery - continuation of inferior mesenteric artery - main blood supply to mucosa
• Middle rectal artery - branch of internal iliac artery - muscular coat mainly
• Inferior rectal artery - branch of internal pudendal artery
• Veins correspond to arteries and form an important portal-systemic anastomosis
• Lymphatic drainage: Upper two thirds - inferior mesenteric nodes; lower third - internal iliac node
• Nerve supply - inferior hypogastric plexus

Question 10 Venous drainage of the right ovary

Options for Questions 10-10

A Superior mesenteric vein B Inferior vena cava


C Left renal vein D Right renal vein
E Inferior mesenteric vein
A(Correct answ er: B)

Explanation
THE OVARY
• 4x2 cm, attached to the posterior aspect of the broad ligament by a mesentery - the mesovarium
• Attached to the lateral pelvic wall by the suspensory ligament of the ovary (infundibulo-pelvic ligament)
• Attached to the upper part of the lateral uterine wall by the round ligament of the ovary (remnant of upper
part of the gubenaculum)
• Position variable, but usually lies within ovarian fossa in lateral pelvic wall. The ovarian fossa is bounded
superiorly by the external iliac vessels, inferiorly by the ureter and internal iliac vessels and its floor is
crossed by the obturator nerve
• The ovary is surrounded by a thin fibrous capsule - the tunica albuginae
• Blood supply - ovarian artery - branch of abdominal aorta at L1
• Venous drainage - LEFT -left ovarian vein drains into left renal vein; RIGHT - right ovarian vein drains into
inferior vena cava
• Lymphatics - para-aortic nodes?Nerve - aortic plexus??

Question 11 The blood supply to the fallopian tube

Options for Questions 11-11

A Ovarian artery B Uterine artery


C Ovarian and uterine arteries D External iliac artery
E Internal iliac artery

A(Correct answ er: C)

Explanation

This Copy is for Dr. Mohamed ElHodiby


FALLOPIAN TUBES
• ~10cm long, 4 parts
• Infundibulum - funnel-shaped lateral end, projects beyond the broad ligament with fimbriae at its free end
• Ampulla - widest and longest part, site of fertilisation
• Isthmus - narrowest part, just lateral to the uterus
• Intra-mural part - pierces uterine wall
• Three coats: serous, muscular, and mucous.
• The external or serous coat is peritoneal. The middle or muscular coat consists of an external longitudinal and an
internal circular layer of smooth muscle fibers continuous with those of the uterus
• The mucosa is thrown into longitudinal folds, which in the ampulla are much more extensive than in the isthmus.
Lined by ciliated columnar epithelium
• Blood - ovarian and uterine arteries
• Lymphatics - aortic and internal iliac nodes (follow arteries)
• Nerves - inferior hypogastric plexus

Question 12 Which one of the above statements regarding the fallopian tube is true?
Options for Questions 12-12

The ampulla is the widest part and the site of


A The fallopian tube is 25cm long B
fertilization
C The intra-mural part is the narrowest segment D It does not have a muscular layer
E The fimbrial end is attached to the ovary

A(Correct answ er: B)

Explanation
FALLOPIAN TUBES
• ~10cm long, 4 parts
• Infundibulum - funnel-shaped lateral end, projects beyond the broad ligament with fimbriae at its free end
• Ampulla - widest and longest part, site of fertilisation
• Isthmus - narrowest part, just lateral to the uterus
• Intra-mural part - pierces uterine wall
• Three coats: serous, muscular, and mucous.
• The external or serous coat is peritoneal. The middle or muscular coat consists of an external longitudinal and an
internal circular layer of smooth muscle fibers continuous with those of the uterus
• The mucosa is thrown into longitudinal folds, which in the ampulla are much more extensive than in the isthmus.
Lined by ciliated columnar epithelium
• Blood - ovarian and uterine arteries
• Lymphatics - aortic and internal iliac nodes (follow arteries)
• Nerves - inferior hypogastric plexus

Question 13 The nerve supply to the body of the uterus

Options for Questions 13-13

A Aortic plexus B Superior hypogastric plexus


C Inferior hypogastric plexus D Superior mesenteric plexus
E Inferior mesenteric plexus

This Copy is for Dr. Mohamed ElHodiby


A(Correct answ er: C)

Explanation
THE UTERUS
• 8cm long x 5cm wide x 2.5cm thick
• Covered by peritoneum except anteriorly below the reflection of the utero-vesical fold of peritoneum and laterally
between the layers of the broad ligament?
• Fundus - that part of the uterus above the entrance of the uterine tubes?
• Cavity - triangular in coronal section, cleft in saggital section?
• Anteverted uterus - long axis of uterine body at 90degrees to long axis of vagina?
• Ante-flexed uterus - long axis of the body of the uterus bent forward at the level of the internal os?
• Retroverted uterus - body and fundus bent backwards on the vagina to lie within the pouch of Douglas?
• Anterior relations: utero-vesical pouch and superior surface of bladder?
• Posterior relations: Pouch of Douglas, sigmoid colon and coils of ileum?
• Lateral relations: uterine vessels, ureter?
• Nerve - inferior hypogastric plexuses (Parasympathetic via the pelvic splanchnic nerves, sympathetic via the lumbar
splanchnic nerves). Pain sensation is transmitted via the sympathetic nerves and the lumbar splanchnic nerves??
Lymph drainage:
• Fundus - accompany ovarian artery to para-aortic nodes at the level of L1
• Body and cervix - internal and external iliac nodes

Question 14 With respect to the innervation and support of the cervix

Options for Questions 14-14

The round ligament provides the most important The broad ligament provides the most important
A B
support to the cervix support to the cervix
The endocervix has autonomic but no sensory Sensory innervation of the cervix is via S2,3,4
C D
innervation spinal segments
The ectocervix has sensory but minimal
E
autonomic innervation

A(Correct answ er: D)

Explanation
?CERVIX
• Lower, narrow portion of the uterus, connected to the uterine fundus by the uterine isthmus - upper limit is the internal
os. Made up mainly of fibrous tissue with very little smooth muscle
• Protrudes through the upper anterior vaginal wall
• Approximately half its length is visible in the vagina (vaginal cervix), the rest being above the vagina (supra-vaginal
cervix)
• The vaginal cervix ~3 cm long and 2.5 cm wide. Size and shape varies widely with age, hormonal state, and parity -
bulkier and the external with a wider and more slit-like external os in multiparous women.
• Ectocervix - portion of the cervix beyond the external os - lined by stratified squamous non-keratinising epithelium.
• Endocervical canal - links external and internal os - lined by columnar epithelium
• The squamo-columnar junction - variable location - high up the endocervical canal before puberty and in the post-
menopausal women. Site of origin of squamous cell carcinoma of the cervix
• The external os is bounded by two lips, an anterior and a posterior, of which the anterior is the shorter and thicker,
although due to the slope of the cervix, it projects lower than the posterior. Both lips are in contact with the posterior
vaginal wall
• The supravaginal cervix is separated in front from the bladder by fibrous tissue (parametrium), which extends also on
to its sides and laterally between the layers of the broad ligaments. Not covered by peritoneum on the anterior aspect
• The uterine arteries reach the margins of the cervix within the parametrium

This Copy is for Dr. Mohamed ElHodiby


• The ureter runs downward and forward 2 cm lateral to the supravaginal cervix
• Posteriorly, the supravaginal cervix is covered by peritoneum, which extends on to the posterior vaginal wall, when it
is reflected on to the rectum, forming the Pouch of Douglas which may contain coils of small intestine.
• The vaginal cervix projects free into the anterior wall of the vagina between the anterior and posterior fornices.
?Blood supply
• Uterine artery, branch of internal iliac
• Cervical and vaginal branches supply the cervix and upper vagina.
• The cervical branches of the uterine arteries descend on the lateral aspects of the cervix at 3 and 9 o'clock. Venous
drainage parallels the arterial supply, eventually emptying into the hypogastric venous plexus.
?Lymphatics
• Regional lymph nodes for the cervix include: paracervical, parametrial, presacral, sacral, external iliac, common iliac,
hypogastric (obturator), internal iliac.

Question 15 Somatic sensory innervation of the lower third of the vagina

Options for Questions 15-15

A Pelvic splanchnic nerves B Lumbar splanchnic nerves


C Pudendal nerve D Superficial femoral nerve
E Obturator nerve

A(Correct answ er: C)

Explanation

THE VAGINA
• ~8cm long, axis directed upwards and backwards from the vulva. Posterior wall longer than anterior wall
• Lined by stratified squamous epithelium which undergoes changes during the menstrual cycle. Does not secrete
mucus
• Has anterior and posterior walls which are normally in apposition, and four fornices (anterior, posterior, left and right
lateral)
• Upper half lies above the level of the pelvic floor
?Relations
• Anterior: bladder, urethra
• Posterior: upper third - pouch of Douglas; middle third - ampulla of the rectum; lower third - perineal body
• Lateral: upper part - ureter, middle part - anterior fibres of levator ani; lower part - uro-genital diaphragm and the bulb
of the vestibule
• Blood: vaginal artery, branch of internal iliac artery
• Lymphatics: upper third - internal and external iliac nodes; middle third - internal iliac nodes; lower third - superficial
inguinal nodes
• Nerve:Upper two thirds - inferior hypogastric plexuses. Parasympathetic from pelvic splanchnic nerves, sympathetic
from lumbar splanchnic nerves (L1 & 2)
• Autonomic innervation to the lower third - pudendal nerve
• Somatic sensation is present mainly in the lower third and is carried by the pudendal nerve

Question 16 The posterior border of the pelvic outlet

Options for Questions 16-16

A Ischial tuberosities B Sacro-spinous & sacro-tuberous ligaments


C Ilio-inguinal ligaments D Coccyx
E Sacral promontory

This Copy is for Dr. Mohamed ElHodiby


A(Correct answ er: D)

Explanation
TRUE PELVIS
That part of the pelvis between the pelvic inlet and the pelvic outlet
The pelvic inlet is oval in shape in the Gynaecoid pelvis and has the following borders:
1) Posterior: Sacral promontory?2) Lateral: Iliopectineal line?3)Anterior: Symphysis pubis
The pelvic outlet is diamond shaped with the following borders:
• Posterior: Coccyx
• Lateral: Ischial tuberosities
• Anterior: Pubic arch formed by the simphysis pubis and the ischio-pubic rami
• Part of the lateral border is formed by the sacro-spinous and sacro-tuberous ligaments which convert the greater and
lesser sciatic notches into the greater and lesser sciatic foramina
• The pelvic cavity has a shallow anterior wall and a deeper posterior wall. The ischial spines are at the level of the
mid-cavity
• The acetabulum is formed from the ilium, ischium and pubic bones

Question 17 The origin of the piriformis muscle

Options for Questions 17-17

A Greater trochanter B Lesser trochanter


C Body of sacral vertebrae D Lateral mass of sacral vertebrae
E Simplysis pubis

A(Correct answ er: D)

Explanation
PYRIFORMIS
• Origin: Lateral mass of sacrum
• Leaves pelvis through greater sciatic foramen
• Insertion: Upper border of greater trochanter
• Action: Laterally rotates femur at hip joint
• Nerve: Sacral plexus

Question 18 The pudendal nerve

Options for Questions 18-18

A Enters the pelvis through the greater sciatic notch B Leaves the pelvis through the lesses sciatic notch
Enters the perineum through the lesser sciatic
C D Is formed from branches of L2,3 4
foramen
E Supplies the gluteus maximus muscle

A(Correct answ er: C)

Explanation
Branches to pelvic organs
• Pudendal nerve - S2,3&4 - leaves the pelvis through the greater sciatic foramen and enters the perineum through the
lesser sciatic foramen
• Nerve to piriformis

This Copy is for Dr. Mohamed ElHodiby


• Pelvic splanchnic nerves - S2,3&4 - sacral part of parasympathetic system
• Perforating cutaneous nerve - skin of lower medial part of buttock

Question 19 Which one of the above organs is correctly paired with its parasympathetic nerve supply?

Options for Questions 19-19

A Urinary bladder – lumbar splanchnic nerves B Small intestine – pelvic splanchnic nerves
C Descending colon – vagus nerve D Vagina – lumbar splanchnic nerves
E Ovary – vagus nerve

A(Correct answ er: E)

Explanation
PELVIC SPLANCHNIC NERVES
• Formed from the ventral (anterior) primary rami of S2 through S4.
• These are the ways in which parasympathetic neurons reach the hypogastric plexus, and therefore the pelvic viscera
and distal colon.
• The parasympathetic part of the autonomic nervous system is the "craniosacral" part. Parasympathetic innervation to
most of the gut comes from the "cranio-" half of that, i.e., the vagus nerve. The rest, to colon distal to the splenic
flexure and to pelvic viscera, is from the "-sacral" half, via the pelvic splanchnic nerves
• There are thoracic, lumbar, sacral, and pelvic splanchnic nerves.
• "Splanchnic" refers to nerves that supply viscera.
• Thoracic, lumbar and sacral splanchnic nerves emerge from sympathetic ganglia and carry sympathetic fibers
• Pelvic splanchnic nerves are parasympathetic
• Contain pre-ganglionic fibres
• Join inferior hypogastric plexus
• Some fibres ascend to the superior hypogastric and eventually inferior mesenteric plexus and supply the hind-gut
• Provide parasympathetic supply to the pelvic viscera
• Afferent impulses from the pelvic viscera are transmitted mainly by the sympathetic pathway
• Parasympathetic nerves innervate detrusor and internal sphincter via the pelvic splanchnic nerves (S2,3,4) and also
innervate the external sphincter via the pudendal nerve - initiate micturiction by inhibiting internal sphincter activity
and stimulating detrusor contraction
• Parasympathetic supply to the ovary is from the vagus nerve. Sympathetic innervation is from the L1/2 segment
• Pain from uterine contractions goes back to T10-L1. This means that uterine contraction pain is sympathetic. But for
the lower portion of the uterus and upper vagina (the cervix) - it is parasympathetic back to the pelvic splanchnic
nerves

Question 20 Which one of the above structures lies anterior to the pelvic sympathetic trunk?

Options for Questions 20-20

A The rectum B The sacrum


C Descending aorta D Inferior vena cava
E Internal iliac vessels

A(Correct answ er: A)

Explanation
PELVIC SYMPATHETIC TRUNK
• Continuous with the abdominal part behind the common iliac vessels
• Lies posterior to the rectum
• Lies anterior to the sacrum

This Copy is for Dr. Mohamed ElHodiby


• Lies medial to the anterior sacral foramina
• Has 4-5 segmentally arranged ganglia
• Gives off grey rami communicantes to the sacral and coccygeal spinal nerves
• Gives off fibres to the pelvic (hypogastric) plexuses
• No white rami communicantes join this part of the sympathetic trunk

Question 21 Which one of the above most accurately describes the Dentate line?

Options for Questions 21-21

A Mucosal fold overlying inferior rectal artery B Mucosal fold over anal columns
Boundary between ectodermal and mesodermal Boundary between ectodermal and endodermal
C D
origin of anal canal origin of anal canal
Boundary between mid-gut and hind-gut origins of
E
the anal canal

A(Correct answ er: D)

Explanation
THE ANAL CANAL
• ~4cm long, extends downwards and backwards from the rectal ampulla to the anus
• LATERAL walls kept in apposition by the levator ani and anal sphincter except during defecation
• Anterior relations: perineal body, urogenital diaphragm and perineal body
• Posterior relations: anococcygeal body and the coccyx
?UPPER HALF
• Derived from hind-gut endoderm
• Lined by columnar epithelium
• Thrown into vertical folds called anal columns
• Autonomic supply from the inferior hypogastric plexus, sensitive to stretch only
• Blood - superior rectal branch of inferior mesenteric artery
• Lymphatics - inferior mesenteric nodes
• Dentate line - boundary between endodermal and ectodermal origin of anal canal
?LOWER HALF
• Derived from the ectoderm of the proctodeum
• Lined by stratified squamous epithelium
• No anal columns
• Blood - inferior rectal artery, branch of internal pudendal artery
• Lymphatics - medial group of superficial inguinal nodes
• Nerve - inferior rectal nerve - sensitive to temperature, pain, touch and pressure

Question 22 The superior epigastric artery

Options for Questions 22-22

A Branch of the internal thoracic artery B Continuation of the internal thoracic artery
C Branch of the descending aorta D Branch of the external iliac artery
E Branch of the internal iliac artery

A(Correct answ er: B)

Explanation
Blood supply – abdominal wall

This Copy is for Dr. Mohamed ElHodiby


• Medial aspects - Superior epigastric artery (above umbilicus - continuation of the internal thoracic artery, branch of
the first part of the subclavian artery. Enters the rectus sheath between the sternal and costal origins of the
diaphragm and descends behind rectus muscle)
• Inferior epigastric artery (below umbilicus - branch of external iliac artery just above inguinal ligament. It pierces the
transversalis fascia to enter the rectus sheath anterior to the arcuate line; runs behind rectus muscle). There is no
anastomosis between the arteries of the left and right side
• Lateral aspects - intercostals, lumbar arteries and deep circumflex iliac artery (branch of external iliac artery above
the inguinal ligament).

Question 23 The sac of this hernia lies below and lateral to the pubic tubercle

Options for Questions 23-23

A Direct inguinal hernia B Indirect inguinal hernia


C Femoral hernia D Richcter’s hernia
E Epigastric hernia

A(Correct answ er: C)

Explanation
HERNIAS
INGUINAL
• Indirect commoner than direct
• Indirect more common in males and on the right side (right testis descends later than the left)
• Direct hernias commoner in (old) males
• Indirect: Enters inguinal canal through deep inguinal ring, lateral to inferior epigastric vessels. Can descent into the
scrotum or labia majora. Sac is formed by the remains of the processus vaginalis
• The sac of all inguinal hernias lies above and medial to the pubic tubercle
FEMORAL
• Commoner in females protrudes through the femoral canal medial to the femoral vein, below and lateral to the pubic
tubercle.
UMBILICAL
• Congenital - exomphalos
• Acquired infantile - weakness in scar of umbilicus
• Acquired adult - para-umbilical, weakness in linea alba above or below umbilicus, commoner in females
EPIGASTRIC
• Weakness in linea alba above umbilicus
RICHCTER’S
• A knuckle of the side-wall of the bowel is incarcerated in the sac but the continuity of the bowel is maintained and
there is no obstruction.

Question 24 This hernia may enter the scrotum or labia majora

Options for Questions 24-24

A Direct inguinal hernia B Indirect inguinal hernia


C Femoral hernia D Richcter’s hernia
E Epigastric hernia

A(Correct answ er: B)

Explanation
HERNIAS

This Copy is for Dr. Mohamed ElHodiby


INGUINAL
• Indirect commoner than direct
• Indirect more common in males and on the right side (right testis descends later than the left)
• Direct hernias commoner in (old) males
• Indirect: Enters inguinal canal through deep inguinal ring, lateral to inferior epigastric vessels. Can descent into the
scrotum or labia majora. Sac is formed by the remains of the processus vaginalis
• The sac of all inguinal hernias lies above and medial to the pubic tubercle
FEMORAL
• Commoner in females protrudes through the femoral canal medial to the femoral vein, below and lateral to the pubic
tubercle.
UMBILICAL
• Congenital - exomphalos
• Acquired infantile - weakness in scar of umbilicus
• Acquired adult - para-umbilical, weakness in linea alba above or below umbilicus, commoner in females
EPIGASTRIC
• Weakness in linea alba above umbilicus
RICHCTER’S
• A knuckle of the side-wall of the bowel is incarcerated in the sac but the continuity of the bowel is maintained and
there is no obstruction.

Question 25 With respect to hernias of the anterior abdominal wall

Options for Questions 25-25

Direct inguinal hernias are more common than The sac of indirect inguinal hernias lies below the
A B
indirect hernias pubic tubercle
The sac of direct inguinal hernias lies below the The sac of direct inguinal hernias lies medial to
C D
pubic tubercle the pubic tubercle
The sac of indirect inguinal hernias lies lateral to
E
the pubic tubercle

A(Correct answ er: D)

Explanation
HERNIAS
INGUINAL
• Indirect commoner than direct
• Indirect more common in males and on the right side (right testis descends later than the left)
• Direct hernias commoner in (old) males
• Indirect: Enters inguinal canal through deep inguinal ring, lateral to inferior epigastric vessels. Can descent into the
scrotum or labia majora. Sac is formed by the remains of the processus vaginalis
• The sac of all inguinal hernias lies above and medial to the pubic tubercle
FEMORAL
• Commoner in females protrudes through the femoral canal medial to the femoral vein, below and lateral to the pubic
tubercle.
UMBILICAL
• Congenital - exomphalos
• Acquired infantile - weakness in scar of umbilicus
• Acquired adult - para-umbilical, weakness in linea alba above or below umbilicus, commoner in females
EPIGASTRIC
• Weakness in linea alba above umbilicus

This Copy is for Dr. Mohamed ElHodiby


RICHCTER’S
• A knuckle of the side-wall of the bowel is incarcerated in the sac but the continuity of the bowel is maintained and
there is no obstruction.

Question 26 The nerve supply to the rectus abdominis muscle

Options for Questions 26-26

A Upper 6 thoracic nerves B Lower 6 thoracic nerves


C Upper 3 lumbar nerves D Lower 3 lumbar nerves
E 12th thoracic nerve

A(Correct answ er: B)

Explanation
NERVES
• External oblique, internal oblique and transversus: Lower 6 thoracic nerves and L1 (ileoinguinal and ileohypogastric
nerves)
• Rectus abdominis: Lower 6 thoracic nerves
• Pyramidalis: T12
FUNCTION
• External / internal oblique / transversus - laterally flex and rotate the trunk, relax during inspiration to accommodate
abdominal viscera, contract during micturiction, defecation and vomiting.
• Rectus abdominis - flexes the trunk and stabilises the pelvis

Question 27 Nerve supply to the internal oblique muscle

Options for Questions 27-27

A Upper 6 thoracic nerves B Lower 6 thoracic nerves


C Upper 3 lumbar nerves D Lower 3 lumbar nerves
E 12th thoracic nerve

A(Correct answ er: B)

Explanation
NERVES
• External oblique, internal oblique and transversus: Lower 6 thoracic nerves and L1 (ileoinguinal and ileohypogastric
nerves)
• Rectus abdominis: Lower 6 thoracic nerves
• Pyramidalis: T12
FUNCTION
• External / internal oblique / transversus - laterally flex and rotate the trunk, relax during inspiration to accommodate
abdominal viscera, contract during micturiction, defecation and vomiting.
• Rectus abdominis - flexes the trunk and stabilises the pelvis

Posterior wall of the rectus sheath between the costal margin and the anterior superior iliac
Question 28
spine
Options for Questions 28-28

A Aponeurosis of external oblique B Aponeurosis of internal oblique

This Copy is for Dr. Mohamed ElHodiby


Aponeurosie of internal oblique and transversus
C D Aponeuroses of external and internal oblique
abdominis
E Aponeurosis of transversus abdominis

A(Correct answ er: C)

Explanation
RECTUS SHEATH
Above costal margin
• Anterior wall: Aponeurosis of external oblique
• Posterior wall: thoracic wall.
Between costal margin and anterior superior iliac spine
• Anterior wall: Aponeurosis of external and internal oblique
• Posterior wall: Aponeurosis of internal oblique and transversus abdominis
• Note that the aponeurosis of the internal oblique splits to enclose the rectus abdominis
Between anterior superior iliac spine and pubis
• Anterior wall: Aponeuroses of external, internal oblique and transversus
• Posterior wall: Transversalis fascia
• Arcuate line : Site where the aponeuroses of the posterior wall pass anterior to the rectus at the level of the anterior
superior iliac spine. The inferior epigastric artery enters the rectus sheath at this point and lies posterior to the rectus
abdominis. Pyramidalis lies within the rectus sheath.

Question 29 Which one of the above does not pass through the superficial inguinal ring?

Options for Questions 29-29

A Round ligament of the uterus B Ilioinguinal nerve


C Genital branch of the genito-femoral nerve D Lateral cutaneous nerve of the thigh
E Sympathetic plexus

A(Correct answ er: D)

Explanation
INGUINAL CANAL
• 4cm long in adults
• Deep ring: - oval shaped hole in transversalis fascia, 1.3cm above inguinal ligament, mid-way between anterior
superior iliac spine and symphysis pubis. Medial relation - inferior epigastric vessels. Gives rise to the internal
spermatic fascia or the internal covering of the round ligament of the uterus
• Superficial ring:- triangular in shape, defect in external oblique aponeurosis, base formed by pubic crest, gives rise to
external spermatic fascia
• The following pass through the superficial ring: round ligament, ilioinguinal nerve, genital branch of the genitofemoral
nerve, lymphatics and sympathetic plexus
• Anterior wall - aponeurosis of external oblique, reinforced laterally by origin of internal oblique
• Posterior wall - transversalis fascia, reinforced medially by conjoint tendon - common insertion of internal oblique and
transversus to the pubic crest and pectineal line
• Floor - Inguinal ligament
• Roof - internal oblique and transversus abdominis
• The femoral neurovascular bundle and lateral cutaneous nerve of the thigh pass beneath the inguinal ligament.
• The femoral branch of the genitofemoral nerve enters the thigh behind the middle of the inguinal ligament.
• The ilioinguinal nerve enters the thigh through the superficial inguinal ring.
• The superficial epigastric vessels cross the inguinal ligament

This Copy is for Dr. Mohamed ElHodiby


Question 30 The cremaster muscle

Options for Questions 30-30

A Is derived from the external oblique muscle B Is derived from the internal oblique muscle
C Is derived from the transversus abdominis muscle D Is supplied by the femoral nerve
E Is supplied by branches of the femoral artery

A(Correct answ er: B)

Explanation
Cremaster muscle / cremasteric reflex
• Derived from internal oblique
• Supplied by cremasteric artery - branch of inferior epigastric
• Supplied by genital branch of genitor-femoral nerve
• Cremasteric reflex - cremaster muscle contracts when skin on medial aspect of thigh is stroked. Afferent - femoral
branch; Efferent - genital branch of genitor-femoral nerve

This Copy is for Dr. Mohamed ElHodiby


Question 1 Which one of the above landmarks is correctly paired with its dermatome?

Options for Questions 1-1

A Xiphoid process – T10 B Umbilicus – T12


C Simphysis pubis – L5 D Xiphoid process – T7
E Anterior superior iliac spine – S1

A(Correct answ er: D)

Explanation
Abdominal wall
Dermatomes
• Xiphoid process - T7
• Umbilicus - T10
• Pubis - L1

Question 2 Blood supply to the lateral part of the abdominal wall above the umbilicus

Options for Questions 2-2

A Superior epigastric artery B Inferior epigastric artery


C Superior mesenteric artery D Inferior mesenteric artery
E Intercostal arteries

A(Correct answ er: E)

Explanation
Blood supply – abdominal wall
• Medial aspects - Superior epigastric artery (above umbilicus - continuation of the internal thoracic artery, branch of
the first part of the subclavian artery. Enters the rectus sheath between the sternal and costal origins of the
diaphragm and descends behind rectus muscle)
• Inferior epigastric artery (below umbilicus - branch of external iliac artery just above inguinal ligament. It pierces the
transversalis fascia to enter the rectus sheath anterior to the arcuate line; runs behind rectus muscle). There is no
anastomosis between the arteries of the left and right side
• Lateral aspects - intercostals, lumbar arteries and deep circumflex iliac artery (branch of external iliac artery above
the inguinal ligament).

Question 3 The sac of this hernia is formed by the remnant of the processus vaginalis

Options for Questions 3-3

A Direct inguinal hernia B Indirect inguinal hernia


C Femoral hernia D Richcter’s hernia
E Epigastric hernia

A(Correct answ er: B)

Explanation

This Copy is for Dr. Mohamed ElHodiby


ABDOMINAL WALL MUSCLES
External oblique
ORIGIN: Outer surface of lower 8 ribs
INSERTION: Xiphoid process, linea alba, pubic crest and pubic tubercle and the anterior half of the
iliac crest
Internal oblique
• ORIGIN: Lumbar fascia, anterior 2/3 of iliac crest and lateral 2/3 of inguinal ligament
• INSERTION: Lower border of lower 3 ribs and costal cartilages, xiphoid process, linea alba and symphysis
pubis
• Lower free border gives rise to cremasteric fascia

Question 4 Insertion of internal oblique muscle

Options for Questions 4-4

A Lateral 2/3 of inguinal ligament B Lumbar fascia


C Anterior 2/3 of iliac crest D Linea alba
E Linea nigra

A(Correct answ er: D)

Explanation

ABDOMINAL WALL MUSCLES


External oblique
ORIGIN: Outer surface of lower 8 ribs
INSERTION: Xiphoid process, linea alba, pubic crest and pubic tubercle and the anterior half of the
iliac crest
Internal oblique
• ORIGIN: Lumbar fascia, anterior 2/3 of iliac crest and lateral 2/3 of inguinal ligament
• INSERTION: Lower border of lower 3 ribs and costal cartilages, xiphoid process, linea alba and symphysis
pubis
• Lower free border gives rise to cremasteric fascia

Question 5 Which one of the above descriptions of the arcuate line is correct?

Options for Questions 5-5

Insertion of the inguinal ligament on the pubic


A Lateral margins of the rectus abdominis muscle B
bone
Point where the aponeurosis of the internal
Point where the aponeurosis of the external
C oblique muscle passes anterior to the rectus D
oblique passes posterior to the rectus abdominis
abdominis
Attachment of the ileo-pectineal ligament on the
E
superior pubic ramus
A(Correct answ er: C)

Explanation
RECTUS SHEATH
Above costal margin
• Anterior wall: Aponeurosis of external oblique
• Posterior wall: thoracic wall.

This Copy is for Dr. Mohamed ElHodiby


Between costal margin and anterior superior iliac spine
• Anterior wall: Aponeurosis of external and internal oblique
• Posterior wall: Aponeurosis of internal oblique and transversus abdominis
• Note that the aponeurosis of the internal oblique splits to enclose the rectus abdominis
Between anterior superior iliac spine and pubis
• Anterior wall: Aponeuroses of external, internal oblique and transversus
• Posterior wall: Transversalis fascia
• Arcuate line : Site where the aponeuroses of the posterior wall pass anterior to the rectus at the level of the anterior
superior iliac spine. The inferior epigastric artery enters the rectus sheath at this point and lies posterior to the rectus
abdominis. Pyramidalis lies within the rectus sheath.

Anterior wall of the rectus sheath between the costal margin and the anterior superior iliac
Question 6
spine
Options for Questions 6-6

A Aponeurosis of external oblique B Aponeurosis of internal oblique


Aponeurosie of internal oblique and transversus
C D Aponeuroses of external and internal oblique
abdominis
E Aponeurosis of transversus abdominis
A(Correct answ er: D)

Explanation
RECTUS SHEATH
Above costal margin
• Anterior wall: Aponeurosis of external oblique
• Posterior wall: thoracic wall.
Between costal margin and anterior superior iliac spine
• Anterior wall: Aponeurosis of external and internal oblique
• Posterior wall: Aponeurosis of internal oblique and transversus abdominis
• Note that the aponeurosis of the internal oblique splits to enclose the rectus abdominis
Between anterior superior iliac spine and pubis
• Anterior wall: Aponeuroses of external, internal oblique and transversus
• Posterior wall: Transversalis fascia
• Arcuate line : Site where the aponeuroses of the posterior wall pass anterior to the rectus at the level of the anterior
superior iliac spine. The inferior epigastric artery enters the rectus sheath at this point and lies posterior to the rectus
abdominis. Pyramidalis lies within the rectus sheath.

Question 7 Posterior wall of the rectus sheath above the costal margin

Options for Questions 7-7

A Aponeurosis of external oblique B Aponeurosis of internal oblique


C Thoracic wall D Aponeuroses of external and internal oblique
E Aponeurosis of transversus abdominis

A(Correct answ er: C)

Explanation
RECTUS SHEATH
Above costal margin
• Anterior wall: Aponeurosis of external oblique
• Posterior wall: thoracic wall.
Between costal margin and anterior superior iliac spine

This Copy is for Dr. Mohamed ElHodiby


• Anterior wall: Aponeurosis of external and internal oblique
• Posterior wall: Aponeurosis of internal oblique and transversus abdominis
• Note that the aponeurosis of the internal oblique splits to enclose the rectus abdominis
Between anterior superior iliac spine and pubis
• Anterior wall: Aponeuroses of external, internal oblique and transversus
• Posterior wall: Transversalis fascia
• Arcuate line : Site where the aponeuroses of the posterior wall pass anterior to the rectus at the level of the anterior
superior iliac spine. The inferior epigastric artery enters the rectus sheath at this point and lies posterior to the rectus
abdominis. Pyramidalis lies within the rectus sheath.

Question 8 The origin of psoas major muscle

Options for Questions 8-8

A Roots of transverse processes of T12 – L5 B Roots of transverse processes of L1 – L5


C Roots of transverse processes of L3, 4, 5 D Roots of transverse processes of T6 – T12
E Vertebral bodies of T6 – 12

A(Correct answ er: A)

Explanation
PSOAS MAJOR
• Origin: Roots of transverse processes, sides of vertebral bodies and inter-vertebral discs T12 - L5
• Insertion: Lesser trochanter of the femur
• Nerve: Lumbar plexus
• Action: Flexes thigh on trunk. If thigh if flexed, flexes trunk on thigh.

ILIACUS
• Origin: Iliac fossa
• Insertion: Lesser trochanter of the femur
• Nerve: Femoral nerve
• Action: Flexes thigh on trunk. If thigh is flexed, flexes trunk on thigh

Question 9 Which one of the above statements about the inguinal ligament is not true?

Options for Questions 9-9

Is formed from the aponeurosis of the external


A B Is attached medially to the pubic tubercle
oblique muscle
Is formed from the aponeuroses of the external
C Forms the floor of the inguinal canal D
and internal oblique muscles
E Forms the inferior wall of the inguinal canal

A(Correct answ er: D)

Explanation
INGUINAL LIGAMENT
• Formed by the aponeurosis of the External oblique muscle
• Attached medially to the pubic tubercle

This Copy is for Dr. Mohamed ElHodiby


• Forms the inferior wall or floor of the inguinal canal

Question 10 Which one of the above does not pass through the superficial inguinal ring?

Options for Questions 10-10

A Round ligament of the uterus B Ilioinguinal nerve


C Genital branch of the genito-femoral nerve D Lateral cutaneous nerve of the thigh
E Sympathetic plexus

A(Correct answ er: D)

Explanation

INGUINAL CANAL
• 4cm long in adults
• Deep ring: - oval shaped hole in transversalis fascia, 1.3cm above inguinal ligament, mid-way between anterior
superior iliac spine and symphysis pubis. Medial relation - inferior epigastric vessels. Gives rise to the internal
spermatic fascia or the internal covering of the round ligament of the uterus
• Superficial ring:- triangular in shape, defect in external oblique aponeurosis, base formed by pubic crest, gives rise to
external spermatic fascia
• The following pass through the superficial ring: round ligament, ilioinguinal nerve, genital branch of the genitofemoral
nerve, lymphatics and sympathetic plexus
• Anterior wall - aponeurosis of external oblique, reinforced laterally by origin of internal oblique
• Posterior wall - transversalis fascia, reinforced medially by conjoint tendon - common insertion of internal oblique and
transversus to the pubic crest and pectineal line
• Floor - Inguinal ligament
• Roof - internal oblique and transversus abdominis
• The femoral neurovascular bundle and lateral cutaneous nerve of the thigh pass beneath the inguinal ligament.
• The femoral branch of the genitofemoral nerve enters the thigh behind the middle of the inguinal ligament.
• The ilioinguinal nerve enters the thigh through the superficial inguinal ring.
• The superficial epigastric vessels cross the inguinal ligament

This Copy is for Dr. Mohamed ElHodiby


Question 1 The main oestrogen produced by the placenta

Options for Questions 1-1

A Oestrone B Oestradiol
C Oestriol D Ethinyl-oestradiol
E Oestradiol glucuronate

A(Correct answ er: C)

Explanation
PLACENTAL OESTROGENS
Mainly oestriol, but also oestradiol and oestrone in smaller amounts. Oestriol is produces from DHES-sulphate from
fetal zone of the fetal adrenal gland and also from the maternal adrenals. Fetal DHEA-S is initially hydroxylated by the
fetal liver

Question 2 The mechanism of amino acid transfer across the placenta

Options for Questions 2-2

A Simple diffusion B Facilitated diffusion


C Active transport D Secondary active transport
E Bulk transfer

A(Correct answ er: D)

Explanation
CARBOHYDRATES & AMINO ACIDS
• Glucose transport across the microvillous and basal plasma membranes is by facilitated diffusion.
• This is dependent on glucose concentration gradient and the activity of transport proteins within the placenta. In
theory, facilitated diffusion can be saturated.
• In practice, the glucose transfer capacity (especially of the microvillous plasma membrane) is so large that it would
not approach saturation under in-vivo conditions
• Fetal amino acid concentrations are generally higher than maternal levels
• Transfer of amino acids from the mother to the fetus is therefore against a concentration gradient and energy (ATP) is
required for this process
• This energy is provided by coupling the transfer of amino acids (up a concentration gradient) to the transfer of sodium
ions (down a concentration gradient) - secondary active transport. These are called Na+-dependent transporters. The
sodium concentration gradient is maintained by Na+K+ATPase.
• Other amino acid transport systems are, however, Na+-independent

Question 3 The mechanism of transfer of carbondioxide across the placenta

Options for Questions 3-3

A Simple diffusion B Facilitated diffusion


C Active transport D Secondary active transport
E Bulk transfer

A(Correct answ er: A)

Explanation
CARBOHYDRATES & AMINO ACIDS
• Glucose transport across the microvillous and basal plasma membranes is by facilitated diffusion.

This Copy is for Dr. Mohamed ElHodiby


• This is dependent on glucose concentration gradient and the activity of transport proteins within the placenta. In
theory, facilitated diffusion can be saturated.
• In practice, the glucose transfer capacity (especially of the microvillous plasma membrane) is so large that it would
not approach saturation under in-vivo conditions
• Fetal amino acid concentrations are generally higher than maternal levels
• Transfer of amino acids from the mother to the fetus is therefore against a concentration gradient and energy (ATP) is
required for this process
• This energy is provided by coupling the transfer of amino acids (up a concentration gradient) to the transfer of sodium
ions (down a concentration gradient) - secondary active transport. These are called Na+-dependent transporters. The
sodium concentration gradient is maintained by Na+K+ATPase.
• Other amino acid transport systems are, however, Na+-independent.

Question 4 Which one of the above is not readily transferred across the placenta?

Options for Questions 4-4

A Amino acids B Glucose


C Thyroxine D IgG
E Ca2+

A(Correct answ er: C)

Explanation
Molecule Placental transfer

Testosterone Minimal transfer - androgens aromatised by placenta. Very high maternal androgen
concentration may virilise female fetus

Ca2+, Mg2+ Active transfer against concentration gradient

PTH, Calcitonin Not transferred

Vitamin D Good transfer

IgA Minimal passive transfer

IgG Good active and active transfer from 7 weeks gestation

IgM No transfer

Glucose Facilitated diffusion - excellent transfer

Amino acids Active transport - excellent transfer

Free fatty acids Very limited transfer - essential fatty acids only

Ketone bodies Excellent transfer - diffusion

Insulin,
No transfer
glucagon
Thyroid
Poor transfer - diffusion
hormone

TRH Excellent transfer

This Copy is for Dr. Mohamed ElHodiby


Iodine and
Excellent transfer
thioamides
Cortisol &
Excellent transfer
aldosterone

ACTH No transfer

Question 5 Which one of the above is readily transferred across the placenta?
Options for Questions 5-5

A Testosterone B Parathyroid hormone


C Vitamin D D IgM
E Calcitonin

A(Correct answ er: C)

Explanation
Molecule Placental transfer

Testosterone Minimal transfer - androgens aromatised by placenta. Very high maternal androgen
concentration may virilise female fetus

Ca2+, Mg2+ Active transfer against concentration gradient

PTH, Calcitonin Not transferred

Vitamin D Good transfer

IgA Minimal passive transfer

IgG Good active and active transfer from 7 weeks gestation

IgM No transfer

Glucose Facilitated diffusion - excellent transfer

Amino acids Active transport - excellent transfer

Free fatty acids Very limited transfer - essential fatty acids only

Ketone bodies Excellent transfer - diffusion

Insulin,
No transfer
glucagon
Thyroid
Poor transfer - diffusion
hormone

TRH Excellent transfer

Iodine and
Excellent transfer
thioamides
Cortisol &
Excellent transfer
aldosterone

ACTH No transfer

This Copy is for Dr. Mohamed ElHodiby


Question 6 Monozygotic twins cannot have this type of placentation

Options for Questions 6-6

A Monochorionic monoamniotic B Dichorionic diamniotic


C Dichorionic monoamniotic D Monochorionic diamniotic
E None of the above

A(Correct answ er: E)

Explanation
MULTIPLE PREGNANCY?Twins
UK data 2007: ~ 1 in 65 (1.5%) pregnancies were twins
Prevalence varies world-wide, being lowest in Japan and highest in Nigeria
Incidence of monozygotic twins relatively constant world-wide at ~ 3.5 per 1,000 births
Incidence of dizygotic twins varies widely
Dizygotic twins
• Fertilisation of two oocytes by different sperm
• Dizygotic twins have no more resemblance than brothers / sisters of different ages
• Both zygotes implant independently in the uterus and there are two separate placentas, amniotic and chorionic sacs
• The placentas and chorionic sacs may come into close approximation and fuse.
• Dizygotic twins cannot be monochorionic and cannot be identical. They are always dichorionic and diamniotic
Monozygotic twins
• A single ovum is fertilised and splits into two at different stages of development
• Earliest separation occurs at the 2 cell stage producing two zygotes which enter the uterus independently. Each
embryo would have its own placenta and chorionic sac, forming dichorionic diamniotic twins
• In the majority of cases, splitting occurs in the early blastocyst stage. The inner cell mass splits into two, producing
two embryos with a common placenta and a common chorionic cavity but separate amniotic cavities. These form
monochorionic diamniotic twins
• Rarely, splitting occurs at the stage of the bilaminar germ disc, resulting in two embryos with a common placenta,
common chorionic cavity and common amniotic cavity. These form monochorionic monoamniotic twins
• Splitting at later stages of development may result in incomplete splitting of the axial area of the germ disc, resulting
in conjoint twins

Question 7 With respect to the sigmoid colon

Options for Questions 7-7

Arterial supply is from the superior mesenteric Venous drainage is into the portal vein via the
A B
artery superior mesenteric vein
Nerve supply is from the inferior mesenteric
C Lymphatic drainage is to the para-aortic nodes D
plexus
The sigmoid colon lies anterior to the sacrum and
E
rectum

A(Correct answ er: E)

Explanation
SIGMOID COLON
• Continuous with the descending colon in front of the LEFT external iliac artery, and with the rectum at the
level of S3
• Intra-peritoneal with an inverted V-shaped mesentery - one limb running on the medial side of the left
external iliac artery and the other from the bifurcation of the left common iliac artery to S3. The recess of the
pelvic mesocolon is located at the apex of the V and the left ureter lies beneath it
• Related anteriorly to the bladder in the MALE and the uterus and upper part of the vagina in the female

This Copy is for Dr. Mohamed ElHodiby


• Related posteriorly to the sacrum and rectum
• Blood supply - inferior mesenteric artery
• Venous drainage - inferior mesenteric vein - to portal vein
• Lymphatics - inferior mesenteric plexus
• Nerve - inferior hypogastric plexuses

Question 8 With respect to the anatomy of the rectum

Options for Questions 8-8

Lymphatic drainage of the upper third is to the Lymphatic drainage of the middle third is to the
A B
superior mesenteric nodes superior mesenteric nodes
Lymphatic drainage of the lower third is to the Nerve supply is from the superior mesenteric
C D
inferior mesenteric nodes plexus
Venous drainage forms part of the portal-systemic
E
anastomoses

A(Correct answ er: E)

Explanation
THE RECTUM
• ~13cm long
• Upper third - covered by peritoneum on its anterior and lateral surfaces, middle third covered by peritoneum on the
anterior surface only and lower third devoid of peritoneum
• The three teniae coli of sigmoid colon come together to form a broad band of longitudinal smooth muscle on its
anterior and posterior surfaces
• Three transverse folds, two on the left and one on the right
• Anterior relations: sigmoid colon and small intestine within pouch of Douglas (upper third); Posterior surface of vagina
(lower third)
• Posterior relations: sacrum, coccyx, piriformis and coccygeus muscles, lavatory ani, sacral plexus and pelvic
sympathetic trunk
Blood supply
• Superior rectal artery - continuation of inferior mesenteric artery - main blood supply to mucosa
• Middle rectal artery - branch of internal iliac artery - muscular coat mainly
• Inferior rectal artery - branch of internal pudendal artery
• Veins correspond to arteries and form an important portal-systemic anastomosis
• Lymphatic drainage: Upper two thirds - inferior mesenteric nodes; lower third - internal iliac node
• Nerve supply - inferior hypogastric plexus

Question 9 The blood supply to the fallopian tube

Options for Questions 9-9

A Ovarian artery B Uterine artery


C Ovarian and uterine arteries D External iliac artery
E Internal iliac artery

A(Correct answ er: C)

Explanation
FALLOPIAN TUBES
• ~10cm long, 4 parts

This Copy is for Dr. Mohamed ElHodiby


• Infundibulum - funnel-shaped lateral end, projects beyond the broad ligament with fimbriae at its free end
• Ampulla - widest and longest part, site of fertilisation
• Isthmus - narrowest part, just lateral to the uterus
• Intra-mural part - pierces uterine wall
• Three coats: serous, muscular, and mucous.
• The external or serous coat is peritoneal. The middle or muscular coat consists of an external longitudinal and an
internal circular layer of smooth muscle fibers continuous with those of the uterus
• The mucosa is thrown into longitudinal folds, which in the ampulla are much more extensive than in the isthmus.
Lined by ciliated columnar epithelium
• Blood - ovarian and uterine arteries
• Lymphatics - aortic and internal iliac nodes (follow arteries)
• Nerves - inferior hypogastric plexus

Question 10 Lies posterior to the body of the uterus

Options for Questions 10-10

A Urinary bladder B Ureters


C Sigmoid colon D Rectum
E Utero-vesical pouch

A(Correct answ er: C)

Explanation
THE UTERUS
• 8cm long x 5cm wide x 2.5cm thick
• Covered by peritoneum except anteriorly below the reflection of the utero-vesical fold of peritoneum and laterally
between the layers of the broad ligament?
• Fundus - that part of the uterus above the entrance of the uterine tubes?
• Cavity - triangular in coronal section, cleft in saggital section?
• Anteverted uterus - long axis of uterine body at 90degrees to long axis of vagina?
• Ante-flexed uterus - long axis of the body of the uterus bent forward at the level of the internal os?
• Retroverted uterus - body and fundus bent backwards on the vagina to lie within the pouch of Douglas?
• Anterior relations: utero-vesical pouch and superior surface of bladder?
• Posterior relations: Pouch of Douglas, sigmoid colon and coils of ileum?
• Lateral relations: uterine vessels, ureter?
• Nerve - inferior hypogastric plexuses (Parasympathetic via the pelvic splanchnic nerves, sympathetic via the lumbar
splanchnic nerves). Pain sensation is transmitted via the sympathetic nerves and the lumbar splanchnic nerves??
Lymph drainage:
• Fundus - accompany ovarian artery to para-aortic nodes at the level of L1
• Body and cervix - internal and external iliac nodes

Question 11 With respect to the innervation and support of the cervix

Options for Questions 11-11

The round ligament provides the most important The broad ligament provides the most important
A B
support to the cervix support to the cervix
The endocervix has autonomic but no sensory Sensory innervation of the cervix is via S2,3,4
C D
innervation spinal segments
The ectocervix has sensory but minimal
E
autonomic innervation

This Copy is for Dr. Mohamed ElHodiby


A(Correct answ er: D)

Explanation
?CERVIX
• Lower, narrow portion of the uterus, connected to the uterine fundus by the uterine isthmus - upper limit is the internal
os. Made up mainly of fibrous tissue with very little smooth muscle
• Protrudes through the upper anterior vaginal wall
• Approximately half its length is visible in the vagina (vaginal cervix), the rest being above the vagina (supra-vaginal
cervix)
• The vaginal cervix ~3 cm long and 2.5 cm wide. Size and shape varies widely with age, hormonal state, and parity -
bulkier and the external with a wider and more slit-like external os in multiparous women.
• Ectocervix - portion of the cervix beyond the external os - lined by stratified squamous non-keratinising epithelium.
• Endocervical canal - links external and internal os - lined by columnar epithelium
• The squamo-columnar junction - variable location - high up the endocervical canal before puberty and in the post-
menopausal women. Site of origin of squamous cell carcinoma of the cervix
• The external os is bounded by two lips, an anterior and a posterior, of which the anterior is the shorter and thicker,
although due to the slope of the cervix, it projects lower than the posterior. Both lips are in contact with the posterior
vaginal wall
• The supravaginal cervix is separated in front from the bladder by fibrous tissue (parametrium), which extends also on
to its sides and laterally between the layers of the broad ligaments. Not covered by peritoneum on the anterior aspect
• The uterine arteries reach the margins of the cervix within the parametrium
• The ureter runs downward and forward 2 cm lateral to the supravaginal cervix
• Posteriorly, the supravaginal cervix is covered by peritoneum, which extends on to the posterior vaginal wall, when it
is reflected on to the rectum, forming the Pouch of Douglas which may contain coils of small intestine.
• The vaginal cervix projects free into the anterior wall of the vagina between the anterior and posterior fornices.
?Blood supply
• Uterine artery, branch of internal iliac
• Cervical and vaginal branches supply the cervix and upper vagina.
• The cervical branches of the uterine arteries descend on the lateral aspects of the cervix at 3 and 9 o'clock. Venous
drainage parallels the arterial supply, eventually emptying into the hypogastric venous plexus.
?Lymphatics
• Regional lymph nodes for the cervix include: paracervical, parametrial, presacral, sacral, external iliac, common iliac,
hypogastric (obturator), internal iliac.

Question 12 Somatic sensory innervation of the lower third of the vagina

Options for Questions 12-12

A Pelvic splanchnic nerves B Lumbar splanchnic nerves


C Pudendal nerve D Superficial femoral nerve
E Obturator nerve

A(Correct answ er: C)

Explanation
THE VAGINA
• ~8cm long, axis directed upwards and backwards from the vulva. Posterior wall longer than anterior wall
• Lined by stratified squamous epithelium which undergoes changes during the menstrual cycle. Does not secrete
mucus
• Has anterior and posterior walls which are normally in apposition, and four fornices (anterior, posterior, left and right
lateral)
• Upper half lies above the level of the pelvic floor
?Relations
• Anterior: bladder, urethra
• Posterior: upper third - pouch of Douglas; middle third - ampulla of the rectum; lower third - perineal body

This Copy is for Dr. Mohamed ElHodiby


• Lateral: upper part - ureter, middle part - anterior fibres of levator ani; lower part - uro-genital diaphragm and the bulb
of the vestibule
• Blood: vaginal artery, branch of internal iliac artery
• Lymphatics: upper third - internal and external iliac nodes; middle third - internal iliac nodes; lower third - superficial
inguinal nodes
• Nerve:Upper two thirds - inferior hypogastric plexuses. Parasympathetic from pelvic splanchnic nerves, sympathetic
from lumbar splanchnic nerves (L1 & 2)
• Autonomic innervation to the lower third - pudendal nerve
• Somatic sensation is present mainly in the lower third and is carried by the pudendal nerve

Question 13 Parasympathetic supply to the lower third of the vagina

Options for Questions 13-13

A Pelvic splanchnic nerves B Lumbar splanchnic nerves


C Pudendal nerve D Superficial femoral nerve
E Obturator nerve

A(Correct answ er: C)

Explanation
THE VAGINA
• ~8cm long, axis directed upwards and backwards from the vulva. Posterior wall longer than anterior wall
• Lined by stratified squamous epithelium which undergoes changes during the menstrual cycle. Does not secrete
mucus
• Has anterior and posterior walls which are normally in apposition, and four fornices (anterior, posterior, left and right
lateral)
• Upper half lies above the level of the pelvic floor
?Relations
• Anterior: bladder, urethra
• Posterior: upper third - pouch of Douglas; middle third - ampulla of the rectum; lower third - perineal body
• Lateral: upper part - ureter, middle part - anterior fibres of levator ani; lower part - uro-genital diaphragm and the bulb
of the vestibule
• Blood: vaginal artery, branch of internal iliac artery
• Lymphatics: upper third - internal and external iliac nodes; middle third - internal iliac nodes; lower third - superficial
inguinal nodes
• Nerve:Upper two thirds - inferior hypogastric plexuses. Parasympathetic from pelvic splanchnic nerves, sympathetic
from lumbar splanchnic nerves (L1 & 2)
• Autonomic innervation to the lower third - pudendal nerve
• Somatic sensation is present mainly in the lower third and is carried by the pudendal nerve

Question 14 Supports the upper third of the vagina

Options for Questions 14-14

A Levator ani muscle B Broad ligament


C Urogenital diaphragm D Perineal body
E Round ligament

A(Correct answ er: A)

Explanation
VAGINAL SUPPORT

This Copy is for Dr. Mohamed ElHodiby


• The vagina is supported by
• Levator ani muscles, transverse cervical, pubo-cervical and utero-sacral ligaments in its upper part
• The uro-genital diaphragm in its middle part
• The perineal body in its lower part

Question 15 The shape of the pelvic outlet in females

Options for Questions 15-15

A Oval shaped B Circular shaped


C Diamond shaped D Heart shaped
E Quadrilateral shaped

A(Correct answ er: C)

Explanation
TRUE PELVIS
That part of the pelvis between the pelvic inlet and the pelvic outlet
The pelvic inlet is oval in shape in the Gynaecoid pelvis and has the following borders:
1) Posterior: Sacral promontory?2) Lateral: Iliopectineal line?3)Anterior: Symphysis pubis
The pelvic outlet is diamond shaped with the following borders:
• Posterior: Coccyx
• Lateral: Ischial tuberosities
• Anterior: Pubic arch formed by the simphysis pubis and the ischio-pubic rami
• Part of the lateral border is formed by the sacro-spinous and sacro-tuberous ligaments which convert the greater and
lesser sciatic notches into the greater and lesser sciatic foramina
• The pelvic cavity has a shallow anterior wall and a deeper posterior wall. The ischial spines are at the level of the
mid-cavity
• The acetabulum is formed from the ilium, ischium and pubic bones

Question 16 Which one of the above statements about the female pelvis is correct?

Options for Questions 16-16

The cavity of the female pelvis is deeper than that The sacrum is longer in the female pelvis than in
A B
of the male pelvis the male pelvis
The sacrum is wider in the female pelvis than in The ischial spines are more prominent in the
C D
the male pelvis female pelvis than in the male pelvis
The sciatic notches are narrower in the female
E
pelvis than in the male pelvis

A(Correct answ er: C)

Explanation
?Differences between the Male and Female Pelvis?The female pelvis:
• Less massive
• The anterior iliac spines more widely separated - greater lateral prominence of the hips.
• Wider pelvic inlet - both antero-posterior and transverse diameters
• Pelvic inlet more circular
• More shallow
• The sacrum is shorter wider, and its upper part is less curved
• The obturator foramina are triangular in shape and smaller in size

This Copy is for Dr. Mohamed ElHodiby


• The outlet is larger and the coccyx more movable.
• The sciatic notches are wider and shallower
• The ischial spines are less prominent.
• The pubic symphysis is less deep, and the pubic arch is wider and more rounded than in the male?

Question 17 Which one of the above does not contribute to the sacral plexus?

Options for Questions 17-17

A Anterior ramus of L4 B Anterior ramus of L5


C Anterior ramus of S2 D Anterior ramus of S4
E Anterior ramus of S5

A(Correct answ er: E)

Explanation
SACRAL PLEXUS
Formed from anterior rami of L4&5 (lumbosacral trunk) and anterior rami of S1,2,3&4
Related anteriorly to internal iliac vessels + branches and the rectum
Related posteriorly to the piriformis muscle

Branches?To lower limb - leave the pelvis through GREATER sciatic foramen
• Sciatic nerve - L4,5, S1,2&3; largest nerve in the body
• Superior gluteal nerve - gluteus medius, minimus and tensor fascia lata
• Inferior gluteal nerve - gluteus maximus
• Nerve to obturator internus - also supplies superior gamellus muscle
• Nerve to quadratus femoris - also supplies inferior gamellus muscle
• Posterior cutaneous nerve of the thigh

Question 18 The pudendal nerve

Options for Questions 18-18

A Enters the pelvis through the greater sciatic notch B Leaves the pelvis through the lesses sciatic notch
Enters the perineum through the lesser sciatic
C D Is formed from branches of L2,3 4
foramen
E Supplies the gluteus maximus muscle

A(Correct answ er: C)

Explanation
Branches to pelvic organs
• Pudendal nerve - S2,3&4 - leaves the pelvis through the greater sciatic foramen and enters the perineum through the
lesser sciatic foramen
• Nerve to piriformis
• Pelvic splanchnic nerves - S2,3&4 - sacral part of parasympathetic system
• Perforating cutaneous nerve - skin of lower medial part of buttock

Question 19 Which one of the above organs is correctly paired with its parasympathetic nerve supply?

Options for Questions 19-19

This Copy is for Dr. Mohamed ElHodiby


A Urinary bladder – lumbar splanchnic nerves B Small intestine – pelvic splanchnic nerves
C Descending colon – vagus nerve D Vagina – lumbar splanchnic nerves
E Ovary – vagus nerve

A(Correct answ er: E)

Explanation
PELVIC SPLANCHNIC NERVES
• Formed from the ventral (anterior) primary rami of S2 through S4.
• These are the ways in which parasympathetic neurons reach the hypogastric plexus, and therefore the pelvic viscera
and distal colon.
• The parasympathetic part of the autonomic nervous system is the "craniosacral" part. Parasympathetic innervation to
most of the gut comes from the "cranio-" half of that, i.e., the vagus nerve. The rest, to colon distal to the splenic
flexure and to pelvic viscera, is from the "-sacral" half, via the pelvic splanchnic nerves
• There are thoracic, lumbar, sacral, and pelvic splanchnic nerves.
• "Splanchnic" refers to nerves that supply viscera.
• Thoracic, lumbar and sacral splanchnic nerves emerge from sympathetic ganglia and carry sympathetic fibers
• Pelvic splanchnic nerves are parasympathetic
• Contain pre-ganglionic fibres
• Join inferior hypogastric plexus
• Some fibres ascend to the superior hypogastric and eventually inferior mesenteric plexus and supply the hind-gut
• Provide parasympathetic supply to the pelvic viscera
• Afferent impulses from the pelvic viscera are transmitted mainly by the sympathetic pathway
• Parasympathetic nerves innervate detrusor and internal sphincter via the pelvic splanchnic nerves (S2,3,4) and also
innervate the external sphincter via the pudendal nerve - initiate micturiction by inhibiting internal sphincter activity
and stimulating detrusor contraction
• Parasympathetic supply to the ovary is from the vagus nerve. Sympathetic innervation is from the L1/2 segment
• Pain from uterine contractions goes back to T10-L1. This means that uterine contraction pain is sympathetic. But for
the lower portion of the uterus and upper vagina (the cervix) - it is parasympathetic back to the pelvic splanchnic
nerves

Question 20 With respect to the anatomy of the female external genitalia

Options for Questions 20-20

The labia majora fuse posteriorly to form the The vestibule is bound laterally by the labia
A B
fourchette majora
The fourchette is located at the apex of the
C The clitoris is located at the base of the vestibule D
vestibule
The urethra opens within the vestibule posterior to
E
the clitoris

A(Correct answ er: E)

Explanation
EXTERNAL GENITALIA
• Labia majora: prominent hair-bearing folds of skin extending from the mons pubis to fuse posteriorly in the
mid-line
• Labia minora: hairless folds of skin within the labia majora; unite posteriorly to form the fourchette; split
anteriorly to enclose the clitoris, forming an anterior prepuce and a posterior frenulum
• Vestibule: triangular area bounded laterally by the labia minora with the clitoris at its apex and the fourchette
at its base
• The urethra opens within the vestibule posterior to the clitoris

This Copy is for Dr. Mohamed ElHodiby


Question 21 The lower half of the anal canal is sensitive to these stimuli

Options for Questions 21-21

A Stretch B Stretch and touch


C stretch, touch and temperature D stretch, touch, temperature and pain
E stretch and pain

A(Correct answ er: D)

Explanation
THE ANAL CANAL
• ~4cm long, extends downwards and backwards from the rectal ampulla to the anus
• LATERAL walls kept in apposition by the levator ani and anal sphincter except during defecation
• Anterior relations: perineal body, urogenital diaphragm and perineal body
• Posterior relations: anococcygeal body and the coccyx
?UPPER HALF
• Derived from hind-gut endoderm
• Lined by columnar epithelium
• Thrown into vertical folds called anal columns
• Autonomic supply from the inferior hypogastric plexus, sensitive to stretch only
• Blood - superior rectal branch of inferior mesenteric artery
• Lymphatics - inferior mesenteric nodes
• Dentate line - boundary between endodermal and ectodermal origin of anal canal
?LOWER HALF
• Derived from the ectoderm of the proctodeum
• Lined by stratified squamous epithelium
• No anal columns
• Blood - inferior rectal artery, branch of internal pudendal artery
• Lymphatics - medial group of superficial inguinal nodes
• Nerve - inferior rectal nerve - sensitive to temperature, pain, touch and pressure

Question 22 Which one of the above most accurately describes the Dentate line?

Options for Questions 22-22

A Mucosal fold overlying inferior rectal artery B Mucosal fold over anal columns
Boundary between ectodermal and mesodermal Boundary between ectodermal and endodermal
C D
origin of anal canal origin of anal canal
Boundary between mid-gut and hind-gut origins of
E
the anal canal

A(Correct answ er: D)

Explanation
THE ANAL CANAL
• ~4cm long, extends downwards and backwards from the rectal ampulla to the anus
• LATERAL walls kept in apposition by the levator ani and anal sphincter except during defecation
• Anterior relations: perineal body, urogenital diaphragm and perineal body
• Posterior relations: anococcygeal body and the coccyx
?UPPER HALF
• Derived from hind-gut endoderm
• Lined by columnar epithelium
• Thrown into vertical folds called anal columns

This Copy is for Dr. Mohamed ElHodiby


• Autonomic supply from the inferior hypogastric plexus, sensitive to stretch only
• Blood - superior rectal branch of inferior mesenteric artery
• Lymphatics - inferior mesenteric nodes
• Dentate line - boundary between endodermal and ectodermal origin of anal canal
?LOWER HALF
• Derived from the ectoderm of the proctodeum
• Lined by stratified squamous epithelium
• No anal columns
• Blood - inferior rectal artery, branch of internal pudendal artery
• Lymphatics - medial group of superficial inguinal nodes
• Nerve - inferior rectal nerve - sensitive to temperature, pain, touch and pressure

Question 23 Which one of the above statements about the anal sphincter is true?

Options for Questions 23-23

The internal sphincter is composed of skeletal The internal sphincter lies deep to the outer layer
A B
muscle of longitudinal smooth muscle
The external sphincter is composed of smooth
C D The external sphincter is inserted onto the coccyx
muscle
The internal sphincter is inserted onto the perineal
E
body anteriorly

A(Correct answ er: B)

Explanation
ANAL SPHINCTER??
Internal
• Thickening of the inner circular layer of smooth muscle in the upper half of the anal canal
• Lies deep to the outer layer of longitudinal smooth muscle
?External
• Skeletal muscle
• Sub-cutaneous part - lower half of anal canal, no bony attachments
• Superficial part - attached to the perineal body anteriorly and the anococcygeal body posteriorly
• Deep part - no bony attachments, blends with puborectalis

Question 24 Main venous drainage of the skin of the abdominal wall above the level of the umbilicus

Options for Questions 24-24

A Internal thoracic vein B Lateral thoracic vein


C Azygous vein D Great saphenous vein
E Portal vein

A(Correct answ er: B)

Explanation
Venous drainage: abdominal wall
• Above umbilicus - axillary vein via lateral thoracic vein
• Below umbilicus - femoral vein via superficial epigastric and great saphenous vein
• Few para-umbilical veins drain into the portal vein via the ligamentun teres, forming a portal-systemic anastomosis.
Distended veins may indicate portal hypertension

This Copy is for Dr. Mohamed ElHodiby


• Deep veins follow the arteries with the posterior intercostals veins draining into the azygous vein and the lumbar
veins into the inferior vena cava
?Lymphatics
• Above umbilicus - anterior axillary nodes
• Below umbilicus - superficial inguinal nodes
• Deep lymphatics follow the arteries into the internal thoracic, external iliac, posterior mediastinal and para-aortic
nodes

Question 25 Nerve supply to the internal oblique muscle

Options for Questions 25-25

A Upper 6 thoracic nerves B Lower 6 thoracic nerves


C Upper 3 lumbar nerves D Lower 3 lumbar nerves
E 12th thoracic nerve

A(Correct answ er: B)

Explanation
NERVES
• External oblique, internal oblique and transversus: Lower 6 thoracic nerves and L1 (ileoinguinal and ileohypogastric
nerves)
• Rectus abdominis: Lower 6 thoracic nerves
• Pyramidalis: T12
FUNCTION
• External / internal oblique / transversus - laterally flex and rotate the trunk, relax during inspiration to accommodate
abdominal viscera, contract during micturiction, defecation and vomiting.
• Rectus abdominis - flexes the trunk and stabilises the pelvis

Question 26 Which one of the above descriptions of the arcuate line is correct?

Options for Questions 26-26

Insertion of the inguinal ligament on the pubic


A Lateral margins of the rectus abdominis muscle B
bone
Point where the aponeurosis of the internal
Point where the aponeurosis of the external
C oblique muscle passes anterior to the rectus D
oblique passes posterior to the rectus abdominis
abdominis
Attachment of the ileo-pectineal ligament on the
E
superior pubic ramus

A(Correct answ er: C)

Explanation
RECTUS SHEATH
Above costal margin
• Anterior wall: Aponeurosis of external oblique
• Posterior wall: thoracic wall.
Between costal margin and anterior superior iliac spine
• Anterior wall: Aponeurosis of external and internal oblique
• Posterior wall: Aponeurosis of internal oblique and transversus abdominis
• Note that the aponeurosis of the internal oblique splits to enclose the rectus abdominis
Between anterior superior iliac spine and pubis
• Anterior wall: Aponeuroses of external, internal oblique and transversus

This Copy is for Dr. Mohamed ElHodiby


• Posterior wall: Transversalis fascia
• Arcuate line : Site where the aponeuroses of the posterior wall pass anterior to the rectus at the level of the anterior
superior iliac spine. The inferior epigastric artery enters the rectus sheath at this point and lies posterior to the rectus
abdominis. Pyramidalis lies within the rectus sheath.

Question 27 The main action of the psoas major muscle

Options for Questions 27-27

A Medially rotate the hip joint B Laterally rotate the hip joint
C Flex the hip joint D Adduct the hip joint
E Adduct the hip joint

A(Correct answ er: C)

Explanation
PSOAS MAJOR
• Origin: Roots of transverse processes, sides of vertebral bodies and inter-vertebral discs T12 - L5
• Insertion: Lesser trochanter of the femur
• Nerve: Lumbar plexus
• Action: Flexes thigh on trunk. If thigh if flexed, flexes trunk on thigh.

ILIACUS
• Origin: Iliac fossa
• Insertion: Lesser trochanter of the femur
• Nerve: Femoral nerve
• Action: Flexes thigh on trunk. If thigh is flexed, flexes trunk on thigh

Question 28 Which one of the above features of the inguinal canal is correct?

Options for Questions 28-28

A Anterior wall – aponeurosis of internal oblique B Posterior wall – aponeurosis of rectus abdominis
C Roof – external oblique muscle D Floor – inguinal ligament
E Roof – transversalis fascia

A(Correct answ er: D)

Explanation
INGUINAL CANAL
• 4cm long in adults
• Deep ring: - oval shaped hole in transversalis fascia, 1.3cm above inguinal ligament, mid-way between anterior
superior iliac spine and symphysis pubis. Medial relation - inferior epigastric vessels. Gives rise to the internal
spermatic fascia or the internal covering of the round ligament of the uterus
• Superficial ring:- triangular in shape, defect in external oblique aponeurosis, base formed by pubic crest, gives rise to
external spermatic fascia
• The following pass through the superficial ring: round ligament, ilioinguinal nerve, genital branch of the genitofemoral
nerve, lymphatics and sympathetic plexus
• Anterior wall - aponeurosis of external oblique, reinforced laterally by origin of internal oblique
• Posterior wall - transversalis fascia, reinforced medially by conjoint tendon - common insertion of internal oblique and
transversus to the pubic crest and pectineal line
• Floor - Inguinal ligament

This Copy is for Dr. Mohamed ElHodiby


• Roof - internal oblique and transversus abdominis
• The femoral neurovascular bundle and lateral cutaneous nerve of the thigh pass beneath the inguinal ligament.
• The femoral branch of the genitofemoral nerve enters the thigh behind the middle of the inguinal ligament.
• The ilioinguinal nerve enters the thigh through the superficial inguinal ring.
• The superficial epigastric vessels cross the inguinal ligament

Question 29 Which one of the above is correct?

Options for Questions 29-29

Superficial inguinal ring – gives rise to internal


A Superficial inguinal ring – oval in shape B
spermatic fascia
Superficial inguinal ring – defect in aponeurosis of Deep inguinal ring – gives rise to external
C D
external oblique spermatic fascia
E Deep inguinal ring – 1.3cm above pubic tubercle

A(Correct answ er: C)

Explanation
INGUINAL CANAL
• 4cm long in adults
• Deep ring: - oval shaped hole in transversalis fascia, 1.3cm above inguinal ligament, mid-way between anterior
superior iliac spine and symphysis pubis. Medial relation - inferior epigastric vessels. Gives rise to the internal
spermatic fascia or the internal covering of the round ligament of the uterus
• Superficial ring:- triangular in shape, defect in external oblique aponeurosis, base formed by pubic crest, gives rise to
external spermatic fascia
• The following pass through the superficial ring: round ligament, ilioinguinal nerve, genital branch of the genitofemoral
nerve, lymphatics and sympathetic plexus
• Anterior wall - aponeurosis of external oblique, reinforced laterally by origin of internal oblique
• Posterior wall - transversalis fascia, reinforced medially by conjoint tendon - common insertion of internal oblique and
transversus to the pubic crest and pectineal line
• Floor - Inguinal ligament
• Roof - internal oblique and transversus abdominis
• The femoral neurovascular bundle and lateral cutaneous nerve of the thigh pass beneath the inguinal ligament.
• The femoral branch of the genitofemoral nerve enters the thigh behind the middle of the inguinal ligament.
• The ilioinguinal nerve enters the thigh through the superficial inguinal ring.
• The superficial epigastric vessels cross the inguinal ligament

Question 30 The cremaster muscle

Options for Questions 30-30

A Is derived from the external oblique muscle B Is derived from the internal oblique muscle
C Is derived from the transversus abdominis muscle D Is supplied by the femoral nerve
E Is supplied by branches of the femoral artery

A(Correct answ er: B)

Explanation

This Copy is for Dr. Mohamed ElHodiby


Cremaster muscle / cremasteric reflex
• Derived from internal oblique
• Supplied by cremasteric artery - branch of inferior epigastric
• Supplied by genital branch of genitor-femoral nerve
• Cremasteric reflex - cremaster muscle contracts when skin on medial aspect of thigh is stroked. Afferent - femoral
branch; Efferent - genital branch of genitor-femoral nerve

This Copy is for Dr. Mohamed ElHodiby


Question 1 With respect to the sigmoid colon

Options for Questions 1-1

Arterial supply is from the superior mesenteric Venous drainage is into the portal vein via the
A B
artery superior mesenteric vein
Nerve supply is from the inferior mesenteric
C Lymphatic drainage is to the para-aortic nodes D
plexus
The sigmoid colon lies anterior to the sacrum and
E
rectum

A(Correct answ er: E)

Explanation
SIGMOID COLON
• Continuous with the descending colon in front of the LEFT external iliac artery, and with the rectum at the
level of S3
• Intra-peritoneal with an inverted V-shaped mesentery - one limb running on the medial side of the left
external iliac artery and the other from the bifurcation of the left common iliac artery to S3. The recess of the
pelvic mesocolon is located at the apex of the V and the left ureter lies beneath it
• Related anteriorly to the bladder in the MALE and the uterus and upper part of the vagina in the female
• Related posteriorly to the sacrum and rectum
• Blood supply - inferior mesenteric artery
• Venous drainage - inferior mesenteric vein - to portal vein
• Lymphatics - inferior mesenteric plexus
• Nerve - inferior hypogastric plexuses

Question 2 Which one of the above statements regarding the rectum is true?

Options for Questions 2-2

Middle third is covered by peritoneum on the


A Upper third has a mesentery B
anterior and lateral surfaces
C Lower third is devoid of peritoneum D Lies anterior to the pouch of Douglas
E Lies posterior to the upper third of the vagina

A(Correct answ er: C)

Explanation
THE RECTUM
• ~13cm long
• Upper third - covered by peritoneum on its anterior and lateral surfaces, middle third covered by peritoneum on the
anterior surface only and lower third devoid of peritoneum
• The three teniae coli of sigmoid colon come together to form a broad band of longitudinal smooth muscle on its
anterior and posterior surfaces
• Three transverse folds, two on the left and one on the right
• Anterior relations: sigmoid colon and small intestine within pouch of Douglas (upper third); Posterior surface of vagina
(lower third)
• Posterior relations: sacrum, coccyx, piriformis and coccygeus muscles, lavatory ani, sacral plexus and pelvic
sympathetic trunk
Blood supply

This Copy is for Dr. Mohamed ElHodiby


• Superior rectal artery - continuation of inferior mesenteric artery - main blood supply to mucosa
• Middle rectal artery - branch of internal iliac artery - muscular coat mainly
• Inferior rectal artery - branch of internal pudendal artery
• Veins correspond to arteries and form an important portal-systemic anastomosis
• Lymphatic drainage: Upper two thirds - inferior mesenteric nodes; lower third - internal iliac node
• Nerve supply - inferior hypogastric plexus

Question 3 Nerve supply to the ovary

Options for Questions 3-3

A Aortic plexus B Superior hypogastric plexus


C Inferior hypogastric plexus D Coeliac plexus
E Superior mesenteric plexus

A(Correct answ er: A)

Explanation
THE OVARY
• 4x2 cm, attached to the posterior aspect of the broad ligament by a mesentery - the mesovarium
• Attached to the lateral pelvic wall by the suspensory ligament of the ovary (infundibulo-pelvic ligament)
• Attached to the upper part of the lateral uterine wall by the round ligament of the ovary (remnant of upper
part of the gubenaculum)
• Position variable, but usually lies within ovarian fossa in lateral pelvic wall. The ovarian fossa is bounded
superiorly by the external iliac vessels, inferiorly by the ureter and internal iliac vessels and its floor is
crossed by the obturator nerve
• The ovary is surrounded by a thin fibrous capsule - the tunica albuginae
• Blood supply - ovarian artery - branch of abdominal aorta at L1
• Venous drainage - LEFT -left ovarian vein drains into left renal vein; RIGHT - right ovarian vein drains into
inferior vena cava
• Lymphatics - para-aortic nodes?Nerve - aortic plexus??

Question 4 The level at which the ovarian artery arises from the descending aorta

Options for Questions 4-4

A T10 B T12
C L1 D L2
E L3

A(Correct answ er: C)

Explanation
THE OVARY
• 4x2 cm, attached to the posterior aspect of the broad ligament by a mesentery - the mesovarium
• Attached to the lateral pelvic wall by the suspensory ligament of the ovary (infundibulo-pelvic ligament)
• Attached to the upper part of the lateral uterine wall by the round ligament of the ovary (remnant of upper
part of the gubenaculum)
• Position variable, but usually lies within ovarian fossa in lateral pelvic wall. The ovarian fossa is bounded
superiorly by the external iliac vessels, inferiorly by the ureter and internal iliac vessels and its floor is
crossed by the obturator nerve
• The ovary is surrounded by a thin fibrous capsule - the tunica albuginae

This Copy is for Dr. Mohamed ElHodiby


• Blood supply - ovarian artery - branch of abdominal aorta at L1
• Venous drainage - LEFT -left ovarian vein drains into left renal vein; RIGHT - right ovarian vein drains into
inferior vena cava
• Lymphatics - para-aortic nodes?Nerve - aortic plexus??

Question 5 The blood supply to the fallopian tube

Options for Questions 5-5

A Ovarian artery B Uterine artery


C Ovarian and uterine arteries D External iliac artery
E Internal iliac artery

A(Correct answ er: C)

Explanation
FALLOPIAN TUBES
• ~10cm long, 4 parts
• Infundibulum - funnel-shaped lateral end, projects beyond the broad ligament with fimbriae at its free end
• Ampulla - widest and longest part, site of fertilisation
• Isthmus - narrowest part, just lateral to the uterus
• Intra-mural part - pierces uterine wall
• Three coats: serous, muscular, and mucous.
• The external or serous coat is peritoneal. The middle or muscular coat consists of an external longitudinal and an
internal circular layer of smooth muscle fibers continuous with those of the uterus
• The mucosa is thrown into longitudinal folds, which in the ampulla are much more extensive than in the isthmus.
Lined by ciliated columnar epithelium
• Blood - ovarian and uterine arteries
• Lymphatics - aortic and internal iliac nodes (follow arteries)
• Nerves - inferior hypogastric plexus

Question 6 This ligament is important in supporting the uterus above the pelvic floor

Options for Questions 6-6

A Broad ligament B Round ligament


C Infundibulo-pelvic ligament D Utero-sacral ligament
E Ileo-pectineal ligament

A(Correct answ er: D)

Explanation
LIGAMENTS
• Transverse cervical ligament: fibro-muscular condensations of pelvic fascia pass from the cervix and upper end of the
vagina to the lateral pelvic wall
• Utero-sacral ligament: cervix and upper end of vagina to the lower end of the sacrum - form two ridges on either side
of the pouch of Douglas
• Pubo-cervical ligament: cervix to posterior surface of pubis
• Round ligament: Of the ovary - from medial margin of ovary to upper part of lateral wall of uterus. Of the uterus - from
upper part of lateral uterine wall to deep inguinal ring
• The uterus is supported mainly by the tone of the pelvic floor muscles (levator ani) which are partly inserted onto the
perineal body and condensations of pelvic fascia forming the transverse cervical, pubo-cervical and utero-sacral
ligaments.

This Copy is for Dr. Mohamed ElHodiby


?
THE BROAD LIGAMENT
• Two layered fold of peritoneum extending from the lateral uterine wall to the lateral pelvic wall
• Has an upper free edge which contains the fallopian tube
• The layers of peritoneum separate inferiorly to cover the pelvic floor
• Has ovary attached to its posterior surface by the mesovarium
• Uterine artery crosses the ureter at the base (lower attached border)
• Round ligament of the uterus forms a ridge an the anterior surface
• Contains vestigial structures: epoophron and paroophron (remnant of the mesonephric system)
• Uterine and ovarian blood vessels and lymphatics run within it

Question 7 Somatic sensory innervation of the lower third of the vagina

Options for Questions 7-7

A Pelvic splanchnic nerves B Lumbar splanchnic nerves


C Pudendal nerve D Superficial femoral nerve
E Obturator nerve

A(Correct answ er: C)

Explanation
THE VAGINA
• ~8cm long, axis directed upwards and backwards from the vulva. Posterior wall longer than anterior wall
• Lined by stratified squamous epithelium which undergoes changes during the menstrual cycle. Does not secrete
mucus
• Has anterior and posterior walls which are normally in apposition, and four fornices (anterior, posterior, left and right
lateral)
• Upper half lies above the level of the pelvic floor
?Relations
• Anterior: bladder, urethra
• Posterior: upper third - pouch of Douglas; middle third - ampulla of the rectum; lower third - perineal body
• Lateral: upper part - ureter, middle part - anterior fibres of levator ani; lower part - uro-genital diaphragm and the bulb
of the vestibule
• Blood: vaginal artery, branch of internal iliac artery
• Lymphatics: upper third - internal and external iliac nodes; middle third - internal iliac nodes; lower third - superficial
inguinal nodes
• Nerve:Upper two thirds - inferior hypogastric plexuses. Parasympathetic from pelvic splanchnic nerves, sympathetic
from lumbar splanchnic nerves (L1 & 2)
• Autonomic innervation to the lower third - pudendal nerve
• Somatic sensation is present mainly in the lower third and is carried by the pudendal nerve

Question 8 Supports the middle third of the vagina

Options for Questions 8-8

A Levator ani muscle B Broad ligament


C Urogenital diaphragm D Perineal body
E Round ligament

A(Correct answ er: C)

Explanation

This Copy is for Dr. Mohamed ElHodiby


VAGINAL SUPPORT
• The vagina is supported by
• Levator ani muscles, transverse cervical, pubo-cervical and utero-sacral ligaments in its upper part
• The uro-genital diaphragm in its middle part
• The perineal body in its lower part

Question 9 The embryological origin of the dome of the bladder

Question 10 The embryological origin of the trigone of the bladder


Options for Questions 9-10

A Urachus B Yolk sac


C Mesonephric duct D Mesonephric mesoderm
E Cloaca

A(Correct answ er: B)

A(Correct answ er: C)

Explanation
THE URINARY BLADDER
• Lined by transitional epithelium - the upper part is derived from the yolk sac and is continuous with the
urachus
• The trogone is derived from the mesonephric duct and is lined by cells of mesodermal origin. It is believed
that these are later replaced by cells of endodermal origin
• The bladder wall is made up of a syncytium of smooth muscle fibres called the detrusor - contraction results
in simultaneous reduction in all dimensions of the bladder
• The female urethral sphincter has an intrinsic and an extrinsic component: the intrinsic component is made
up of epithelial, vascular and connective tissue and the rhabdosphincter which is a circular ring of striated
muscle with slow twitch fibres. The ring is well developed anteriorly, thins laterally and is virtually absent
posteriorly
• The extrinsic sphincter mechanism is made up of the striated muscles of the levator ani - mainly fast twitch
fibres

This Copy is for Dr. Mohamed ElHodiby


Question 1 Trophoblast cells that invade the decidua and fuse to form multi-nucleated giant cells

Options for Questions 1-1

A Extra-villous cytotrophoblasts B Villous cytotrophoblasts


C Endovascular trophoblasts D Extravascular trophoblasts
E Interstitial trophoblasts

A(Correct answ er: E)

Explanation
DEVELOPMENT OF THE VILLOUS TREE
• Primary villous stems become infiltrated by cytotrophoblasts between days 13-21 post-ovulation
• Villous stems are subsequently infiltrated by extra-embryonic mysenchyme which differentiates into fetal blood
vessels
• The distal parts of the villous stems are not vascularised. Here, cytotrophoblasts proliferate and spread laterally to
form a cytotrophoblastic shell, splitting the syncytiotrophoblast into a definitive syncytiotrophoblast on the fetal side
and the peripheral syncytium on the decidual side which degenerates and is replaced by fibrinoid material (Nitabuch's
layer)
• Sprouts extend from primary villous stems, initially made up of syncytiotrophoblast and then infiltrated by
cytotrophoblast and mesenchyme - these are primary stem villi and the placenta is a true villous structure by day 21
of gestation. These villi grow and divide into secondary, tertiary and terminal villi
• The villi oriented towards the uterine cavity degenerate between day 21 and the 4th month to form the chorion laeve.
The overlying decidua degenerates and the chorion laeve comes in contact with the deciduas of the opposite uterine
wall
• The rest of the villi form the chorion frondosum which develops into the definitive placenta
• Division and modification of the villous tree continues until term. First trimester villi are larger, have a complete layer
of cytotrophoblasts and have a loose mysenchymal core which is vascularised towards the end of the first trimester
• At term, the villi are smaller, cytotrophoblasts are few in number, the syncytiotrophoblast is irregularly thinned. Fetal
vessels are sinusoidal and occupy most of the villous core and lie close to the syncytiotrophoblast, forming
vasculusyncytial membranes which maximise materno-fetal transfer.
• Sometimes, the syncytiotrophoblast nuclei appear in clusters called syncytial knots - more common in placentas from
IUGR / pre-eclamptic pregnancies
• Maternal blood is separated from fetal blood by the syncytiotrophoblast and the fetal capillary endothelium

Question 2 The fetal component of the placental bed

Options for Questions 2-2

A Extra-villous trophoblasts B Villous and extra-villous trophoblasts


C Endovascular trophoblasts D Endovascular and extra-villous trophoblasts
E Villous and endovascular trophoblasts

A(Correct answ er: A)

Explanation
THE PLACENTAL BED
• Refers to the decidua and myometrium directly underlying the placenta
• The fetal component is made up of extra-villous trophoblast
• The maternal component is made up of decidualised endometrial stromal cells, macrophages and granular
lymphocytes. Residual endometrial glands are also present.

This Copy is for Dr. Mohamed ElHodiby


Question 3 The volume of amniotic fluid at term

Options for Questions 3-3

A 100-150 ml B 200-250 ml
C 300-400 ml D 800-1000 ml
E 2000-3000 ml

A(Correct answ er: D)

Explanation
AMNIOTIC FLUID
• Initially produced by primitive cells around the amniotic vesicle
• Later formed from transudate from fetal skin and umbilical cord and diffusion across the amniotic membrane
• Fetal skin becomes keratinised in the second trimester and amniotic fluid is mainly formed from fetal urine
and lung secretions. The term fetus passes 500-700ml urine per day
• Fetal swallowing is a major route of amniotic fluid re-circulation and begins at 12 weeks. At term. ~500ml
amniotic fluid is exchanged / 24h
• Amniotic fluid volume = 50ml at 12 weeks, 150ml at 16 weeks gestation and ~1000ml at term. Peak volume
is at 32-36 weeks
• Osmolarity: 275 mOsmol/l (lower than maternal or fetal), decreases as pregnancy progresses
• Cells: at term, contains fetal epithelial cells, amniocytes and dermal fibroblasts. Epithelial cells and
amniocytes grow poorly in culture. Glial cells present if neural tube defect
• Protein: concentration increases with gestation but plateaus after 30 weeks. Mainly albumin and globulins.
Also contains AFP (1/10TH concentration in fetal blood - rises until 12 weeks then declines). Virtually no
fibrinogen.
• Urea, creatinine and urate concentration increases with gestation
• Amino acids: concentration similar to that in maternal plasma
• Lipids: mainly free fatty acids. Also contains phospholipids, cholesterol and lecithin (secreted by lungs during
maturation)
• Carbohydrates: mainly glucose; concentration ~ half that of maternal serum
• PO2 = 2-15mmHg while PCO2 = 50-60mmHg
• pH = 7.0 (acidic relative to fetal blood)
• Bilirubin concentration falls in the third trimester (except in haemolytic disease)

Question 4 The mechanism of transfer of oxygen across the placenta

Options for Questions 4-4

A Simple diffusion B Facilitated diffusion


C Active transport D Secondary active transport
E Bulk transfer

A(Correct answ er: A)

Explanation

CARBOHYDRATES & AMINO ACIDS


• Glucose transport across the microvillous and basal plasma membranes is by facilitated diffusion.
• This is dependent on glucose concentration gradient and the activity of transport proteins within the placenta. In
theory, facilitated diffusion can be saturated.

This Copy is for Dr. Mohamed ElHodiby


• In practice, the glucose transfer capacity (especially of the microvillous plasma membrane) is so large that it would
not approach saturation under in-vivo conditions
• Fetal amino acid concentrations are generally higher than maternal levels
• Transfer of amino acids from the mother to the fetus is therefore against a concentration gradient and energy (ATP) is
required for this process
• This energy is provided by coupling the transfer of amino acids (up a concentration gradient) to the transfer of sodium
ions (down a concentration gradient) - secondary active transport. These are called Na+-dependent transporters. The
sodium concentration gradient is maintained by Na+K+ATPase.
• Other amino acid transport systems are, however, Na+-independent.

Question 5 Which one of the above is not readily transferred across the placenta?

Options for Questions 5-5

A Amino acids B Glucose


C Thyroxine D IgG
E Ca2+

A(Correct answ er: C)

Explanation
Molecule Placental transfer

Testosterone Minimal transfer - androgens aromatised by placenta. Very high maternal androgen
concentration may virilise female fetus

Ca2+, Mg2+ Active transfer against concentration gradient

PTH, Calcitonin Not transferred

Vitamin D Good transfer

IgA Minimal passive transfer

IgG Good active and active transfer from 7 weeks gestation

IgM No transfer

Glucose Facilitated diffusion - excellent transfer

Amino acids Active transport - excellent transfer

Free fatty acids Very limited transfer - essential fatty acids only

Ketone bodies Excellent transfer - diffusion

Insulin,
No transfer
glucagon
Thyroid
Poor transfer - diffusion
hormone

TRH Excellent transfer

This Copy is for Dr. Mohamed ElHodiby


Iodine and
Excellent transfer
thioamides
Cortisol &
Excellent transfer
aldosterone

ACTH No transfer

Question 6 Which one of the above is readily transferred across the placenta?
Options for Questions 6-6

A Testosterone B Parathyroid hormone


C Vitamin D D IgM
E Calcitonin

A(Correct answ er: C)

Explanation
Molecule Placental transfer

Testosterone Minimal transfer - androgens aromatised by placenta. Very high maternal androgen
concentration may virilise female fetus

Ca2+, Mg2+ Active transfer against concentration gradient

PTH, Calcitonin Not transferred

Vitamin D Good transfer

IgA Minimal passive transfer

IgG Good active and active transfer from 7 weeks gestation

IgM No transfer

Glucose Facilitated diffusion - excellent transfer

Amino acids Active transport - excellent transfer

Free fatty acids Very limited transfer - essential fatty acids only

Ketone bodies Excellent transfer - diffusion

Insulin,
No transfer
glucagon
Thyroid
Poor transfer - diffusion
hormone

TRH Excellent transfer

Iodine and
Excellent transfer
thioamides
Cortisol &
Excellent transfer
aldosterone

ACTH No transfer

This Copy is for Dr. Mohamed ElHodiby


Question 7 Dizygotic twins

Options for Questions 7-7

Have no more resemblance than siblings of


A Are less common than monozygotic twins B
different ages
C Can share the same placenta D Can share the same chorionic sac
E Can be monochorionic

A(Correct answ er: B)

Explanation
MULTIPLE PREGNANCY?Twins
UK data 2007: ~ 1 in 65 (1.5%) pregnancies were twins
Prevalence varies world-wide, being lowest in Japan and highest in Nigeria
Incidence of monozygotic twins relatively constant world-wide at ~ 3.5 per 1,000 births
Incidence of dizygotic twins varies widely
Dizygotic twins
• Fertilisation of two oocytes by different sperm
• Dizygotic twins have no more resemblance than brothers / sisters of different ages
• Both zygotes implant independently in the uterus and there are two separate placentas, amniotic and chorionic sacs
• The placentas and chorionic sacs may come into close approximation and fuse.
• Dizygotic twins cannot be monochorionic and cannot be identical. They are always dichorionic and diamniotic
Monozygotic twins
• A single ovum is fertilised and splits into two at different stages of development
• Earliest separation occurs at the 2 cell stage producing two zygotes which enter the uterus independently. Each
embryo would have its own placenta and chorionic sac, forming dichorionic diamniotic twins
• In the majority of cases, splitting occurs in the early blastocyst stage. The inner cell mass splits into two, producing
two embryos with a common placenta and a common chorionic cavity but separate amniotic cavities. These form
monochorionic diamniotic twins
• Rarely, splitting occurs at the stage of the bilaminar germ disc, resulting in two embryos with a common placenta,
common chorionic cavity and common amniotic cavity. These form monochorionic monoamniotic twins
• Splitting at later stages of development may result in incomplete splitting of the axial area of the germ disc, resulting
in conjoint twins

Question 8 With respect to the anatomy of the rectum

Options for Questions 8-8

Lymphatic drainage of the upper third is to the Lymphatic drainage of the middle third is to the
A B
superior mesenteric nodes superior mesenteric nodes
Lymphatic drainage of the lower third is to the Nerve supply is from the superior mesenteric
C D
inferior mesenteric nodes plexus
Venous drainage forms part of the portal-systemic
E
anastomoses

A(Correct answ er: E)

Explanation
THE RECTUM
• ~13cm long
• Upper third - covered by peritoneum on its anterior and lateral surfaces, middle third covered by peritoneum on the
anterior surface only and lower third devoid of peritoneum
• The three teniae coli of sigmoid colon come together to form a broad band of longitudinal smooth muscle on its
anterior and posterior surfaces

This Copy is for Dr. Mohamed ElHodiby


• Three transverse folds, two on the left and one on the right
• Anterior relations: sigmoid colon and small intestine within pouch of Douglas (upper third); Posterior surface of vagina
(lower third)
• Posterior relations: sacrum, coccyx, piriformis and coccygeus muscles, lavatory ani, sacral plexus and pelvic
sympathetic trunk
Blood supply
• Superior rectal artery - continuation of inferior mesenteric artery - main blood supply to mucosa
• Middle rectal artery - branch of internal iliac artery - muscular coat mainly
• Inferior rectal artery - branch of internal pudendal artery
• Veins correspond to arteries and form an important portal-systemic anastomosis
• Lymphatic drainage: Upper two thirds - inferior mesenteric nodes; lower third - internal iliac node
• Nerve supply - inferior hypogastric plexus

Question 9 The blood supply to the fallopian tube

Options for Questions 9-9

A Ovarian artery B Uterine artery


C Ovarian and uterine arteries D External iliac artery
E Internal iliac artery

A(Correct answ er: C)

Explanation

FALLOPIAN TUBES
• ~10cm long, 4 parts
• Infundibulum - funnel-shaped lateral end, projects beyond the broad ligament with fimbriae at its free end
• Ampulla - widest and longest part, site of fertilisation
• Isthmus - narrowest part, just lateral to the uterus
• Intra-mural part - pierces uterine wall
• Three coats: serous, muscular, and mucous.
• The external or serous coat is peritoneal. The middle or muscular coat consists of an external longitudinal and an
internal circular layer of smooth muscle fibers continuous with those of the uterus
• The mucosa is thrown into longitudinal folds, which in the ampulla are much more extensive than in the isthmus.
Lined by ciliated columnar epithelium
• Blood - ovarian and uterine arteries
• Lymphatics - aortic and internal iliac nodes (follow arteries)
• Nerves - inferior hypogastric plexus

Question 10 Which one of the above statements regarding the fallopian tube is true?

Options for Questions 10-10

The ampulla is the widest part and the site of


A The fallopian tube is 25cm long B
fertilization
C The intra-mural part is the narrowest segment D It does not have a muscular layer
E The fimbrial end is attached to the ovary

This Copy is for Dr. Mohamed ElHodiby


A(Correct answ er: B)

Explanation
FALLOPIAN TUBES
• ~10cm long, 4 parts
• Infundibulum - funnel-shaped lateral end, projects beyond the broad ligament with fimbriae at its free end
• Ampulla - widest and longest part, site of fertilisation
• Isthmus - narrowest part, just lateral to the uterus
• Intra-mural part - pierces uterine wall
• Three coats: serous, muscular, and mucous.
• The external or serous coat is peritoneal. The middle or muscular coat consists of an external longitudinal and an
internal circular layer of smooth muscle fibers continuous with those of the uterus
• The mucosa is thrown into longitudinal folds, which in the ampulla are much more extensive than in the isthmus.
Lined by ciliated columnar epithelium
• Blood - ovarian and uterine arteries
• Lymphatics - aortic and internal iliac nodes (follow arteries)
• Nerves - inferior hypogastric plexus

Question 11 Lies posterior to the body of the uterus

Options for Questions 11-11

A Urinary bladder B Ureters


C Sigmoid colon D Rectum
E Utero-vesical pouch

A(Correct answ er: C)

Explanation
THE UTERUS
• 8cm long x 5cm wide x 2.5cm thick
• Covered by peritoneum except anteriorly below the reflection of the utero-vesical fold of peritoneum and laterally
between the layers of the broad ligament?
• Fundus - that part of the uterus above the entrance of the uterine tubes?
• Cavity - triangular in coronal section, cleft in saggital section?
• Anteverted uterus - long axis of uterine body at 90degrees to long axis of vagina?
• Ante-flexed uterus - long axis of the body of the uterus bent forward at the level of the internal os?
• Retroverted uterus - body and fundus bent backwards on the vagina to lie within the pouch of Douglas?
• Anterior relations: utero-vesical pouch and superior surface of bladder?
• Posterior relations: Pouch of Douglas, sigmoid colon and coils of ileum?
• Lateral relations: uterine vessels, ureter?
• Nerve - inferior hypogastric plexuses (Parasympathetic via the pelvic splanchnic nerves, sympathetic via the lumbar
splanchnic nerves). Pain sensation is transmitted via the sympathetic nerves and the lumbar splanchnic nerves??
Lymph drainage:
• Fundus - accompany ovarian artery to para-aortic nodes at the level of L1
• Body and cervix - internal and external iliac nodes

Question 12 With respect to the innervation and support of the cervix

Options for Questions 12-12

This Copy is for Dr. Mohamed ElHodiby


The round ligament provides the most important The broad ligament provides the most important
A B
support to the cervix support to the cervix
The endocervix has autonomic but no sensory Sensory innervation of the cervix is via S2,3,4
C D
innervation spinal segments
The ectocervix has sensory but minimal
E
autonomic innervation

A(Correct answ er: D)

Explanation
?CERVIX
• Lower, narrow portion of the uterus, connected to the uterine fundus by the uterine isthmus - upper limit is the internal
os. Made up mainly of fibrous tissue with very little smooth muscle
• Protrudes through the upper anterior vaginal wall
• Approximately half its length is visible in the vagina (vaginal cervix), the rest being above the vagina (supra-vaginal
cervix)
• The vaginal cervix ~3 cm long and 2.5 cm wide. Size and shape varies widely with age, hormonal state, and parity -
bulkier and the external with a wider and more slit-like external os in multiparous women.
• Ectocervix - portion of the cervix beyond the external os - lined by stratified squamous non-keratinising epithelium.
• Endocervical canal - links external and internal os - lined by columnar epithelium
• The squamo-columnar junction - variable location - high up the endocervical canal before puberty and in the post-
menopausal women. Site of origin of squamous cell carcinoma of the cervix
• The external os is bounded by two lips, an anterior and a posterior, of which the anterior is the shorter and thicker,
although due to the slope of the cervix, it projects lower than the posterior. Both lips are in contact with the posterior
vaginal wall
• The supravaginal cervix is separated in front from the bladder by fibrous tissue (parametrium), which extends also on
to its sides and laterally between the layers of the broad ligaments. Not covered by peritoneum on the anterior aspect
• The uterine arteries reach the margins of the cervix within the parametrium
• The ureter runs downward and forward 2 cm lateral to the supravaginal cervix
• Posteriorly, the supravaginal cervix is covered by peritoneum, which extends on to the posterior vaginal wall, when it
is reflected on to the rectum, forming the Pouch of Douglas which may contain coils of small intestine.
• The vaginal cervix projects free into the anterior wall of the vagina between the anterior and posterior fornices.
?Blood supply
• Uterine artery, branch of internal iliac
• Cervical and vaginal branches supply the cervix and upper vagina.
• The cervical branches of the uterine arteries descend on the lateral aspects of the cervix at 3 and 9 o'clock. Venous
drainage parallels the arterial supply, eventually emptying into the hypogastric venous plexus.
?Lymphatics
• Regional lymph nodes for the cervix include: paracervical, parametrial, presacral, sacral, external iliac, common iliac,
hypogastric (obturator), internal iliac.

Question 13 Which one of the above statements regarding the cervix is correct?

Options for Questions 13-13

The endocervical canal is lined by cuboidal The ectocervix is lined by stratified squamous
A B
epithelium non-keratinising epithelium
The squamo-columnar junction is located within The endocervical canal links the internal os to the
C D
the ectocervix uterine cavity
Squamous cell carcinoma of the cervix typically
E
arises from the ectocervix

A(Correct answ er: B)

This Copy is for Dr. Mohamed ElHodiby


Explanation
?CERVIX
• Lower, narrow portion of the uterus, connected to the uterine fundus by the uterine isthmus - upper limit is the internal
os. Made up mainly of fibrous tissue with very little smooth muscle
• Protrudes through the upper anterior vaginal wall
• Approximately half its length is visible in the vagina (vaginal cervix), the rest being above the vagina (supra-vaginal
cervix)
• The vaginal cervix ~3 cm long and 2.5 cm wide. Size and shape varies widely with age, hormonal state, and parity -
bulkier and the external with a wider and more slit-like external os in multiparous women.
• Ectocervix - portion of the cervix beyond the external os - lined by stratified squamous non-keratinising epithelium.
• Endocervical canal - links external and internal os - lined by columnar epithelium
• The squamo-columnar junction - variable location - high up the endocervical canal before puberty and in the post-
menopausal women. Site of origin of squamous cell carcinoma of the cervix
• The external os is bounded by two lips, an anterior and a posterior, of which the anterior is the shorter and thicker,
although due to the slope of the cervix, it projects lower than the posterior. Both lips are in contact with the posterior
vaginal wall
• The supravaginal cervix is separated in front from the bladder by fibrous tissue (parametrium), which extends also on
to its sides and laterally between the layers of the broad ligaments. Not covered by peritoneum on the anterior aspect
• The uterine arteries reach the margins of the cervix within the parametrium
• The ureter runs downward and forward 2 cm lateral to the supravaginal cervix
• Posteriorly, the supravaginal cervix is covered by peritoneum, which extends on to the posterior vaginal wall, when it
is reflected on to the rectum, forming the Pouch of Douglas which may contain coils of small intestine.
• The vaginal cervix projects free into the anterior wall of the vagina between the anterior and posterior fornices.
?Blood supply
• Uterine artery, branch of internal iliac
• Cervical and vaginal branches supply the cervix and upper vagina.
• The cervical branches of the uterine arteries descend on the lateral aspects of the cervix at 3 and 9 o'clock. Venous
drainage parallels the arterial supply, eventually emptying into the hypogastric venous plexus.
?Lymphatics
• Regional lymph nodes for the cervix include: paracervical, parametrial, presacral, sacral, external iliac, common iliac,
hypogastric (obturator), internal iliac.

Question 14 Somatic sensory innervation of the lower third of the vagina

Options for Questions 14-14

A Pelvic splanchnic nerves B Lumbar splanchnic nerves


C Pudendal nerve D Superficial femoral nerve
E Obturator nerve

A(Correct answ er: C)

Explanation
THE VAGINA
• ~8cm long, axis directed upwards and backwards from the vulva. Posterior wall longer than anterior wall
• Lined by stratified squamous epithelium which undergoes changes during the menstrual cycle. Does not secrete
mucus
• Has anterior and posterior walls which are normally in apposition, and four fornices (anterior, posterior, left and right
lateral)
• Upper half lies above the level of the pelvic floor
?Relations
• Anterior: bladder, urethra
• Posterior: upper third - pouch of Douglas; middle third - ampulla of the rectum; lower third - perineal body

This Copy is for Dr. Mohamed ElHodiby


• Lateral: upper part - ureter, middle part - anterior fibres of levator ani; lower part - uro-genital diaphragm and the bulb
of the vestibule
• Blood: vaginal artery, branch of internal iliac artery
• Lymphatics: upper third - internal and external iliac nodes; middle third - internal iliac nodes; lower third - superficial
inguinal nodes
• Nerve:Upper two thirds - inferior hypogastric plexuses. Parasympathetic from pelvic splanchnic nerves, sympathetic
from lumbar splanchnic nerves (L1 & 2)
• Autonomic innervation to the lower third - pudendal nerve
• Somatic sensation is present mainly in the lower third and is carried by the pudendal nerve

Question 15 The embryological origin of the trigone of the bladder

Options for Questions 15-15

A Urachus B Yolk sac


C Mesonephric duct D Mesonephric mesoderm
E Cloaca

A(Correct answ er: C)

Explanation
THE URINARY BLADDER
• Lined by transitional epithelium - the upper part is derived from the yolk sac and is continuous with the
urachus
• The trogone is derived from the mesonephric duct and is lined by cells of mesodermal origin. It is believed
that these are later replaced by cells of endodermal origin
• The bladder wall is made up of a syncytium of smooth muscle fibres called the detrusor - contraction results
in simultaneous reduction in all dimensions of the bladder
• The female urethral sphincter has an intrinsic and an extrinsic component: the intrinsic component is made
up of epithelial, vascular and connective tissue and the rhabdosphincter which is a circular ring of striated
muscle with slow twitch fibres. The ring is well developed anteriorly, thins laterally and is virtually absent
posteriorly
• The extrinsic sphincter mechanism is made up of the striated muscles of the levator ani - mainly fast twitch
fibres

Question 16 The posterior border of the pelvic outlet

Options for Questions 16-16

A Ischial tuberosities B Sacro-spinous & sacro-tuberous ligaments


C Ilio-inguinal ligaments D Coccyx
E Sacral promontory

A(Correct answ er: D)

Explanation
TRUE PELVIS
That part of the pelvis between the pelvic inlet and the pelvic outlet
The pelvic inlet is oval in shape in the Gynaecoid pelvis and has the following borders:
1) Posterior: Sacral promontory?2) Lateral: Iliopectineal line?3)Anterior: Symphysis pubis
The pelvic outlet is diamond shaped with the following borders:
• Posterior: Coccyx
• Lateral: Ischial tuberosities

This Copy is for Dr. Mohamed ElHodiby


• Anterior: Pubic arch formed by the simphysis pubis and the ischio-pubic rami
• Part of the lateral border is formed by the sacro-spinous and sacro-tuberous ligaments which convert the greater and
lesser sciatic notches into the greater and lesser sciatic foramina
• The pelvic cavity has a shallow anterior wall and a deeper posterior wall. The ischial spines are at the level of the
mid-cavity
• The acetabulum is formed from the ilium, ischium and pubic bones

Question 17 Insertion of the piriformis muscle

Options for Questions 17-17

A Greater trochanter B Lesser trochanter


C Body of sacral vertebrae D Lateral mass of sacral vertebrae
E Simplysis pubis

A(Correct answ er: A)

Explanation

PYRIFORMIS
• Origin: Lateral mass of sacrum
• Leaves pelvis through greater sciatic foramen
• Insertion: Upper border of greater trochanter
• Action: Laterally rotates femur at hip joint
• Nerve: Sacral plexus

Question 18 Which one of the above statements about the female pelvis is correct?

Options for Questions 18-18

The cavity of the female pelvis is deeper than that The sacrum is longer in the female pelvis than in
A B
of the male pelvis the male pelvis
The sacrum is wider in the female pelvis than in The ischial spines are more prominent in the
C D
the male pelvis female pelvis than in the male pelvis
The sciatic notches are narrower in the female
E
pelvis than in the male pelvis

A(Correct answ er: C)

Explanation
?Differences between the Male and Female Pelvis?The female pelvis:
• Less massive
• The anterior iliac spines more widely separated - greater lateral prominence of the hips.
• Wider pelvic inlet - both antero-posterior and transverse diameters
• Pelvic inlet more circular
• More shallow
• The sacrum is shorter wider, and its upper part is less curved
• The obturator foramina are triangular in shape and smaller in size
• The outlet is larger and the coccyx more movable.
• The sciatic notches are wider and shallower

This Copy is for Dr. Mohamed ElHodiby


• The ischial spines are less prominent.
• The pubic symphysis is less deep, and the pubic arch is wider and more rounded than in the male?

Question 19 Which one of the above does not contribute to the sacral plexus?

Options for Questions 19-19

A Anterior ramus of L4 B Anterior ramus of L5


C Anterior ramus of S2 D Anterior ramus of S4
E Anterior ramus of S5

A(Correct answ er: E)

Explanation
SACRAL PLEXUS
Formed from anterior rami of L4&5 (lumbosacral trunk) and anterior rami of S1,2,3&4
Related anteriorly to internal iliac vessels + branches and the rectum
Related posteriorly to the piriformis muscle

Branches?To lower limb - leave the pelvis through GREATER sciatic foramen
• Sciatic nerve - L4,5, S1,2&3; largest nerve in the body
• Superior gluteal nerve - gluteus medius, minimus and tensor fascia lata
• Inferior gluteal nerve - gluteus maximus
• Nerve to obturator internus - also supplies superior gamellus muscle
• Nerve to quadratus femoris - also supplies inferior gamellus muscle
• Posterior cutaneous nerve of the thigh

Question 20 Which one of the above organs is correctly paired with its parasympathetic nerve supply?

Options for Questions 20-20

A Urinary bladder – lumbar splanchnic nerves B Small intestine – pelvic splanchnic nerves
C Descending colon – vagus nerve D Vagina – lumbar splanchnic nerves
E Ovary – vagus nerve

A(Correct answ er: E)

Explanation
PELVIC SPLANCHNIC NERVES
• Formed from the ventral (anterior) primary rami of S2 through S4.
• These are the ways in which parasympathetic neurons reach the hypogastric plexus, and therefore the pelvic viscera
and distal colon.
• The parasympathetic part of the autonomic nervous system is the "craniosacral" part. Parasympathetic innervation to
most of the gut comes from the "cranio-" half of that, i.e., the vagus nerve. The rest, to colon distal to the splenic
flexure and to pelvic viscera, is from the "-sacral" half, via the pelvic splanchnic nerves
• There are thoracic, lumbar, sacral, and pelvic splanchnic nerves.
• "Splanchnic" refers to nerves that supply viscera.
• Thoracic, lumbar and sacral splanchnic nerves emerge from sympathetic ganglia and carry sympathetic fibers
• Pelvic splanchnic nerves are parasympathetic
• Contain pre-ganglionic fibres
• Join inferior hypogastric plexus
• Some fibres ascend to the superior hypogastric and eventually inferior mesenteric plexus and supply the hind-gut
• Provide parasympathetic supply to the pelvic viscera

This Copy is for Dr. Mohamed ElHodiby


• Afferent impulses from the pelvic viscera are transmitted mainly by the sympathetic pathway
• Parasympathetic nerves innervate detrusor and internal sphincter via the pelvic splanchnic nerves (S2,3,4) and also
innervate the external sphincter via the pudendal nerve - initiate micturiction by inhibiting internal sphincter activity
and stimulating detrusor contraction
• Parasympathetic supply to the ovary is from the vagus nerve. Sympathetic innervation is from the L1/2 segment
• Pain from uterine contractions goes back to T10-L1. This means that uterine contraction pain is sympathetic. But for
the lower portion of the uterus and upper vagina (the cervix) - it is parasympathetic back to the pelvic splanchnic
nerves

Question 21 The upper half of the anal canal is sensitive to these stimuli

Options for Questions 21-21

A Stretch B Stretch and touch


C Stretch, touch and temperature D Stretch, touch, temperature and pain
E Stretch and pain
A(Correct answ er: A)

Explanation
THE ANAL CANAL
• ~4cm long, extends downwards and backwards from the rectal ampulla to the anus
• LATERAL walls kept in apposition by the levator ani and anal sphincter except during defecation
• Anterior relations: perineal body, urogenital diaphragm and perineal body
• Posterior relations: anococcygeal body and the coccyx
?UPPER HALF
• Derived from hind-gut endoderm
• Lined by columnar epithelium
• Thrown into vertical folds called anal columns
• Autonomic supply from the inferior hypogastric plexus, sensitive to stretch only
• Blood - superior rectal branch of inferior mesenteric artery
• Lymphatics - inferior mesenteric nodes
• Dentate line - boundary between endodermal and ectodermal origin of anal canal
?LOWER HALF
• Derived from the ectoderm of the proctodeum
• Lined by stratified squamous epithelium
• No anal columns
• Blood - inferior rectal artery, branch of internal pudendal artery
• Lymphatics - medial group of superficial inguinal nodes
• Nerve - inferior rectal nerve - sensitive to temperature, pain, touch and pressure

Question 22 Which one of the above statements about the anal sphincter is true?

Options for Questions 22-22

The internal sphincter is composed of skeletal The internal sphincter lies deep to the outer layer
A B
muscle of longitudinal smooth muscle
The external sphincter is composed of smooth
C D The external sphincter is inserted onto the coccyx
muscle
The internal sphincter is inserted onto the perineal
E
body anteriorly

This Copy is for Dr. Mohamed ElHodiby


A(Correct answ er: B)

Explanation
ANAL SPHINCTER??
Internal
• Thickening of the inner circular layer of smooth muscle in the upper half of the anal canal
• Lies deep to the outer layer of longitudinal smooth muscle
?External
• Skeletal muscle
• Sub-cutaneous part - lower half of anal canal, no bony attachments
• Superficial part - attached to the perineal body anteriorly and the anococcygeal body posteriorly
• Deep part - no bony attachments, blends with puborectalis

Question 23 The bulbospongiosus muscle

Options for Questions 23-23

A Originates anterior to the vaginal orifice B Is inserted onto the body of the pubic bone
Is inserted onto the corpus spongiosus of the
C D Compresses the artery supplying the clitoris
clitoris
E Originates from the ischial tuberosities

A(Correct answ er: C)

Explanation
BULBOSPONGIOSUS MUSCLE
• Surrounds vaginal orifice and covers the bulb of the vestibule
• Origin - perineal body
• Inserted onto the corpus spongiosus of the clitoris
• Compresses dorsal vein of the clitoris and assists in erection

Question 24 Main venous drainage of the skin of the abdominal wall above the level of the umbilicus

Options for Questions 24-24

A Internal thoracic vein B Lateral thoracic vein


C Azygous vein D Great saphenous vein
E Portal vein

A(Correct answ er: B)

Explanation
Venous drainage: abdominal wall
• Above umbilicus - axillary vein via lateral thoracic vein
• Below umbilicus - femoral vein via superficial epigastric and great saphenous vein
• Few para-umbilical veins drain into the portal vein via the ligamentun teres, forming a portal-systemic anastomosis.
Distended veins may indicate portal hypertension
• Deep veins follow the arteries with the posterior intercostals veins draining into the azygous vein and the lumbar
veins into the inferior vena cava
?Lymphatics

This Copy is for Dr. Mohamed ElHodiby


• Above umbilicus - anterior axillary nodes
• Below umbilicus - superficial inguinal nodes
• Deep lymphatics follow the arteries into the internal thoracic, external iliac, posterior mediastinal and para-aortic
nodes

Question 25 The sac of this hernia lies below and lateral to the pubic tubercle

Options for Questions 25-25

A Direct inguinal hernia B Indirect inguinal hernia


C Femoral hernia D Richcter’s hernia
E Epigastric hernia

A(Correct answ er: C)

Explanation
HERNIAS
INGUINAL
• Indirect commoner than direct
• Indirect more common in males and on the right side (right testis descends later than the left)
• Direct hernias commoner in (old) males
• Indirect: Enters inguinal canal through deep inguinal ring, lateral to inferior epigastric vessels. Can descent into the
scrotum or labia majora. Sac is formed by the remains of the processus vaginalis
• The sac of all inguinal hernias lies above and medial to the pubic tubercle
FEMORAL
• Commoner in females protrudes through the femoral canal medial to the femoral vein, below and lateral to the pubic
tubercle.
UMBILICAL
• Congenital - exomphalos
• Acquired infantile - weakness in scar of umbilicus
• Acquired adult - para-umbilical, weakness in linea alba above or below umbilicus, commoner in females
EPIGASTRIC
• Weakness in linea alba above umbilicus
RICHCTER’S
• A knuckle of the side-wall of the bowel is incarcerated in the sac but the continuity of the bowel is maintained and
there is no obstruction.

Question 26 This hernia may enter the scrotum or labia majora

Options for Questions 26-26

A Direct inguinal hernia B Indirect inguinal hernia


C Femoral hernia D Richcter’s hernia
E Epigastric hernia

A(Correct answ er: B)

Explanation
HERNIAS
INGUINAL
• Indirect commoner than direct

This Copy is for Dr. Mohamed ElHodiby


• Indirect more common in males and on the right side (right testis descends later than the left)
• Direct hernias commoner in (old) males
• Indirect: Enters inguinal canal through deep inguinal ring, lateral to inferior epigastric vessels. Can descent into the
scrotum or labia majora. Sac is formed by the remains of the processus vaginalis
• The sac of all inguinal hernias lies above and medial to the pubic tubercle
FEMORAL
• Commoner in females protrudes through the femoral canal medial to the femoral vein, below and lateral to the pubic
tubercle.
UMBILICAL
• Congenital - exomphalos
• Acquired infantile - weakness in scar of umbilicus
• Acquired adult - para-umbilical, weakness in linea alba above or below umbilicus, commoner in females
EPIGASTRIC
• Weakness in linea alba above umbilicus
RICHCTER’S
• A knuckle of the side-wall of the bowel is incarcerated in the sac but the continuity of the bowel is maintained and
there is no obstruction.

Question 27 Nerve supply to the internal oblique muscle


Options for Questions 27-27

A Upper 6 thoracic nerves B Lower 6 thoracic nerves


C Upper 3 lumbar nerves D Lower 3 lumbar nerves
E 12th thoracic nerve

A(Correct answ er: B)

Explanation
NERVES
• External oblique, internal oblique and transversus: Lower 6 thoracic nerves and L1 (ileoinguinal and ileohypogastric
nerves)
• Rectus abdominis: Lower 6 thoracic nerves
• Pyramidalis: T12
FUNCTION
• External / internal oblique / transversus - laterally flex and rotate the trunk, relax during inspiration to accommodate
abdominal viscera, contract during micturiction, defecation and vomiting.
• Rectus abdominis - flexes the trunk and stabilises the pelvis

Question 28 Which one of the above structures is not cut or split when making a Pfannestiel incision?

Options for Questions 28-28

A Scarpa’s fascia B Rectus abdominis muscle


C Linea alba D Transversalis fascia
E Parietal peritoneum

A(Correct answ er: B)

Explanation
The Pfannestiel incision cuts through
• Skin and superficial fascia including Scarpa’s fascia
• The rectus sheath which is formed at this level by the aponeuroses of the external, internal oblique and transversus
abdominis

This Copy is for Dr. Mohamed ElHodiby


• The linea alba is split to separate the recti abdominis which are NOT cut
• The transversalis fascia
• The parietal peritoneum
Incisions should be made along Langer’s lines which run horizontally in the lower abdomen?

Question 29 Which one of the above is correct?

Options for Questions 29-29

Superficial inguinal ring – gives rise to internal


A Superficial inguinal ring – oval in shape B
spermatic fascia
Superficial inguinal ring – defect in aponeurosis of Deep inguinal ring – gives rise to external
C D
external oblique spermatic fascia
E Deep inguinal ring – 1.3cm above pubic tubercle

A(Correct answ er: C)

Explanation
INGUINAL CANAL
• 4cm long in adults
• Deep ring: - oval shaped hole in transversalis fascia, 1.3cm above inguinal ligament, mid-way between anterior
superior iliac spine and symphysis pubis. Medial relation - inferior epigastric vessels. Gives rise to the internal
spermatic fascia or the internal covering of the round ligament of the uterus
• Superficial ring:- triangular in shape, defect in external oblique aponeurosis, base formed by pubic crest, gives rise to
external spermatic fascia
• The following pass through the superficial ring: round ligament, ilioinguinal nerve, genital branch of the genitofemoral
nerve, lymphatics and sympathetic plexus
• Anterior wall - aponeurosis of external oblique, reinforced laterally by origin of internal oblique
• Posterior wall - transversalis fascia, reinforced medially by conjoint tendon - common insertion of internal oblique and
transversus to the pubic crest and pectineal line
• Floor - Inguinal ligament
• Roof - internal oblique and transversus abdominis
• The femoral neurovascular bundle and lateral cutaneous nerve of the thigh pass beneath the inguinal ligament.
• The femoral branch of the genitofemoral nerve enters the thigh behind the middle of the inguinal ligament.
• The ilioinguinal nerve enters the thigh through the superficial inguinal ring.
• The superficial epigastric vessels cross the inguinal ligament

Question 30 The cremaster muscle

Options for Questions 30-30

A Is derived from the external oblique muscle B Is derived from the internal oblique muscle
C Is derived from the transversus abdominis muscle D Is supplied by the femoral nerve
E Is supplied by branches of the femoral artery

A(Correct answ er: B)

Explanation
Cremaster muscle / cremasteric reflex
• Derived from internal oblique
• Supplied by cremasteric artery - branch of inferior epigastric
• Supplied by genital branch of genitor-femoral nerve

This Copy is for Dr. Mohamed ElHodiby


• Cremasteric reflex - cremaster muscle contracts when skin on medial aspect of thigh is stroked. Afferent - femoral
branch; Efferent - genital branch of genitor-femoral nerve

This Copy is for Dr. Mohamed ElHodiby


Question 1 The posterior border of the pelvic inlet

Options for Questions 1-1

A Sacral promontory B 4th sacral vertebra


C T12 vertebral body D The ala of the sacrum
E Piriformis muscle

A(Correct answ er: A)

Explanation
TRUE PELVIS
That part of the pelvis between the pelvic inlet and the pelvic outlet
The pelvic inlet is oval in shape in the Gynaecoid pelvis and has the following borders:
1) Posterior: Sacral promontory?2) Lateral: Iliopectineal line?3)Anterior: Symphysis pubis
The pelvic outlet is diamond shaped with the following borders:
• Posterior: Coccyx
• Lateral: Ischial tuberosities
• Anterior: Pubic arch formed by the simphysis pubis and the ischio-pubic rami
• Part of the lateral border is formed by the sacro-spinous and sacro-tuberous ligaments which convert the greater and
lesser sciatic notches into the greater and lesser sciatic foramina
• The pelvic cavity has a shallow anterior wall and a deeper posterior wall. The ischial spines are at the level of the
mid-cavity
• The acetabulum is formed from the ilium, ischium and pubic bones

Question 2 The shape of the pelvic outlet in females

Options for Questions 2-2

A Oval shaped B Circular shaped


C Diamond shaped D Heart shaped
E Quadrilateral shaped

A(Correct answ er: A)

Explanation
TRUE PELVIS
That part of the pelvis between the pelvic inlet and the pelvic outlet
The pelvic inlet is oval in shape in the Gynaecoid pelvis and has the following borders:
1) Posterior: Sacral promontory?2) Lateral: Iliopectineal line?3)Anterior: Symphysis pubis
The pelvic outlet is diamond shaped with the following borders:
• Posterior: Coccyx
• Lateral: Ischial tuberosities
• Anterior: Pubic arch formed by the simphysis pubis and the ischio-pubic rami
• Part of the lateral border is formed by the sacro-spinous and sacro-tuberous ligaments which convert the greater and
lesser sciatic notches into the greater and lesser sciatic foramina
• The pelvic cavity has a shallow anterior wall and a deeper posterior wall. The ischial spines are at the level of the
mid-cavity
• The acetabulum is formed from the ilium, ischium and pubic bones

The shape of the pelvic outlet in females

This Copy is for Dr. Mohamed ElHodiby


Diam

Coccyx
Ischial tuberosities
Sacro-spinous & sacro-tuberous ligaments
Ilio-inguinal ligaments
Sacral promontory

The posterior border of the pelvic outlet


Coccyx

Coccyx
Ischial spines
Sacro-spinous & sacro-tuberous ligaments
Ilio-inguinal ligaments
Sacral promontory

The lateral border of the pelvic outlet


Sacro-spinous & sacro-tuberous ligaments

With respect to the anatomy of the pelvis


The cavity of the pelvis has a shallow posterior wall and a deeper anterior wall
The ischial spines are at the level of the mid-cavity T
The pubic bone does not form part of the acetabulum
The ischial tuberosities form the medial border of the pelvic outlet
The true pelvis is that part of the pelvis between the pelvic inlet and the ischial spines

Question 3 The posterior border of the pelvic outlet

Options for Questions 3-3

A Ischial tuberosities B Sacro-spinous & sacro-tuberous ligaments


C Ilio-inguinal ligaments D Coccyx
E Sacral promontory

A(Correct answ er: D)

Explanation
TRUE PELVIS
That part of the pelvis between the pelvic inlet and the pelvic outlet
The pelvic inlet is oval in shape in the Gynaecoid pelvis and has the following borders:
1) Posterior: Sacral promontory?2) Lateral: Iliopectineal line?3)Anterior: Symphysis pubis
The pelvic outlet is diamond shaped with the following borders:
• Posterior: Coccyx
• Lateral: Ischial tuberosities
• Anterior: Pubic arch formed by the simphysis pubis and the ischio-pubic rami
• Part of the lateral border is formed by the sacro-spinous and sacro-tuberous ligaments which convert the greater and
lesser sciatic notches into the greater and lesser sciatic foramina
• The pelvic cavity has a shallow anterior wall and a deeper posterior wall. The ischial spines are at the level of the
mid-cavity
• The acetabulum is formed from the ilium, ischium and pubic bones

Question 4 The lateral border of the pelvic outlet

Options for Questions 4-4

This Copy is for Dr. Mohamed ElHodiby


A Coccyx B Ischial spines
C Sacro-spinous & sacro-tuberous ligaments D Ilio-inguinal ligaments
E Sacral promontory

A(Correct answ er: C)

Explanation
TRUE PELVIS
That part of the pelvis between the pelvic inlet and the pelvic outlet
The pelvic inlet is oval in shape in the Gynaecoid pelvis and has the following borders:
1) Posterior: Sacral promontory?2) Lateral: Iliopectineal line?3)Anterior: Symphysis pubis
The pelvic outlet is diamond shaped with the following borders:
• Posterior: Coccyx
• Lateral: Ischial tuberosities
• Anterior: Pubic arch formed by the simphysis pubis and the ischio-pubic rami
• Part of the lateral border is formed by the sacro-spinous and sacro-tuberous ligaments which convert the greater and
lesser sciatic notches into the greater and lesser sciatic foramina
• The pelvic cavity has a shallow anterior wall and a deeper posterior wall. The ischial spines are at the level of the
mid-cavity
• The acetabulum is formed from the ilium, ischium and pubic bones

Question 5 With respect to the anatomy of the sacrum, which of the above structures are correctly paired?

Options for Questions 5-5

A Vertebral foramina – cauda equina B Vertebral foramina – sacral hiatus


C S3 and S4 – lumbo-sacral angle D Laminae of S1 & S2 – Sacral hiatus
E Sacral promontory – origin of piriformis muscle
A(Correct answ er: A)

Explanation
?THE SACRUM
• 5 rudimentary vertebrae
• The anterior and upper margin of the first sacral vertebra bulges forward as the posterior margin of the pelvic inlet
and forms the sacral promontory
• Articulates with innominate bone at the sacro-iliac joint, with the 5th lumbar vertebra superiorly (where it is tilted
forwards at the lumbo-sacral angle) and with the coccyx inferiorly
• The laminae of S5 and occasionally S4 fail to fuse in the mid-line, forming the sacral hiatus
• The vertebral foramina form the sacral canal which contains the cauda equine, filum terminale and meninges as far
as the lower border of S2 while the rest of the canal contains the filum terminale and lower sacral and coccygeal
nerves
• The anterior / posterior surfaces contain 4 foramina for the anterior and posterior rami of the upper 4 sacral nerves
respectively
• The piriformis muscle arises from the front of the lateral mass of the sacrum and leaves the pelvis through the greater
sciatic foramen to insert onto the greater trochanter

Question 6 The main action of the obturator internus muscle

Options for Questions 6-6

This Copy is for Dr. Mohamed ElHodiby


A Flexes the hip joint B Extends the hip joint
C Laterally rotates the hip joint D Medially rotates the hip joint
E Adducts the hip joint

A(Correct answ er: C)

Explanation
OBTURATOR INTERNUS
• Origin: Pelvic surface of obturator membrane and adjoining part of the hip bone
• Leaves pelvis through Lesser sciatic foramen
• Covered by the obturator fascia which gives rise to the origin of the levator ani muscle
• Forms the lateral wall of the roof of the ischio-rectal fossa (medial wall of the roof formed by the levator ani)
• Insertion: Greater trochanter of femur
• Action: Laterally rotates femur at hip joint
• Nerve: nerve to obturator internus from sacral plexus

Question 7 The nerve supply to the levator ani muscle

Options for Questions 7-7

A Pudendal nerve B Perineal branch of the pudendal nerve


C Superficial femoral nerve D Pelvic splanchnic nerves
E Obturator nerve

A(Correct answ er: B)

Explanation
LEVATOR ANI MUSCLE
• Origin: Back of body of pubis, obturator fascia and ischial spine
• Insertion: Anterior fibres (sphincter vaginae) - perineal body
• Intermediate fibres (puborectalis) - median raphe and anococcygeal body
• Posterior fibres (iliococcygeus) - anococcygeal body and coccyx
• Action: Support pelvic viscera and resist rise in intra-pelvic pressure during straining; sphincter action at anorectal
junction and vagina
• Nerve: Perineal branch of S4 and perineal branch of the pudendal nerve

COCCYGEUS
• Origin: ischial spine
• Insertion: lower end of sacrum and upper part of coccyx
• Action: supports pelvic viscera, flexes coccyx
• Nerve: branch of S4 and S5

Question 8 The levator ani muscle is not attached to which one of the above structures?

Options for Questions 8-8

A Body of the pubic bone B Obturator fascia


C Ischial spines D Perineal body
E The ala of the sacrum

This Copy is for Dr. Mohamed ElHodiby


A(Correct answ er: E)

Explanation
LEVATOR ANI MUSCLE
• Origin: Back of body of pubis, obturator fascia and ischial spine
• Insertion: Anterior fibres (sphincter vaginae) - perineal body
• Intermediate fibres (puborectalis) - median raphe and anococcygeal body
• Posterior fibres (iliococcygeus) - anococcygeal body and coccyx
• Action: Support pelvic viscera and resist rise in intra-pelvic pressure during straining; sphincter action at anorectal
junction and vagina
• Nerve: Perineal branch of S4 and perineal branch of the pudendal nerve

COCCYGEUS
• Origin: ischial spine
• Insertion: lower end of sacrum and upper part of coccyx
• Action: supports pelvic viscera, flexes coccyx
• Nerve: branch of S4 and S5

Question 9 Which one of the above does not contribute to the sacral plexus?

Options for Questions 9-9

A Anterior ramus of L4 B Anterior ramus of L5


C Anterior ramus of S2 D Anterior ramus of S4
E Anterior ramus of S5

A(Correct answ er: E)

Explanation
SACRAL PLEXUS
Formed from anterior rami of L4&5 (lumbosacral trunk) and anterior rami of S1,2,3&4
Related anteriorly to internal iliac vessels + branches and the rectum
Related posteriorly to the piriformis muscle

Branches?To lower limb - leave the pelvis through GREATER sciatic foramen
• Sciatic nerve - L4,5, S1,2&3; largest nerve in the body
• Superior gluteal nerve - gluteus medius, minimus and tensor fascia lata
• Inferior gluteal nerve - gluteus maximus
• Nerve to obturator internus - also supplies superior gamellus muscle
• Nerve to quadratus femoris - also supplies inferior gamellus muscle
• Posterior cutaneous nerve of the thigh

Which one of the above statements about the anatomy of the autonomic nervous system is
Question 10
true?
Options for Questions 10-10

Thoracic splanchnic nerves contain Lumbar splanchnic nerves do not contain


A B
parasympathetic fibres sympathetic fibres
Pelvic splanchnic nerves provide parasympathetic Pelvic splanchnic nerves provide sympathetic
C D
supply to the bladder and hind-gut supply to the bladder and hind-gut
Pelvic splanchnic nerves provide parasympathetic
E
supply to the ovary

This Copy is for Dr. Mohamed ElHodiby


A(Correct answ er: C)

Explanation
PELVIC SPLANCHNIC NERVES
• Formed from the ventral (anterior) primary rami of S2 through S4.
• These are the ways in which parasympathetic neurons reach the hypogastric plexus, and therefore the pelvic viscera
and distal colon.
• The parasympathetic part of the autonomic nervous system is the "craniosacral" part. Parasympathetic innervation to
most of the gut comes from the "cranio-" half of that, i.e., the vagus nerve. The rest, to colon distal to the splenic
flexure and to pelvic viscera, is from the "-sacral" half, via the pelvic splanchnic nerves
• There are thoracic, lumbar, sacral, and pelvic splanchnic nerves.
• "Splanchnic" refers to nerves that supply viscera.
• Thoracic, lumbar and sacral splanchnic nerves emerge from sympathetic ganglia and carry sympathetic fibers
• Pelvic splanchnic nerves are parasympathetic
• Contain pre-ganglionic fibres
• Join inferior hypogastric plexus
• Some fibres ascend to the superior hypogastric and eventually inferior mesenteric plexus and supply the hind-gut
• Provide parasympathetic supply to the pelvic viscera
• Afferent impulses from the pelvic viscera are transmitted mainly by the sympathetic pathway
• Parasympathetic nerves innervate detrusor and internal sphincter via the pelvic splanchnic nerves (S2,3,4) and also
innervate the external sphincter via the pudendal nerve - initiate micturiction by inhibiting internal sphincter activity
and stimulating detrusor contraction
• Parasympathetic supply to the ovary is from the vagus nerve. Sympathetic innervation is from the L1/2 segment
• Pain from uterine contractions goes back to T10-L1. This means that uterine contraction pain is sympathetic. But for
the lower portion of the uterus and upper vagina (the cervix) - it is parasympathetic back to the pelvic splanchnic
nerves

This Copy is for Dr. Mohamed ElHodiby


Question 1 The main oestrogen produced by the placenta

Options for Questions 1-1

A Oestrone B Oestradiol
C Oestriol D Ethinyl-oestradiol
E Oestradiol glucuronate

A(Correct answ er: C)

Explanation
PLACENTAL OESTROGENS
Mainly oestriol, but also oestradiol and oestrone in smaller amounts. Oestriol is produces from DHES-sulphate from
fetal zone of the fetal adrenal gland and also from the maternal adrenals. Fetal DHEA-S is initially hydroxylated by the
fetal liver

Question 2 Which one of the above is not readily transferred across the placenta?

Options for Questions 2-2

A Amino acids B Glucose


C Thyroxine D IgG
E Ca2+

A(Correct answ er: C)

Explanation
Molecule Placental transfer

Testosterone Minimal transfer - androgens aromatised by placenta. Very high maternal androgen
concentration may virilise female fetus

Ca2+, Mg2+ Active transfer against concentration gradient

PTH, Calcitonin Not transferred

Vitamin D Good transfer

IgA Minimal passive transfer

IgG Good active and active transfer from 7 weeks gestation

IgM No transfer

Glucose Facilitated diffusion - excellent transfer

Amino acids Active transport - excellent transfer

Free fatty acids Very limited transfer - essential fatty acids only

This Copy is for Dr. Mohamed ElHodiby


Ketone bodies Excellent transfer - diffusion

Insulin,
No transfer
glucagon
Thyroid
Poor transfer - diffusion
hormone

TRH Excellent transfer

Iodine and
Excellent transfer
thioamides
Cortisol &
Excellent transfer
aldosterone

ACTH No transfer

Question 3 Which one of the above is readily transferred across the placenta?

Options for Questions 3-3

A Testosterone B Parathyroid hormone


C Vitamin D D IgM
E Calcitonin

A(Correct answ er: C)

Explanation
Molecule Placental transfer

Testosterone Minimal transfer - androgens aromatised by placenta. Very high maternal androgen
concentration may virilise female fetus

Ca2+, Mg2+ Active transfer against concentration gradient

PTH, Calcitonin Not transferred

Vitamin D Good transfer

IgA Minimal passive transfer

IgG Good active and active transfer from 7 weeks gestation

IgM No transfer

Glucose Facilitated diffusion - excellent transfer

Amino acids Active transport - excellent transfer

Free fatty acids Very limited transfer - essential fatty acids only

Ketone bodies Excellent transfer - diffusion

Insulin,
No transfer
glucagon
Thyroid
Poor transfer - diffusion
hormone

This Copy is for Dr. Mohamed ElHodiby


TRH Excellent transfer

Iodine and
Excellent transfer
thioamides
Cortisol &
Excellent transfer
aldosterone

ACTH No transfer

Question 4 Monozygotic twins cannot have this type of placentation

Options for Questions 4-4

A Monochorionic monoamniotic B Dichorionic diamniotic


C Dichorionic monoamniotic D Monochorionic diamniotic
E None of the above

A(Correct answ er: E)

Explanation
MULTIPLE PREGNANCY?Twins
UK data 2007: ~ 1 in 65 (1.5%) pregnancies were twins
Prevalence varies world-wide, being lowest in Japan and highest in Nigeria
Incidence of monozygotic twins relatively constant world-wide at ~ 3.5 per 1,000 births
Incidence of dizygotic twins varies widely
Dizygotic twins
• Fertilisation of two oocytes by different sperm
• Dizygotic twins have no more resemblance than brothers / sisters of different ages
• Both zygotes implant independently in the uterus and there are two separate placentas, amniotic and chorionic sacs
• The placentas and chorionic sacs may come into close approximation and fuse.
• Dizygotic twins cannot be monochorionic and cannot be identical. They are always dichorionic and diamniotic
Monozygotic twins
• A single ovum is fertilised and splits into two at different stages of development
• Earliest separation occurs at the 2 cell stage producing two zygotes which enter the uterus independently. Each
embryo would have its own placenta and chorionic sac, forming dichorionic diamniotic twins
• In the majority of cases, splitting occurs in the early blastocyst stage. The inner cell mass splits into two, producing
two embryos with a common placenta and a common chorionic cavity but separate amniotic cavities. These form
monochorionic diamniotic twins
• Rarely, splitting occurs at the stage of the bilaminar germ disc, resulting in two embryos with a common placenta,
common chorionic cavity and common amniotic cavity. These form monochorionic monoamniotic twins
• Splitting at later stages of development may result in incomplete splitting of the axial area of the germ disc, resulting
in conjoint twins

Question 5 Dizygotic twins

Options for Questions 5-5

Can occur from the fertilisation of a single oocyte


A Can be mono-amniotic B
by two sperm
C Cannot be of the same sex D Cannot be identical
Can arise following in-vitro fertilisation and single
E
embryo replacement

A(Correct answ er: D)

Explanation

This Copy is for Dr. Mohamed ElHodiby


MULTIPLE PREGNANCY?Twins
UK data 2007: ~ 1 in 65 (1.5%) pregnancies were twins
Prevalence varies world-wide, being lowest in Japan and highest in Nigeria
Incidence of monozygotic twins relatively constant world-wide at ~ 3.5 per 1,000 births
Incidence of dizygotic twins varies widely
Dizygotic twins
• Fertilisation of two oocytes by different sperm
• Dizygotic twins have no more resemblance than brothers / sisters of different ages
• Both zygotes implant independently in the uterus and there are two separate placentas, amniotic and chorionic sacs
• The placentas and chorionic sacs may come into close approximation and fuse.
• Dizygotic twins cannot be monochorionic and cannot be identical. They are always dichorionic and diamniotic
Monozygotic twins
• A single ovum is fertilised and splits into two at different stages of development
• Earliest separation occurs at the 2 cell stage producing two zygotes which enter the uterus independently. Each
embryo would have its own placenta and chorionic sac, forming dichorionic diamniotic twins
• In the majority of cases, splitting occurs in the early blastocyst stage. The inner cell mass splits into two, producing
two embryos with a common placenta and a common chorionic cavity but separate amniotic cavities. These form
monochorionic diamniotic twins
• Rarely, splitting occurs at the stage of the bilaminar germ disc, resulting in two embryos with a common placenta,
common chorionic cavity and common amniotic cavity. These form monochorionic monoamniotic twins
• Splitting at later stages of development may result in incomplete splitting of the axial area of the germ disc, resulting
in conjoint twins

Question 6 Lymphatic drainage of the ovaries

Options for Questions 6-6

A Internal iliac nodes B External iliac nodes


C Femoral nodes D Para-aortic nodes
E Inferior mesenteric nodes

A(Correct answ er: D)

Explanation
E OVARY
• 4x2 cm, attached to the posterior aspect of the broad ligament by a mesentery - the mesovarium
• Attached to the lateral pelvic wall by the suspensory ligament of the ovary (infundibulo-pelvic ligament)
• Attached to the upper part of the lateral uterine wall by the round ligament of the ovary (remnant of upper
part of the gubenaculum)
• Position variable, but usually lies within ovarian fossa in lateral pelvic wall. The ovarian fossa is bounded
superiorly by the external iliac vessels, inferiorly by the ureter and internal iliac vessels and its floor is
crossed by the obturator nerve
• The ovary is surrounded by a thin fibrous capsule - the tunica albuginae
• Blood supply - ovarian artery - branch of abdominal aorta at L1
• Venous drainage - LEFT -left ovarian vein drains into left renal vein; RIGHT - right ovarian vein drains into
inferior vena cava
• Lymphatics - para-aortic nodes?Nerve - aortic plexus??

Question 7 Venous drainage of the left ovary

Options for Questions 7-7

A Superior mesenteric vein B Inferior vena cava


C Left renal vein D Right renal vein

This Copy is for Dr. Mohamed ElHodiby


E Inferior mesenteric vein

A(Correct answ er: C)

Explanation
THE OVARY
• 4x2 cm, attached to the posterior aspect of the broad ligament by a mesentery - the mesovarium
• Attached to the lateral pelvic wall by the suspensory ligament of the ovary (infundibulo-pelvic ligament)
• Attached to the upper part of the lateral uterine wall by the round ligament of the ovary (remnant of upper
part of the gubenaculum)
• Position variable, but usually lies within ovarian fossa in lateral pelvic wall. The ovarian fossa is bounded
superiorly by the external iliac vessels, inferiorly by the ureter and internal iliac vessels and its floor is
crossed by the obturator nerve
• The ovary is surrounded by a thin fibrous capsule - the tunica albuginae
• Blood supply - ovarian artery - branch of abdominal aorta at L1
• Venous drainage - LEFT -left ovarian vein drains into left renal vein; RIGHT - right ovarian vein drains into
inferior vena cava
• Lymphatics - para-aortic nodes?Nerve - aortic plexus??

Question 8 Lie(s) at the superior border of the ovarian fossa

Options for Questions 8-8

A External iliac artery and vein B Obturator nerve


C Internal iliac artery and vein D Sacral plexus
E Femoral nerve

A(Correct answ er: A)

Explanation
THE OVARY
• 4x2 cm, attached to the posterior aspect of the broad ligament by a mesentery - the mesovarium
• Attached to the lateral pelvic wall by the suspensory ligament of the ovary (infundibulo-pelvic ligament)
• Attached to the upper part of the lateral uterine wall by the round ligament of the ovary (remnant of upper
part of the gubenaculum)
• Position variable, but usually lies within ovarian fossa in lateral pelvic wall. The ovarian fossa is bounded
superiorly by the external iliac vessels, inferiorly by the ureter and internal iliac vessels and its floor is
crossed by the obturator nerve
• The ovary is surrounded by a thin fibrous capsule - the tunica albuginae
• Blood supply - ovarian artery - branch of abdominal aorta at L1
• Venous drainage - LEFT -left ovarian vein drains into left renal vein; RIGHT - right ovarian vein drains into
inferior vena cava
• Lymphatics - para-aortic nodes?
• Nerve - aortic plexus??

Question 9 The uterine artery

Options for Questions 9-9

A Is a branch of the internal pudendal artery B Is crossed by the ureter


C Reaches the cervix at the level of the external os D Does not enter the broad ligament
E Is a branch of the anterior division of the internal

This Copy is for Dr. Mohamed ElHodiby


iliac artery

A(Correct answ er: E)

Explanation
THE UTERUS
• 8cm long x 5cm wide x 2.5cm thick
• Covered by peritoneum except anteriorly below the reflection of the utero-vesical fold of peritoneum and laterally
between the layers of the broad ligament?
• Fundus - that part of the uterus above the entrance of the uterine tubes?
• Cavity - triangular in coronal section, cleft in saggital section?
• Anteverted uterus - long axis of uterine body at 90degrees to long axis of vagina?
• Ante-flexed uterus - long axis of the body of the uterus bent forward at the level of the internal os?
• Retroverted uterus - body and fundus bent backwards on the vagina to lie within the pouch of Douglas?
• Anterior relations: utero-vesical pouch and superior surface of bladder?
• Posterior relations: Pouch of Douglas, sigmoid colon and coils of ileum?
• Lateral relations: uterine vessels, ureter?
• Nerve - inferior hypogastric plexuses (Parasympathetic via the pelvic splanchnic nerves, sympathetic via the lumbar
splanchnic nerves). Pain sensation is transmitted via the sympathetic nerves and the lumbar splanchnic nerves??
Lymph drainage:
• Fundus - accompany ovarian artery to para-aortic nodes at the level of L1
• Body and cervix - internal and external iliac nodes

Question 10 This ligament forms two ridges on either side of the Pouch of Douglas

Options for Questions 10-10

A Round ligament B Infundibulo-pelvic ligament


C Utero-sacral ligament D Ileo-pectineal ligament
E Transverse cervical ligament

A(Correct answ er: C)

Explantion
LIGAMENTS
• Transverse cervical ligament: fibro-muscular condensations of pelvic fascia pass from the cervix and upper end of the
vagina to the lateral pelvic wall
• Utero-sacral ligament: cervix and upper end of vagina to the lower end of the sacrum - form two ridges on either side
of the pouch of Douglas
• Pubo-cervical ligament: cervix to posterior surface of pubis
• Round ligament: Of the ovary - from medial margin of ovary to upper part of lateral wall of uterus. Of the uterus - from
upper part of lateral uterine wall to deep inguinal ring
• The uterus is supported mainly by the tone of the pelvic floor muscles (levator ani) which are partly inserted onto the
perineal body and condensations of pelvic fascia forming the transverse cervical, pubo-cervical and utero-sacral
ligaments.
?
THE BROAD LIGAMENT
• Two layered fold of peritoneum extending from the lateral uterine wall to the lateral pelvic wall
• Has an upper free edge which contains the fallopian tube
• The layers of peritoneum separate inferiorly to cover the pelvic floor
• Has ovary attached to its posterior surface by the mesovarium
• Uterine artery crosses the ureter at the base (lower attached border)
• Round ligament of the uterus forms a ridge an the anterior surface
• Contains vestigial structures: epoophron and paroophron (remnant of the mesonephric system)

This Copy is for Dr. Mohamed ElHodiby


• Uterine and ovarian blood vessels and lymphatics run within it

Question 11 Which one of the above statements regarding the cervix is correct?

Options for Questions 11-11

The endocervical canal is lined by cuboidal The ectocervix is lined by stratified squamous
A B
epithelium non-keratinising epithelium
The squamo-columnar junction is located within The endocervical canal links the internal os to the
C D
the ectocervix uterine cavity
Squamous cell carcinoma of the cervix typically
E
arises from the ectocervix

A(Correct answ er: B)

Explanation
?CERVIX
• Lower, narrow portion of the uterus, connected to the uterine fundus by the uterine isthmus - upper limit is the internal
os. Made up mainly of fibrous tissue with very little smooth muscle
• Protrudes through the upper anterior vaginal wall
• Approximately half its length is visible in the vagina (vaginal cervix), the rest being above the vagina (supra-vaginal
cervix)
• The vaginal cervix ~3 cm long and 2.5 cm wide. Size and shape varies widely with age, hormonal state, and parity -
bulkier and the external with a wider and more slit-like external os in multiparous women.
• Ectocervix - portion of the cervix beyond the external os - lined by stratified squamous non-keratinising epithelium.
• Endocervical canal - links external and internal os - lined by columnar epithelium
• The squamo-columnar junction - variable location - high up the endocervical canal before puberty and in the post-
menopausal women. Site of origin of squamous cell carcinoma of the cervix
• The external os is bounded by two lips, an anterior and a posterior, of which the anterior is the shorter and thicker,
although due to the slope of the cervix, it projects lower than the posterior. Both lips are in contact with the posterior
vaginal wall
• The supravaginal cervix is separated in front from the bladder by fibrous tissue (parametrium), which extends also on
to its sides and laterally between the layers of the broad ligaments. Not covered by peritoneum on the anterior aspect
• The uterine arteries reach the margins of the cervix within the parametrium
• The ureter runs downward and forward 2 cm lateral to the supravaginal cervix
• Posteriorly, the supravaginal cervix is covered by peritoneum, which extends on to the posterior vaginal wall, when it
is reflected on to the rectum, forming the Pouch of Douglas which may contain coils of small intestine.
• The vaginal cervix projects free into the anterior wall of the vagina between the anterior and posterior fornices.
?Blood supply
• Uterine artery, branch of internal iliac
• Cervical and vaginal branches supply the cervix and upper vagina.
• The cervical branches of the uterine arteries descend on the lateral aspects of the cervix at 3 and 9 o'clock. Venous
drainage parallels the arterial supply, eventually emptying into the hypogastric venous plexus.
?Lymphatics
• Regional lymph nodes for the cervix include: paracervical, parametrial, presacral, sacral, external iliac, common iliac,
hypogastric (obturator), internal iliac.

Question 12 Somatic sensory innervation of the lower third of the vagina

Options for Questions 12-12

A Pelvic splanchnic nerves B Lumbar splanchnic nerves


C Pudendal nerve D Superficial femoral nerve
E Obturator nerve

This Copy is for Dr. Mohamed ElHodiby


A(Correct answ er: C)

Explanation
THE VAGINA
• ~8cm long, axis directed upwards and backwards from the vulva. Posterior wall longer than anterior wall
• Lined by stratified squamous epithelium which undergoes changes during the menstrual cycle. Does not secrete
mucus
• Has anterior and posterior walls which are normally in apposition, and four fornices (anterior, posterior, left and right
lateral)
• Upper half lies above the level of the pelvic floor
?Relations
• Anterior: bladder, urethra
• Posterior: upper third - pouch of Douglas; middle third - ampulla of the rectum; lower third - perineal body
• Lateral: upper part - ureter, middle part - anterior fibres of levator ani; lower part - uro-genital diaphragm and the bulb
of the vestibule
• Blood: vaginal artery, branch of internal iliac artery
• Lymphatics: upper third - internal and external iliac nodes; middle third - internal iliac nodes; lower third - superficial
inguinal nodes
• Nerve:Upper two thirds - inferior hypogastric plexuses. Parasympathetic from pelvic splanchnic nerves, sympathetic
from lumbar splanchnic nerves (L1 & 2)
• Autonomic innervation to the lower third - pudendal nerve
• Somatic sensation is present mainly in the lower third and is carried by the pudendal nerve

Question 13 Parasympathetic supply to the lower third of the vagina

Options for Questions 13-13

A Pelvic splanchnic nerves B Lumbar splanchnic nerves


C Pudendal nerve D Superficial femoral nerve
E Obturator nerve

A(Correct answ er: C)

Explanation
THE VAGINA
• ~8cm long, axis directed upwards and backwards from the vulva. Posterior wall longer than anterior wall
• Lined by stratified squamous epithelium which undergoes changes during the menstrual cycle. Does not secrete
mucus
• Has anterior and posterior walls which are normally in apposition, and four fornices (anterior, posterior, left and right
lateral)
• Upper half lies above the level of the pelvic floor
?Relations
• Anterior: bladder, urethra
• Posterior: upper third - pouch of Douglas; middle third - ampulla of the rectum; lower third - perineal body
• Lateral: upper part - ureter, middle part - anterior fibres of levator ani; lower part - uro-genital diaphragm and the bulb
of the vestibule
• Blood: vaginal artery, branch of internal iliac artery
• Lymphatics: upper third - internal and external iliac nodes; middle third - internal iliac nodes; lower third - superficial
inguinal nodes
• Nerve:Upper two thirds - inferior hypogastric plexuses. Parasympathetic from pelvic splanchnic nerves, sympathetic
from lumbar splanchnic nerves (L1 & 2)
• Autonomic innervation to the lower third - pudendal nerve
• Somatic sensation is present mainly in the lower third and is carried by the pudendal nerve

This Copy is for Dr. Mohamed ElHodiby


Question 14 Lies posterior to the lower third of the vagina

Options for Questions 14-14

A Urethra B Pouch of Douglas


C Ampulla of the rectum D Perineal body
E Ureter

A(Correct answ er: D)

Explanation
THE VAGINA
• ~8cm long, axis directed upwards and backwards from the vulva. Posterior wall longer than anterior wall
• Lined by stratified squamous epithelium which undergoes changes during the menstrual cycle. Does not secrete
mucus
• Has anterior and posterior walls which are normally in apposition, and four fornices (anterior, posterior, left and right
lateral)
• Upper half lies above the level of the pelvic floor
?Relations
• Anterior: bladder, urethra
• Posterior: upper third - pouch of Douglas; middle third - ampulla of the rectum; lower third - perineal body
• Lateral: upper part - ureter, middle part - anterior fibres of levator ani; lower part - uro-genital diaphragm and the bulb
of the vestibule
• Blood: vaginal artery, branch of internal iliac artery
• Lymphatics: upper third - internal and external iliac nodes; middle third - internal iliac nodes; lower third - superficial
inguinal nodes
• Nerve:Upper two thirds - inferior hypogastric plexuses. Parasympathetic from pelvic splanchnic nerves, sympathetic
from lumbar splanchnic nerves (L1 & 2)
• Autonomic innervation to the lower third - pudendal nerve
• Somatic sensation is present mainly in the lower third and is carried by the pudendal nerve

Question 15 The shape of the pelvic outlet in females

Options for Questions 15-15

A Oval shaped B Circular shaped


C Diamond shaped D Heart shaped
E Quadrilateral shaped

A(Correct answ er: C)

Explanation
TRUE PELVIS
That part of the pelvis between the pelvic inlet and the pelvic outlet
The pelvic inlet is oval in shape in the Gynaecoid pelvis and has the following borders:
1) Posterior: Sacral promontory?2) Lateral: Iliopectineal line?3)Anterior: Symphysis pubis
The pelvic outlet is diamond shaped with the following borders:
• Posterior: Coccyx
• Lateral: Ischial tuberosities
• Anterior: Pubic arch formed by the simphysis pubis and the ischio-pubic rami
• Part of the lateral border is formed by the sacro-spinous and sacro-tuberous ligaments which convert the greater and
lesser sciatic notches into the greater and lesser sciatic foramina
• The pelvic cavity has a shallow anterior wall and a deeper posterior wall. The ischial spines are at the level of the
mid-cavity
• The acetabulum is formed from the ilium, ischium and pubic bones

This Copy is for Dr. Mohamed ElHodiby


Question 16 The sacro-iliac joint

Options for Questions 16-16

A Fibrous joint B Cartilaginous joint


C Synovial joint D Fibro-cartilagenous joint
E Ball & socket joint

A(Correct answ er: C)

Explanation
THE SACRO-ILIAC JOINT
• Synovial joint - the irregular articular surfaces of the joint make a contribution to joint stability but this is mainly
maintained by the very strong posterior and inter-osseous sacro-iliac ligaments.
• The sacro-spinous and sacro-tuberous ligaments also contribute to joint stability
• Supplied by branches of the sacral plexus and POSTERIOR rami of S1

Question 17 Which one of the above statements about the female pelvis is correct?

Options for Questions 17-17

The cavity of the female pelvis is deeper than that The sacrum is longer in the female pelvis than in
A B
of the male pelvis the male pelvis
The sacrum is wider in the female pelvis than in The ischial spines are more prominent in the
C D
the male pelvis female pelvis than in the male pelvis
The sciatic notches are narrower in the female
E
pelvis than in the male pelvis

A(Correct answ er: C)

Explanation
?Differences between the Male and Female Pelvis?The female pelvis:
• Less massive
• The anterior iliac spines more widely separated - greater lateral prominence of the hips.
• Wider pelvic inlet - both antero-posterior and transverse diameters
• Pelvic inlet more circular
• More shallow
• The sacrum is shorter wider, and its upper part is less curved
• The obturator foramina are triangular in shape and smaller in size
• The outlet is larger and the coccyx more movable.
• The sciatic notches are wider and shallower
• The ischial spines are less prominent.
• The pubic symphysis is less deep, and the pubic arch is wider and more rounded than in the male?

Question 18 Which one of the above organs is correctly paired with its parasympathetic nerve supply?

Options for Questions 18-18

A Urinary bladder – lumbar splanchnic nerves B Small intestine – pelvic splanchnic nerves
C Descending colon – vagus nerve D Vagina – lumbar splanchnic nerves
E Ovary – vagus nerve

This Copy is for Dr. Mohamed ElHodiby


A(Correct answ er: E)

Explanation
PELVIC SPLANCHNIC NERVES
• Formed from the ventral (anterior) primary rami of S2 through S4.
• These are the ways in which parasympathetic neurons reach the hypogastric plexus, and therefore the pelvic viscera
and distal colon.
• The parasympathetic part of the autonomic nervous system is the "craniosacral" part. Parasympathetic innervation to
most of the gut comes from the "cranio-" half of that, i.e., the vagus nerve. The rest, to colon distal to the splenic
flexure and to pelvic viscera, is from the "-sacral" half, via the pelvic splanchnic nerves
• There are thoracic, lumbar, sacral, and pelvic splanchnic nerves.
• "Splanchnic" refers to nerves that supply viscera.
• Thoracic, lumbar and sacral splanchnic nerves emerge from sympathetic ganglia and carry sympathetic fibers
• Pelvic splanchnic nerves are parasympathetic
• Contain pre-ganglionic fibres
• Join inferior hypogastric plexus
• Some fibres ascend to the superior hypogastric and eventually inferior mesenteric plexus and supply the hind-gut
• Provide parasympathetic supply to the pelvic viscera
• Afferent impulses from the pelvic viscera are transmitted mainly by the sympathetic pathway
• Parasympathetic nerves innervate detrusor and internal sphincter via the pelvic splanchnic nerves (S2,3,4) and also
innervate the external sphincter via the pudendal nerve - initiate micturiction by inhibiting internal sphincter activity
and stimulating detrusor contraction
• Parasympathetic supply to the ovary is from the vagus nerve. Sympathetic innervation is from the L1/2 segment
• Pain from uterine contractions goes back to T10-L1. This means that uterine contraction pain is sympathetic. But for
the lower portion of the uterus and upper vagina (the cervix) - it is parasympathetic back to the pelvic splanchnic
nerves

Question 19 With respect to the anatomy of the female external genitalia

Options for Questions 19-19

The labia majora fuse posteriorly to form the The vestibule is bound laterally by the labia
A B
fourchette majora
The fourchette is located at the apex of the
C The clitoris is located at the base of the vestibule D
vestibule
The urethra opens within the vestibule posterior to
E
the clitoris

A(Correct answ er: E)

Explanation
EXTERNAL GENITALIA
• Labia majora: prominent hair-bearing folds of skin extending from the mons pubis to fuse posteriorly in the
mid-line
• Labia minora: hairless folds of skin within the labia majora; unite posteriorly to form the fourchette; split
anteriorly to enclose the clitoris, forming an anterior prepuce and a posterior frenulum
• Vestibule: triangular area bounded laterally by the labia minora with the clitoris at its apex and the fourchette
at its base
• The urethra opens within the vestibule posterior to the clitoris

Question 20 The lower half of the anal canal is sensitive to these stimuli

Options for Questions 20-20

This Copy is for Dr. Mohamed ElHodiby


A Stretch B Stretch and touch
C stretch, touch and temperature D stretch, touch, temperature and pain
E stretch and pain

A(Correct answ er: D)

Explanation
THE ANAL CANAL
• ~4cm long, extends downwards and backwards from the rectal ampulla to the anus
• LATERAL walls kept in apposition by the levator ani and anal sphincter except during defecation
• Anterior relations: perineal body, urogenital diaphragm and perineal body
• Posterior relations: anococcygeal body and the coccyx
?UPPER HALF
• Derived from hind-gut endoderm
• Lined by columnar epithelium
• Thrown into vertical folds called anal columns
• Autonomic supply from the inferior hypogastric plexus, sensitive to stretch only
• Blood - superior rectal branch of inferior mesenteric artery
• Lymphatics - inferior mesenteric nodes
• Dentate line - boundary between endodermal and ectodermal origin of anal canal
?LOWER HALF
• Derived from the ectoderm of the proctodeum
• Lined by stratified squamous epithelium
• No anal columns
• Blood - inferior rectal artery, branch of internal pudendal artery
• Lymphatics - medial group of superficial inguinal nodes
• Nerve - inferior rectal nerve - sensitive to temperature, pain, touch and pressure

Question 21 Which one of the above is not a branch of the pudendal nerve?

Options for Questions 21-21

A Perineal nerve B Dorsal nerve of the clitoris


C Inferior rectal nerve D Nerve to obturator internus
E None of the above

A(Correct answ er: D)

Explanation
PUDENDAL NERVE
• Branch of sacral plexus (S2,3,4)
• Leaves pelvic cavity through greater sciatic foramen
• Enters perineum through lesser sciatic foramen
• Enters perineum below and medial to the ischial spine
• Lies medial to the internal pudendal artery as it exits from the pelvis into the perineum
Branches
• Inferior rectal nerve - supplies external anal sphincter and mucous membrane of the lower half of the anal canal
• Dorsal nerve of the clitoris (penis)
• Perineal nerve - supplies muscles of the urogenital triangle and the skin on the posterior surface of the labia majora

Question 22 Blood supply to the lateral part of the abdominal wall above the umbilicus

Options for Questions 22-22

This Copy is for Dr. Mohamed ElHodiby


A Superior epigastric artery B Inferior epigastric artery
C Superior mesenteric artery D Inferior mesenteric artery
E Intercostal arteries

A(Correct answ er: E)

Explanation
Blood supply – abdominal wall
• Medial aspects - Superior epigastric artery (above umbilicus - continuation of the internal thoracic artery, branch of
the first part of the subclavian artery. Enters the rectus sheath between the sternal and costal origins of the
diaphragm and descends behind rectus muscle)
• Inferior epigastric artery (below umbilicus - branch of external iliac artery just above inguinal ligament. It pierces the
transversalis fascia to enter the rectus sheath anterior to the arcuate line; runs behind rectus muscle). There is no
anastomosis between the arteries of the left and right side
• Lateral aspects - intercostals, lumbar arteries and deep circumflex iliac artery (branch of external iliac artery above
the inguinal ligament).

Question 23 Blood supply to the medial part of the anterior abdominal wall above the umbilicus

Options for Questions 23-23

A Superior epigastric artery B Inferior epigastric artery


C Superior mesenteric artery D Inferior mesenteric artery
E Intercostal arteries

A(Correct answ er: A)

Explanation
Blood supply – abdominal wall
• Medial aspects - Superior epigastric artery (above umbilicus - continuation of the internal thoracic artery, branch of
the first part of the subclavian artery. Enters the rectus sheath between the sternal and costal origins of the
diaphragm and descends behind rectus muscle)
• Inferior epigastric artery (below umbilicus - branch of external iliac artery just above inguinal ligament. It pierces the
transversalis fascia to enter the rectus sheath anterior to the arcuate line; runs behind rectus muscle). There is no
anastomosis between the arteries of the left and right side
• Lateral aspects - intercostals, lumbar arteries and deep circumflex iliac artery (branch of external iliac artery above
the inguinal ligament).

Question 24 The transversus abdominis muscle does not originate from which one of the above structures?

Options for Questions 24-24

A Outer surface of lower 8 ribs B Deep surface of lower 6 costal cartilages


C Lumbar fascia D Lateral 1/3 of inguinal ligament
E Anterior 2/3 of iliac crest

A(Correct answ er: A)

Explanation
Transversus abdominis

This Copy is for Dr. Mohamed ElHodiby


• ORIGIN: Deep surface of lower 6 costal cartilages, lumbar fascia, anterior 2/3 of iliac crest and lateral 1/3 of inguinal
ligament
• INSERTION: Xiphoid process, linea alba and symphysis pubis
• Conjoint tendon formed by insertion of internal oblique and transversus abdominis onto pubic crest and pectineal line

Question 25 Nerve supply to the internal oblique muscle

Options for Questions 25-25

A Upper 6 thoracic nerves B Lower 6 thoracic nerves


C Upper 3 lumbar nerves D Lower 3 lumbar nerves
E 12th thoracic nerve

A(Correct answ er: B)

Explanation
NERVES
• External oblique, internal oblique and transversus: Lower 6 thoracic nerves and L1 (ileoinguinal and ileohypogastric
nerves)
• Rectus abdominis: Lower 6 thoracic nerves
• Pyramidalis: T12
FUNCTION
• External / internal oblique / transversus - laterally flex and rotate the trunk, relax during inspiration to accommodate
abdominal viscera, contract during micturiction, defecation and vomiting.
• Rectus abdominis - flexes the trunk and stabilises the pelvis

Question 26 Which one of the above descriptions of the arcuate line is correct?

Options for Questions 26-26

Insertion of the inguinal ligament on the pubic


A Lateral margins of the rectus abdominis muscle B
bone
Point where the aponeurosis of the internal
Point where the aponeurosis of the external
C oblique muscle passes anterior to the rectus D
oblique passes posterior to the rectus abdominis
abdominis
Attachment of the ileo-pectineal ligament on the
E
superior pubic ramus

A(Correct answ er: C)

Explanation
RECTUS SHEATH
Above costal margin
• Anterior wall: Aponeurosis of external oblique
• Posterior wall: thoracic wall.
Between costal margin and anterior superior iliac spine
• Anterior wall: Aponeurosis of external and internal oblique
• Posterior wall: Aponeurosis of internal oblique and transversus abdominis
• Note that the aponeurosis of the internal oblique splits to enclose the rectus abdominis
Between anterior superior iliac spine and pubis
• Anterior wall: Aponeuroses of external, internal oblique and transversus
• Posterior wall: Transversalis fascia

This Copy is for Dr. Mohamed ElHodiby


• Arcuate line : Site where the aponeuroses of the posterior wall pass anterior to the rectus at the level of the anterior
superior iliac spine. The inferior epigastric artery enters the rectus sheath at this point and lies posterior to the rectus
abdominis. Pyramidalis lies within the rectus sheath.

Question 27 Posterior wall of the rectus sheath above the costal margin

Options for Questions 27-27

A Aponeurosis of external oblique B Aponeurosis of internal oblique


C Thoracic wall D Aponeuroses of external and internal oblique
E Aponeurosis of transversus abdominis

A(Correct answ er: C)

Explanation
RECTUS SHEATH
Above costal margin
• Anterior wall: Aponeurosis of external oblique
• Posterior wall: thoracic wall.
Between costal margin and anterior superior iliac spine
• Anterior wall: Aponeurosis of external and internal oblique
• Posterior wall: Aponeurosis of internal oblique and transversus abdominis
• Note that the aponeurosis of the internal oblique splits to enclose the rectus abdominis
Between anterior superior iliac spine and pubis
• Anterior wall: Aponeuroses of external, internal oblique and transversus
• Posterior wall: Transversalis fascia
• Arcuate line : Site where the aponeuroses of the posterior wall pass anterior to the rectus at the level of the anterior
superior iliac spine. The inferior epigastric artery enters the rectus sheath at this point and lies posterior to the rectus
abdominis. Pyramidalis lies within the rectus sheath.

Question 28 The main action of the psoas major muscle

Options for Questions 28-28

A Medially rotate the hip joint B Laterally rotate the hip joint
C Flex the hip joint D Adduct the hip joint
E Adduct the hip joint

A(Correct answ er: C)

Explanation
PSOAS MAJOR
• Origin: Roots of transverse processes, sides of vertebral bodies and inter-vertebral discs T12 - L5
• Insertion: Lesser trochanter of the femur
• Nerve: Lumbar plexus
• Action: Flexes thigh on trunk. If thigh if flexed, flexes trunk on thigh.

ILIACUS
• Origin: Iliac fossa
• Insertion: Lesser trochanter of the femur
• Nerve: Femoral nerve
• Action: Flexes thigh on trunk. If thigh is flexed, flexes trunk on thigh

This Copy is for Dr. Mohamed ElHodiby


Question 29 The origin of the iliacus muscle

Options for Questions 29-29

A Greater trochanter B Lesser trochanter


C Iliac crest D Iliac fossa
E Anterior superior iliac spine

A(Correct answ er: D)

Explanation
ILIACUS
• Origin: Iliac fossa
• Insertion: Lesser trochanter of the femur
• Nerve: Femoral nerve
• Action: Flexes thigh on trunk. If thigh is flexed, flexes trunk on thigh

Question 30 The cremaster muscle

Options for Questions 30-30

A Is derived from the external oblique muscle B Is derived from the internal oblique muscle
C Is derived from the transversus abdominis muscle D Is supplied by the femoral nerve
E Is supplied by branches of the femoral artery

A(Correct answ er: B)

Explanation
Cremaster muscle / cremasteric reflex
• Derived from internal oblique
• Supplied by cremasteric artery - branch of inferior epigastric
• Supplied by genital branch of genitor-femoral nerve
• Cremasteric reflex - cremaster muscle contracts when skin on medial aspect of thigh is stroked. Afferent - femoral
branch; Efferent - genital branch of genitor-femoral nerve

This Copy is for Dr. Mohamed ElHodiby


Question 1 With respect to the anatomy of the perineum

Options for Questions 1-1

The anal triangle is bound laterally by the anterior


A The anal triangle is bound posteriorly by the anus B
superior iliac spines
The skin of the anal triangle is supplied by the Lymphatic drainage of the anal triangle is to the
C D
superior rectal nerve superficial inguinal nodes
The opening of the urethra lies anterior to the
E
clitoris

A(Correct answ er: D)

Explanation
THE ANAL TRIANGLE
• Bounded posteriorly by the tip of the coccyx and laterally by the ischial tuberosities
• Skin supplied by inferior rectal nerve
• Lymphatic drainage is to the medial group of superficial inguinal nodes

Question 2 Which one of the above most accurately describes the Dentate line?

Options for Questions 2-2

A Mucosal fold overlying inferior rectal artery B Mucosal fold over anal columns
Boundary between ectodermal and mesodermal Boundary between ectodermal and endodermal
C D
origin of anal canal origin of anal canal
Boundary between mid-gut and hind-gut origins of
E
the anal canal

A(Correct answ er: D)

Explanation
THE ANAL CANAL
• ~4cm long, extends downwards and backwards from the rectal ampulla to the anus
• LATERAL walls kept in apposition by the levator ani and anal sphincter except during defecation
• Anterior relations: perineal body, urogenital diaphragm and perineal body
• Posterior relations: anococcygeal body and the coccyx
?UPPER HALF
• Derived from hind-gut endoderm
• Lined by columnar epithelium
• Thrown into vertical folds called anal columns
• Autonomic supply from the inferior hypogastric plexus, sensitive to stretch only
• Blood - superior rectal branch of inferior mesenteric artery
• Lymphatics - inferior mesenteric nodes
• Dentate line - boundary between endodermal and ectodermal origin of anal canal
?LOWER HALF
• Derived from the ectoderm of the proctodeum
• Lined by stratified squamous epithelium
• No anal columns
• Blood - inferior rectal artery, branch of internal pudendal artery
• Lymphatics - medial group of superficial inguinal nodes

This Copy is for Dr. Mohamed ElHodiby


• Nerve - inferior rectal nerve - sensitive to temperature, pain, touch and pressure

Question 3 Arterial supply to the lower half of the anal canal

Options for Questions 3-3

A Inferior rectal artery B Pudendal artery


Superior rectal branch of inferior mesenteric
C Obturator artery D
artery
E Middle rectal artery

A(Correct answ er: A)

Explanation
THE ANAL CANAL
• ~4cm long, extends downwards and backwards from the rectal ampulla to the anus
• LATERAL walls kept in apposition by the levator ani and anal sphincter except during defecation
• Anterior relations: perineal body, urogenital diaphragm and perineal body
• Posterior relations: anococcygeal body and the coccyx
?UPPER HALF
• Derived from hind-gut endoderm
• Lined by columnar epithelium
• Thrown into vertical folds called anal columns
• Autonomic supply from the inferior hypogastric plexus, sensitive to stretch only
• Blood - superior rectal branch of inferior mesenteric artery
• Lymphatics - inferior mesenteric nodes
• Dentate line - boundary between endodermal and ectodermal origin of anal canal
?LOWER HALF
• Derived from the ectoderm of the proctodeum
• Lined by stratified squamous epithelium
• No anal columns
• Blood - inferior rectal artery, branch of internal pudendal artery
• Lymphatics - medial group of superficial inguinal nodes
• Nerve - inferior rectal nerve - sensitive to temperature, pain, touch and pressure

Question 4 Which one of the above lies anterior to the anal canal?

Options for Questions 4-4

A Upper 2/3 of the vagina B Anococcygeal body


C Coccyx D Perineal body
E Pouch of Douglas

A(Correct answ er: D)

Explanation
THE ANAL CANAL
• ~4cm long, extends downwards and backwards from the rectal ampulla to the anus
• LATERAL walls kept in apposition by the levator ani and anal sphincter except during defecation
• Anterior relations: perineal body, urogenital diaphragm and perineal body
• Posterior relations: anococcygeal body and the coccyx
?UPPER HALF
• Derived from hind-gut endoderm

This Copy is for Dr. Mohamed ElHodiby


• Lined by columnar epithelium
• Thrown into vertical folds called anal columns
• Autonomic supply from the inferior hypogastric plexus, sensitive to stretch only
• Blood - superior rectal branch of inferior mesenteric artery
• Lymphatics - inferior mesenteric nodes
• Dentate line - boundary between endodermal and ectodermal origin of anal canal
?LOWER HALF
• Derived from the ectoderm of the proctodeum
• Lined by stratified squamous epithelium
• No anal columns
• Blood - inferior rectal artery, branch of internal pudendal artery
• Lymphatics - medial group of superficial inguinal nodes
• Nerve - inferior rectal nerve - sensitive to temperature, pain, touch and pressure

Question 5 Forms the base of the ischio-rectal fossa

Question 6 Forms the lateral wall of the ischio-rectal fossa

Question 7 Forms the medial wall of the ischio-rectal fossa

Options for Questions 5-7

A Skin B Anal canal


C Obturator internus muscle D Vagina
E Rectum

A(Correct answ er: A)

A(Correct answ er: C)

A(Correct answ er: B)

Explanation
SCHIO-RECTAL FOSSA
• Wedge shaped space filled with dense fat
• Base formed by skin
• Medial wall formed by anal canal and levator ani muscles
• Lateral wall formed by lower part of obturator internus muscle
• Contains the pudendal nerve and internal pudendal vessels within the pudendal canal on the lateral wall

Question 8 The urogenital diaphragm

Options for Questions 8-8

Has fascial layers that fuse anteriorly with the Has fascial layers that are attached laterally to the
A B
simphysis pubis ischial tuberosities
C Contains the sphincter urethrae D Contains the clitoris
E Contains the anococcygeal body

A(Correct answ er: C)

Explanation
UROGENITAL DIAPHRAGM

This Copy is for Dr. Mohamed ElHodiby


• Musculo-fascial diaphragm with a superior and inferior fascial layer containing the deep transverse perineal muscle
and the sphincter urethrae
• The two layers fuse anteriorly, leaving a gap beneath the simphysis pubis
• The two layers fuse posteriorly with the perineal body
• The two layers are attached laterally to the pubic arch
• The enclosed space between the two fascial layers is the deep perineal pouch
?DEEP PERINEAL POUCH
• Enclosed space between superior and inferior fascial layers of the urogenital diaphragm
• Contains:
1. The sphincter urethrae and the deep transverse perineal muscle?
2. Part of the vagina?
3. Part of the urethra?
4. Internal pudendal vessels?
5. Dorsal nerve of the clitoris??

Question 9 The deep transverse perineal muscle


Options for Questions 9-9

A Originates from the perineal body B Is inserted onto the body of the pubic bone
C Is supplied by the obturator nerve D Fixes the perineal body within the perineum
E Lies within the superficial perineal pouch

A(Correct answ er: D)

Explanation
MUSCLES OF THE DEEP PERINEAL POUCH??
Deep transverse perineal muscle
• Origin - ischial ramus
• Insertion - perineal body
• Nerve - perineal branch of pudendal nerve
• Action - fixes perineal body

Sphincter urethrae
• Origin - pubic arch
• Insertion - surrounds urethrae
• Nerve - perineal branch of pudendal nerve
• Action - voluntary control of micturiction

Question 10 The sphincter urethrae

Options for Questions 10-10

A Is composed of smooth muscle B Originates from the perineal body


Is supplied by the perineal branch of the pudendal
C Is inserted onto the body of the pubic bone D
nerve
E Lies within the superficial perineal pouch

A(Correct answ er: D)

Explanation
MUSCLES OF THE DEEP PERINEAL POUCH??
Deep transverse perineal muscle
• Origin - ischial ramus

This Copy is for Dr. Mohamed ElHodiby


• Insertion - perineal body
• Nerve - perineal branch of pudendal nerve
• Action - fixes perineal body

Sphincter urethrae
• Origin - pubic arch
• Insertion - surrounds urethrae
• Nerve - perineal branch of pudendal nerve
• Action - voluntary control of micturiction

This Copy is for Dr. Mohamed ElHodiby


Question 1 The second wave of trophoblast invasion occurs at this gestation

Options for Questions 1-1

A 4-6 weeks B 8-12 weeks


C 12-16 weeks D 16-18 weeks
E 22-24 weeks

A(Correct answ er: D)

Explanation
DEVELOPMENT OF THE VILLOUS TREE
• Primary villous stems become infiltrated by cytotrophoblasts between days 13-21 post-ovulation
• Villous stems are subsequently infiltrated by extra-embryonic mysenchyme which differentiates into fetal blood
vessels
• The distal parts of the villous stems are not vascularised. Here, cytotrophoblasts proliferate and spread laterally to
form a cytotrophoblastic shell, splitting the syncytiotrophoblast into a definitive syncytiotrophoblast on the fetal side
and the peripheral syncytium on the decidual side which degenerates and is replaced by fibrinoid material (Nitabuch's
layer)
• Sprouts extend from primary villous stems, initially made up of syncytiotrophoblast and then infiltrated by
cytotrophoblast and mesenchyme - these are primary stem villi and the placenta is a true villous structure by day 21
of gestation. These villi grow and divide into secondary, tertiary and terminal villi
• The villi oriented towards the uterine cavity degenerate between day 21 and the 4th month to form the chorion laeve.
The overlying decidua degenerates and the chorion laeve comes in contact with the deciduas of the opposite uterine
wall
• The rest of the villi form the chorion frondosum which develops into the definitive placenta
• Division and modification of the villous tree continues until term. First trimester villi are larger, have a complete layer
of cytotrophoblasts and have a loose mysenchymal core which is vascularised towards the end of the first trimester
• At term, the villi are smaller, cytotrophoblasts are few in number, the syncytiotrophoblast is irregularly thinned. Fetal
vessels are sinusoidal and occupy most of the villous core and lie close to the syncytiotrophoblast, forming
vasculusyncytial membranes which maximise materno-fetal transfer.
• Sometimes, the syncytiotrophoblast nuclei appear in clusters called syncytial knots - more common in placentas from
IUGR / pre-eclamptic pregnancies
• Maternal blood is separated from fetal blood by the syncytiotrophoblast and the fetal capillary endothelium

Question 2 Amniotic fluid

Options for Questions 2-2

Has a glucose concentration which is higher than


A Has a pH which is lower than that of fetal blood B
that of maternal blood
C Does not contain albumin D Does not contain globulins
E Has a high fibrinogen concentration

A(Correct answ er: A)

Explanation
AMNIOTIC FLUID
• Initially produced by primitive cells around the amniotic vesicle
• Later formed from transudate from fetal skin and umbilical cord and diffusion across the amniotic membrane
• Fetal skin becomes keratinised in the second trimester and amniotic fluid is mainly formed from fetal urine
and lung secretions. The term fetus passes 500-700ml urine per day

This Copy is for Dr. Mohamed ElHodiby


• Fetal swallowing is a major route of amniotic fluid re-circulation and begins at 12 weeks. At term. ~500ml
amniotic fluid is exchanged / 24h
• Amniotic fluid volume = 50ml at 12 weeks, 150ml at 16 weeks gestation and ~1000ml at term. Peak volume
is at 32-36 weeks
• Osmolarity: 275 mOsmol/l (lower than maternal or fetal), decreases as pregnancy progresses
• Cells: at term, contains fetal epithelial cells, amniocytes and dermal fibroblasts. Epithelial cells and
amniocytes grow poorly in culture. Glial cells present if neural tube defect
• Protein: concentration increases with gestation but plateaus after 30 weeks. Mainly albumin and globulins.
Also contains AFP (1/10TH concentration in fetal blood - rises until 12 weeks then declines). Virtually no
fibrinogen.
• Urea, creatinine and urate concentration increases with gestation
• Amino acids: concentration similar to that in maternal plasma
• Lipids: mainly free fatty acids. Also contains phospholipids, cholesterol and lecithin (secreted by lungs during
maturation)
• Carbohydrates: mainly glucose; concentration ~ half that of maternal serum
• PO2 = 2-15mmHg while PCO2 = 50-60mmHg
• pH = 7.0 (acidic relative to fetal blood)
• Bilirubin concentration falls in the third trimester (except in haemolytic disease)

Question 3 Concentrations of this hormone peak in the first trimester and rise again at term

Options for Questions 3-3

A Inhibin A B Activin
C Human chorionic gonadotrophin D Human placental lactogen
E Progesterone

A(Correct answ er: A)

Explanation
INHIBIN & ACTIVIN
Produced by the feto-placental unit (mainly by the ovary in non-pregnant state)
Inhibin-A levels peak in early pregnancy and rise again at term and are increased in pre-eclampsia
Activin levels increase with gestation age and a marked increase occurs with the onset of labour and in pre-
eclampsia

Question 4 Which one of the above is not readily transferred across the placenta?

Options for Questions 4-4

A Amino acids B Glucose


C Thyroxine D IgG
E Ca2+

A(Correct answ er: C)

Explanation
Molecule Placental transfer

Testosterone Minimal transfer - androgens aromatised by placenta. Very high maternal androgen
concentration may virilise female fetus

This Copy is for Dr. Mohamed ElHodiby


Ca2+, Mg2+ Active transfer against concentration gradient

PTH, Calcitonin Not transferred

Vitamin D Good transfer

IgA Minimal passive transfer

IgG Good active and active transfer from 7 weeks gestation

IgM No transfer

Glucose Facilitated diffusion - excellent transfer

Amino acids Active transport - excellent transfer

Free fatty acids Very limited transfer - essential fatty acids only

Ketone bodies Excellent transfer - diffusion

Insulin,
No transfer
glucagon
Thyroid
Poor transfer - diffusion
hormone

TRH Excellent transfer

Iodine and
Excellent transfer
thioamides
Cortisol &
Excellent transfer
aldosterone

ACTH No transfer

Question 5 Which one of the above is readily transferred across the placenta?

Options for Questions 5-5

A Testosterone B Parathyroid hormone


C Vitamin D D IgM
E Calcitonin

A(Correct answ er: C)

Explanation

Molecule Placental transfer

Testosterone Minimal transfer - androgens aromatised by placenta. Very high maternal androgen
concentration may virilise female fetus

Ca2+, Mg2+ Active transfer against concentration gradient

PTH, Calcitonin Not transferred

This Copy is for Dr. Mohamed ElHodiby


Vitamin D Good transfer

IgA Minimal passive transfer

IgG Good active and active transfer from 7 weeks gestation

IgM No transfer

Glucose Facilitated diffusion - excellent transfer

Amino acids Active transport - excellent transfer

Free fatty acids Very limited transfer - essential fatty acids only

Ketone bodies Excellent transfer - diffusion

Insulin,
No transfer
glucagon
Thyroid
Poor transfer - diffusion
hormone

TRH Excellent transfer

Iodine and
Excellent transfer
thioamides
Cortisol &
Excellent transfer
aldosterone

ACTH No transfer

Question 6 With respect to twin pregnancies

Options for Questions 6-6

1 in 150 pregnancies in the UK are twin The prevalence of twin pregnancies is highest in
A B
pregnancies the Far East
The incidence of monozygotic twins is relatively Monozygotic twin pregnancies account for 1 in 65
C D
constant world-wide pregnancies in the UK
E Dizygotic twins can develop from the same oocyte

A(Correct answ er: C)

Explanation
MULTIPLE PREGNANCY?Twins
UK data 2007: ~ 1 in 65 (1.5%) pregnancies were twins
Prevalence varies world-wide, being lowest in Japan and highest in Nigeria
Incidence of monozygotic twins relatively constant world-wide at ~ 3.5 per 1,000 births
Incidence of dizygotic twins varies widely
Dizygotic twins
• Fertilisation of two oocytes by different sperm
• Dizygotic twins have no more resemblance than brothers / sisters of different ages
• Both zygotes implant independently in the uterus and there are two separate placentas, amniotic and chorionic sacs
• The placentas and chorionic sacs may come into close approximation and fuse.
• Dizygotic twins cannot be monochorionic and cannot be identical. They are always dichorionic and diamniotic
Monozygotic twins
• A single ovum is fertilised and splits into two at different stages of development
• Earliest separation occurs at the 2 cell stage producing two zygotes which enter the uterus independently. Each
embryo would have its own placenta and chorionic sac, forming dichorionic diamniotic twins

This Copy is for Dr. Mohamed ElHodiby


• In the majority of cases, splitting occurs in the early blastocyst stage. The inner cell mass splits into two, producing
two embryos with a common placenta and a common chorionic cavity but separate amniotic cavities. These form
monochorionic diamniotic twins
• Rarely, splitting occurs at the stage of the bilaminar germ disc, resulting in two embryos with a common placenta,
common chorionic cavity and common amniotic cavity. These form monochorionic monoamniotic twins
• Splitting at later stages of development may result in incomplete splitting of the axial area of the germ disc, resulting
in conjoint twins

Question 7 Lymphatic drainage of the ovaries

Options for Questions 7-7

A Internal iliac nodes B External iliac nodes


C Femoral nodes D Para-aortic nodes
E Inferior mesenteric nodes

A(Correct answ er: D)

Explanation
THE OVARY
• 4x2 cm, attached to the posterior aspect of the broad ligament by a mesentery - the mesovarium
• Attached to the lateral pelvic wall by the suspensory ligament of the ovary (infundibulo-pelvic ligament)
• Attached to the upper part of the lateral uterine wall by the round ligament of the ovary (remnant of upper
part of the gubenaculum)
• Position variable, but usually lies within ovarian fossa in lateral pelvic wall. The ovarian fossa is bounded
superiorly by the external iliac vessels, inferiorly by the ureter and internal iliac vessels and its floor is
crossed by the obturator nerve
• The ovary is surrounded by a thin fibrous capsule - the tunica albuginae
• Blood supply - ovarian artery - branch of abdominal aorta at L1
• Venous drainage - LEFT -left ovarian vein drains into left renal vein; RIGHT - right ovarian vein drains into
inferior vena cava
• Lymphatics - para-aortic nodes?Nerve - aortic plexus??

Question 8 Venous drainage of the left ovary

Options for Questions 8-8

A Superior mesenteric vein B Inferior vena cava


C Left renal vein D Right renal vein
E Inferior mesenteric vein

A(Correct answ er: C)

Explanation
THE OVARY
• 4x2 cm, attached to the posterior aspect of the broad ligament by a mesentery - the mesovarium
• Attached to the lateral pelvic wall by the suspensory ligament of the ovary (infundibulo-pelvic ligament)
• Attached to the upper part of the lateral uterine wall by the round ligament of the ovary (remnant of upper
part of the gubenaculum)
• Position variable, but usually lies within ovarian fossa in lateral pelvic wall. The ovarian fossa is bounded
superiorly by the external iliac vessels, inferiorly by the ureter and internal iliac vessels and its floor is
crossed by the obturator nerve
• The ovary is surrounded by a thin fibrous capsule - the tunica albuginae

This Copy is for Dr. Mohamed ElHodiby


• Blood supply - ovarian artery - branch of abdominal aorta at L1
• Venous drainage - LEFT -left ovarian vein drains into left renal vein; RIGHT - right ovarian vein drains into
inferior vena cava
• Lymphatics - para-aortic nodes?Nerve - aortic plexus??

Question 9 The blood supply to the fallopian tube

Options for Questions 9-9

A Ovarian artery B Uterine artery


C Ovarian and uterine arteries D External iliac artery
E Internal iliac artery

A(Correct answ er: C)

Explanation
FALLOPIAN TUBES
• ~10cm long, 4 parts
• Infundibulum - funnel-shaped lateral end, projects beyond the broad ligament with fimbriae at its free end
• Ampulla - widest and longest part, site of fertilisation
• Isthmus - narrowest part, just lateral to the uterus
• Intra-mural part - pierces uterine wall
• Three coats: serous, muscular, and mucous.
• The external or serous coat is peritoneal. The middle or muscular coat consists of an external longitudinal and an
internal circular layer of smooth muscle fibers continuous with those of the uterus
• The mucosa is thrown into longitudinal folds, which in the ampulla are much more extensive than in the isthmus.
Lined by ciliated columnar epithelium
• Blood - ovarian and uterine arteries
• Lymphatics - aortic and internal iliac nodes (follow arteries)
• Nerves - inferior hypogastric plexus

Question 10 The uterine artery

Options for Questions 10-10

A Is a branch of the internal pudendal artery B Is crossed by the ureter


C Reaches the cervix at the level of the external os D Does not enter the broad ligament
Is a branch of the anterior division of the internal
E
iliac artery

A(Correct answ er: E)

Explanation
THE UTERUS
• 8cm long x 5cm wide x 2.5cm thick
• Covered by peritoneum except anteriorly below the reflection of the utero-vesical fold of peritoneum and laterally
between the layers of the broad ligament?
• Fundus - that part of the uterus above the entrance of the uterine tubes?
• Cavity - triangular in coronal section, cleft in saggital section?
• Anteverted uterus - long axis of uterine body at 90degrees to long axis of vagina?
• Ante-flexed uterus - long axis of the body of the uterus bent forward at the level of the internal os?
• Retroverted uterus - body and fundus bent backwards on the vagina to lie within the pouch of Douglas?

This Copy is for Dr. Mohamed ElHodiby


• Anterior relations: utero-vesical pouch and superior surface of bladder?
• Posterior relations: Pouch of Douglas, sigmoid colon and coils of ileum?
• Lateral relations: uterine vessels, ureter?
• Nerve - inferior hypogastric plexuses (Parasympathetic via the pelvic splanchnic nerves, sympathetic via the lumbar
splanchnic nerves). Pain sensation is transmitted via the sympathetic nerves and the lumbar splanchnic nerves??
Lymph drainage:
• Fundus - accompany ovarian artery to para-aortic nodes at the level of L1
• Body and cervix - internal and external iliac nodes

Question 11 This ligament is important in supporting the uterus above the pelvic floor

Options for Questions 11-11

A Broad ligament B Round ligament


C Infundibulo-pelvic ligament D Utero-sacral ligament
E Ileo-pectineal ligament

A(Correct answ er: D)

Explanation
LIGAMENTS
• Transverse cervical ligament: fibro-muscular condensations of pelvic fascia pass from the cervix and upper end of the
vagina to the lateral pelvic wall
• Utero-sacral ligament: cervix and upper end of vagina to the lower end of the sacrum - form two ridges on either side
of the pouch of Douglas
• Pubo-cervical ligament: cervix to posterior surface of pubis
• Round ligament: Of the ovary - from medial margin of ovary to upper part of lateral wall of uterus. Of the uterus - from
upper part of lateral uterine wall to deep inguinal ring
• The uterus is supported mainly by the tone of the pelvic floor muscles (levator ani) which are partly inserted onto the
perineal body and condensations of pelvic fascia forming the transverse cervical, pubo-cervical and utero-sacral
ligaments.
?
THE BROAD LIGAMENT
• Two layered fold of peritoneum extending from the lateral uterine wall to the lateral pelvic wall
• Has an upper free edge which contains the fallopian tube
• The layers of peritoneum separate inferiorly to cover the pelvic floor
• Has ovary attached to its posterior surface by the mesovarium
• Uterine artery crosses the ureter at the base (lower attached border)
• Round ligament of the uterus forms a ridge an the anterior surface
• Contains vestigial structures: epoophron and paroophron (remnant of the mesonephric system)
• Uterine and ovarian blood vessels and lymphatics run within it

Question 12 This ligament runs from the lateral part of the uterus to the deep inguinal ring

Options for Questions 12-12

A Broad ligament B Round ligament


C Infundibulo-pelvic ligament D Utero-sacral ligament
E Ileo-pectineal ligament

A(Correct answ er: B)

This Copy is for Dr. Mohamed ElHodiby


Explanation
LIGAMENTS
• Transverse cervical ligament: fibro-muscular condensations of pelvic fascia pass from the cervix and upper end of the
vagina to the lateral pelvic wall
• Utero-sacral ligament: cervix and upper end of vagina to the lower end of the sacrum - form two ridges on either side
of the pouch of Douglas
• Pubo-cervical ligament: cervix to posterior surface of pubis
• Round ligament: Of the ovary - from medial margin of ovary to upper part of lateral wall of uterus. Of the uterus - from
upper part of lateral uterine wall to deep inguinal ring
• The uterus is supported mainly by the tone of the pelvic floor muscles (levator ani) which are partly inserted onto the
perineal body and condensations of pelvic fascia forming the transverse cervical, pubo-cervical and utero-sacral
ligaments.
?
THE BROAD LIGAMENT
• Two layered fold of peritoneum extending from the lateral uterine wall to the lateral pelvic wall
• Has an upper free edge which contains the fallopian tube
• The layers of peritoneum separate inferiorly to cover the pelvic floor
• Has ovary attached to its posterior surface by the mesovarium
• Uterine artery crosses the ureter at the base (lower attached border)
• Round ligament of the uterus forms a ridge an the anterior surface
• Contains vestigial structures: epoophron and paroophron (remnant of the mesonephric system)
• Uterine and ovarian blood vessels and lymphatics run within it

Question 13 With respect to the innervation and support of the cervix

Options for Questions 13-13

The round ligament provides the most important The broad ligament provides the most important
A B
support to the cervix support to the cervix
The endocervix has autonomic but no sensory Sensory innervation of the cervix is via S2,3,4
C D
innervation spinal segments
The ectocervix has sensory but minimal
E
autonomic innervation

A(Correct answ er: D)

Explanation
?CERVIX
• Lower, narrow portion of the uterus, connected to the uterine fundus by the uterine isthmus - upper limit is the internal
os. Made up mainly of fibrous tissue with very little smooth muscle
• Protrudes through the upper anterior vaginal wall
• Approximately half its length is visible in the vagina (vaginal cervix), the rest being above the vagina (supra-vaginal
cervix)
• The vaginal cervix ~3 cm long and 2.5 cm wide. Size and shape varies widely with age, hormonal state, and parity -
bulkier and the external with a wider and more slit-like external os in multiparous women.
• Ectocervix - portion of the cervix beyond the external os - lined by stratified squamous non-keratinising epithelium.
• Endocervical canal - links external and internal os - lined by columnar epithelium
• The squamo-columnar junction - variable location - high up the endocervical canal before puberty and in the post-
menopausal women. Site of origin of squamous cell carcinoma of the cervix
• The external os is bounded by two lips, an anterior and a posterior, of which the anterior is the shorter and thicker,
although due to the slope of the cervix, it projects lower than the posterior. Both lips are in contact with the posterior
vaginal wall
• The supravaginal cervix is separated in front from the bladder by fibrous tissue (parametrium), which extends also on
to its sides and laterally between the layers of the broad ligaments. Not covered by peritoneum on the anterior aspect
• The uterine arteries reach the margins of the cervix within the parametrium

This Copy is for Dr. Mohamed ElHodiby


• The ureter runs downward and forward 2 cm lateral to the supravaginal cervix
• Posteriorly, the supravaginal cervix is covered by peritoneum, which extends on to the posterior vaginal wall, when it
is reflected on to the rectum, forming the Pouch of Douglas which may contain coils of small intestine.
• The vaginal cervix projects free into the anterior wall of the vagina between the anterior and posterior fornices.
?Blood supply
• Uterine artery, branch of internal iliac
• Cervical and vaginal branches supply the cervix and upper vagina.
• The cervical branches of the uterine arteries descend on the lateral aspects of the cervix at 3 and 9 o'clock. Venous
drainage parallels the arterial supply, eventually emptying into the hypogastric venous plexus.
?Lymphatics
• Regional lymph nodes for the cervix include: paracervical, parametrial, presacral, sacral, external iliac, common iliac,
hypogastric (obturator), internal iliac.

Question 14 Which one of the above statements regarding the cervix is correct?

Options for Questions 14-14

The endocervical canal is lined by cuboidal The ectocervix is lined by stratified squamous
A B
epithelium non-keratinising epithelium
The squamo-columnar junction is located within The endocervical canal links the internal os to the
C D
the ectocervix uterine cavity
Squamous cell carcinoma of the cervix typically
E
arises from the ectocervix

A(Correct answ er: B)

Explanation
?CERVIX
• Lower, narrow portion of the uterus, connected to the uterine fundus by the uterine isthmus - upper limit is the internal
os. Made up mainly of fibrous tissue with very little smooth muscle
• Protrudes through the upper anterior vaginal wall
• Approximately half its length is visible in the vagina (vaginal cervix), the rest being above the vagina (supra-vaginal
cervix)
• The vaginal cervix ~3 cm long and 2.5 cm wide. Size and shape varies widely with age, hormonal state, and parity -
bulkier and the external with a wider and more slit-like external os in multiparous women.
• Ectocervix - portion of the cervix beyond the external os - lined by stratified squamous non-keratinising epithelium.
• Endocervical canal - links external and internal os - lined by columnar epithelium
• The squamo-columnar junction - variable location - high up the endocervical canal before puberty and in the post-
menopausal women. Site of origin of squamous cell carcinoma of the cervix
• The external os is bounded by two lips, an anterior and a posterior, of which the anterior is the shorter and thicker,
although due to the slope of the cervix, it projects lower than the posterior. Both lips are in contact with the posterior
vaginal wall
• The supravaginal cervix is separated in front from the bladder by fibrous tissue (parametrium), which extends also on
to its sides and laterally between the layers of the broad ligaments. Not covered by peritoneum on the anterior aspect
• The uterine arteries reach the margins of the cervix within the parametrium
• The ureter runs downward and forward 2 cm lateral to the supravaginal cervix
• Posteriorly, the supravaginal cervix is covered by peritoneum, which extends on to the posterior vaginal wall, when it
is reflected on to the rectum, forming the Pouch of Douglas which may contain coils of small intestine.
• The vaginal cervix projects free into the anterior wall of the vagina between the anterior and posterior fornices.
?Blood supply
• Uterine artery, branch of internal iliac
• Cervical and vaginal branches supply the cervix and upper vagina.
• The cervical branches of the uterine arteries descend on the lateral aspects of the cervix at 3 and 9 o'clock. Venous
drainage parallels the arterial supply, eventually emptying into the hypogastric venous plexus.
?Lymphatics

This Copy is for Dr. Mohamed ElHodiby


• Regional lymph nodes for the cervix include: paracervical, parametrial, presacral, sacral, external iliac, common iliac,
hypogastric (obturator), internal iliac.

Question 15 Somatic sensory innervation of the lower third of the vagina

Options for Questions 15-15

A Pelvic splanchnic nerves B Lumbar splanchnic nerves


C Pudendal nerve D Superficial femoral nerve
E Obturator nerve

A(Correct answ er: C)

Explanation
THE VAGINA
• ~8cm long, axis directed upwards and backwards from the vulva. Posterior wall longer than anterior wall
• Lined by stratified squamous epithelium which undergoes changes during the menstrual cycle. Does not secrete
mucus
• Has anterior and posterior walls which are normally in apposition, and four fornices (anterior, posterior, left and right
lateral)
• Upper half lies above the level of the pelvic floor
?Relations
• Anterior: bladder, urethra
• Posterior: upper third - pouch of Douglas; middle third - ampulla of the rectum; lower third - perineal body
• Lateral: upper part - ureter, middle part - anterior fibres of levator ani; lower part - uro-genital diaphragm and the bulb
of the vestibule
• Blood: vaginal artery, branch of internal iliac artery
• Lymphatics: upper third - internal and external iliac nodes; middle third - internal iliac nodes; lower third - superficial
inguinal nodes
• Nerve:Upper two thirds - inferior hypogastric plexuses. Parasympathetic from pelvic splanchnic nerves, sympathetic
from lumbar splanchnic nerves (L1 & 2)
• Autonomic innervation to the lower third - pudendal nerve
• Somatic sensation is present mainly in the lower third and is carried by the pudendal nerve

Question 16 Parasympathetic supply to the lower third of the vagina


Options for Questions 16-16

A Pelvic splanchnic nerves B Lumbar splanchnic nerves


C Pudendal nerve D Superficial femoral nerve
E Obturator nerve

A(Correct answ er: C)

Explanation
THE VAGINA
• ~8cm long, axis directed upwards and backwards from the vulva. Posterior wall longer than anterior wall
• Lined by stratified squamous epithelium which undergoes changes during the menstrual cycle. Does not secrete
mucus
• Has anterior and posterior walls which are normally in apposition, and four fornices (anterior, posterior, left and right
lateral)
• Upper half lies above the level of the pelvic floor
?Relations
• Anterior: bladder, urethra

This Copy is for Dr. Mohamed ElHodiby


• Posterior: upper third - pouch of Douglas; middle third - ampulla of the rectum; lower third - perineal body
• Lateral: upper part - ureter, middle part - anterior fibres of levator ani; lower part - uro-genital diaphragm and the bulb
of the vestibule
• Blood: vaginal artery, branch of internal iliac artery
• Lymphatics: upper third - internal and external iliac nodes; middle third - internal iliac nodes; lower third - superficial
inguinal nodes
• Nerve:Upper two thirds - inferior hypogastric plexuses. Parasympathetic from pelvic splanchnic nerves, sympathetic
from lumbar splanchnic nerves (L1 & 2)
• Autonomic innervation to the lower third - pudendal nerve
• Somatic sensation is present mainly in the lower third and is carried by the pudendal nerve

Question 17 Lies posterior to the upper third of the vagina

Options for Questions 17-17

A Urethra B Pouch of Douglas


C Ampulla of the rectum D Perineal body
E Ureter

A(Correct answ er: B)

Explanation
THE VAGINA
• ~8cm long, axis directed upwards and backwards from the vulva. Posterior wall longer than anterior wall
• Lined by stratified squamous epithelium which undergoes changes during the menstrual cycle. Does not secrete
mucus
• Has anterior and posterior walls which are normally in apposition, and four fornices (anterior, posterior, left and right
lateral)
• Upper half lies above the level of the pelvic floor
?Relations
• Anterior: bladder, urethra
• Posterior: upper third - pouch of Douglas; middle third - ampulla of the rectum; lower third - perineal body
• Lateral: upper part - ureter, middle part - anterior fibres of levator ani; lower part - uro-genital diaphragm and the bulb
of the vestibule
• Blood: vaginal artery, branch of internal iliac artery
• Lymphatics: upper third - internal and external iliac nodes; middle third - internal iliac nodes; lower third - superficial
inguinal nodes
• Nerve:Upper two thirds - inferior hypogastric plexuses. Parasympathetic from pelvic splanchnic nerves, sympathetic
from lumbar splanchnic nerves (L1 & 2)
• Autonomic innervation to the lower third - pudendal nerve
• Somatic sensation is present mainly in the lower third and is carried by the pudendal nerve

Question 18 The shape of the pelvic outlet in females

Options for Questions 18-18

A Oval shaped B Circular shaped


C Diamond shaped D Heart shaped
E Quadrilateral shaped

A(Correct answ er: C)

Explanation

This Copy is for Dr. Mohamed ElHodiby


TRUE PELVIS
That part of the pelvis between the pelvic inlet and the pelvic outlet
The pelvic inlet is oval in shape in the Gynaecoid pelvis and has the following borders:
1) Posterior: Sacral promontory?2) Lateral: Iliopectineal line?3)Anterior: Symphysis pubis
The pelvic outlet is diamond shaped with the following borders:
• Posterior: Coccyx
• Lateral: Ischial tuberosities
• Anterior: Pubic arch formed by the simphysis pubis and the ischio-pubic rami
• Part of the lateral border is formed by the sacro-spinous and sacro-tuberous ligaments which convert the greater and
lesser sciatic notches into the greater and lesser sciatic foramina
• The pelvic cavity has a shallow anterior wall and a deeper posterior wall. The ischial spines are at the level of the
mid-cavity
• The acetabulum is formed from the ilium, ischium and pubic bones

Question 19 With respect to the anatomy of the pelvis

Options for Questions 19-19

The cavity of the pelvis has a shallow posterior The ischial spines are at the level of the mid-
A B
wall and a deeper anterior wall cavity
The pubic bone does not form part of the The ischial tuberosities form the medial border of
C D
acetabulum the pelvic outlet
The true pelvis is that part of the pelvis between
E
the pelvic inlet and the ischial spines

A(Correct answ er: B)

Explanation
TRUE PELVIS
That part of the pelvis between the pelvic inlet and the pelvic outlet
The pelvic inlet is oval in shape in the Gynaecoid pelvis and has the following borders:
1) Posterior: Sacral promontory?2) Lateral: Iliopectineal line?3)Anterior: Symphysis pubis
The pelvic outlet is diamond shaped with the following borders:
• Posterior: Coccyx
• Lateral: Ischial tuberosities
• Anterior: Pubic arch formed by the simphysis pubis and the ischio-pubic rami
• Part of the lateral border is formed by the sacro-spinous and sacro-tuberous ligaments which convert the greater and
lesser sciatic notches into the greater and lesser sciatic foramina
• The pelvic cavity has a shallow anterior wall and a deeper posterior wall. The ischial spines are at the level of the
mid-cavity
• The acetabulum is formed from the ilium, ischium and pubic bones

Question 20 The sacro-iliac joint

Options for Questions 20-20

A Fibrous joint B Cartilaginous joint


C Synovial joint D Fibro-cartilagenous joint
E Ball & socket joint

A(Correct answ er: C)

Explanation

This Copy is for Dr. Mohamed ElHodiby


THE SACRO-ILIAC JOINT
• Synovial joint - the irregular articular surfaces of the joint make a contribution to joint stability but this is mainly
maintained by the very strong posterior and inter-osseous sacro-iliac ligaments.
• The sacro-spinous and sacro-tuberous ligaments also contribute to joint stability
• Supplied by branches of the sacral plexus and POSTERIOR rami of S1

Question 21 Which one of the above is not a branch of the sacral plexus?

Options for Questions 21-21

A Sciatic nerve B Obturator nerve


C Nerve to obturator internus D Inferior gluteal nerve
E Superior gluteal nerve

A(Correct answ er: B)

Explanation
SACRAL PLEXUS
Formed from anterior rami of L4&5 (lumbosacral trunk) and anterior rami of S1,2,3&4
Related anteriorly to internal iliac vessels + branches and the rectum
Related posteriorly to the piriformis muscle

Branches?To lower limb - leave the pelvis through GREATER sciatic foramen
• Sciatic nerve - L4,5, S1,2&3; largest nerve in the body
• Superior gluteal nerve - gluteus medius, minimus and tensor fascia lata
• Inferior gluteal nerve - gluteus maximus
• Nerve to obturator internus - also supplies superior gamellus muscle
• Nerve to quadratus femoris - also supplies inferior gamellus muscle
• Posterior cutaneous nerve of the thigh

Question 22 With respect to the anatomy of the female external genitalia

Options for Questions 22-22

The labia majora fuse posteriorly to form the The vestibule is bound laterally by the labia
A B
fourchette majora
The fourchette is located at the apex of the
C The clitoris is located at the base of the vestibule D
vestibule
The urethra opens within the vestibule posterior to
E
the clitoris

A(Correct answ er: E)

Explanation
EXTERNAL GENITALIA
• Labia majora: prominent hair-bearing folds of skin extending from the mons pubis to fuse posteriorly in the
mid-line
• Labia minora: hairless folds of skin within the labia majora; unite posteriorly to form the fourchette; split
anteriorly to enclose the clitoris, forming an anterior prepuce and a posterior frenulum
• Vestibule: triangular area bounded laterally by the labia minora with the clitoris at its apex and the fourchette
at its base
• The urethra opens within the vestibule posterior to the clitoris

This Copy is for Dr. Mohamed ElHodiby


Question 23 Forms the base of the ischio-rectal fossa

Options for Questions 23-23

A Skin B Anal canal


C Obturator internus muscle D Vagina
E Rectum

A(Correct answ er: A)

Explanation
ISCHIO-RECTAL FOSSA
• Wedge shaped space filled with dense fat
• Base formed by skin
• Medial wall formed by anal canal and levator ani muscles
• Lateral wall formed by lower part of obturator internus muscle
• Contains the pudendal nerve and internal pudendal vessels within the pudendal canal on the lateral wall

Question 24 Which one of the above is not a branch of the pudendal nerve?

Options for Questions 24-24

A Perineal nerve B Dorsal nerve of the clitoris


C Inferior rectal nerve D Nerve to obturator internus
E None of the above

A(Correct answ er: D)

Explanation
PUDENDAL NERVE
• Branch of sacral plexus (S2,3,4)
• Leaves pelvic cavity through greater sciatic foramen
• Enters perineum through lesser sciatic foramen
• Enters perineum below and medial to the ischial spine
• Lies medial to the internal pudendal artery as it exits from the pelvis into the perineum
Branches
• Inferior rectal nerve - supplies external anal sphincter and mucous membrane of the lower half of the anal canal
• Dorsal nerve of the clitoris (penis)
• Perineal nerve - supplies muscles of the urogenital triangle and the skin on the posterior surface of the labia majora

Question 25 Blood supply to the medial part of the anterior abdominal wall below the umbilicus

Options for Questions 25-25

A Superior epigastric artery B Inferior epigastric artery


C Superior mesenteric artery D Inferior mesenteric artery
E Intercostal arteries

A(Correct answ er: B)

Explanation
Blood supply – abdominal wall

This Copy is for Dr. Mohamed ElHodiby


• Medial aspects - Superior epigastric artery (above umbilicus - continuation of the internal thoracic artery, branch of
the first part of the subclavian artery. Enters the rectus sheath between the sternal and costal origins of the
diaphragm and descends behind rectus muscle)
• Inferior epigastric artery (below umbilicus - branch of external iliac artery just above inguinal ligament. It pierces the
transversalis fascia to enter the rectus sheath anterior to the arcuate line; runs behind rectus muscle). There is no
anastomosis between the arteries of the left and right side
• Lateral aspects - intercostals, lumbar arteries and deep circumflex iliac artery (branch of external iliac artery above
the inguinal ligament).

Question 26 The sac of this hernia lies below and lateral to the pubic tubercle

Options for Questions 26-26

A Direct inguinal hernia B Indirect inguinal hernia


C Femoral hernia D Richcter’s hernia
E Epigastric hernia

A(Correct answ er: C)

Explanation
HERNIAS
INGUINAL
• Indirect commoner than direct
• Indirect more common in males and on the right side (right testis descends later than the left)
• Direct hernias commoner in (old) males
• Indirect: Enters inguinal canal through deep inguinal ring, lateral to inferior epigastric vessels. Can descent into the
scrotum or labia majora. Sac is formed by the remains of the processus vaginalis
• The sac of all inguinal hernias lies above and medial to the pubic tubercle
FEMORAL
• Commoner in females protrudes through the femoral canal medial to the femoral vein, below and lateral to the pubic
tubercle.
UMBILICAL
• Congenital - exomphalos
• Acquired infantile - weakness in scar of umbilicus
• Acquired adult - para-umbilical, weakness in linea alba above or below umbilicus, commoner in females
EPIGASTRIC
• Weakness in linea alba above umbilicus
RICHCTER’S
• A knuckle of the side-wall of the bowel is incarcerated in the sac but the continuity of the bowel is maintained and
there is no obstruction.

Question 27 Insertion of pyramidalis muscle

Options for Questions 27-27

A Anterior surface of the pubic bone B Posterior surface of the pubic bone
C Linea alba? D Anterior superior iliac spine
E Medial 1/3 of inguinal ligament

A(Correct answ er: C)

Explanation
Pyramidalis

This Copy is for Dr. Mohamed ElHodiby


• ORIGIN: Anterior surface of pubis
• INSERTION: Linea alba

Question 28 Nerve supply to the internal oblique muscle

Options for Questions 28-28

A Upper 6 thoracic nerves B Lower 6 thoracic nerves


C Upper 3 lumbar nerves D Lower 3 lumbar nerves
E 12th thoracic nerve

A(Correct answ er: B)

Explanation
NERVES
• External oblique, internal oblique and transversus: Lower 6 thoracic nerves and L1 (ileoinguinal and ileohypogastric
nerves)
• Rectus abdominis: Lower 6 thoracic nerves
• Pyramidalis: T12
FUNCTION
• External / internal oblique / transversus - laterally flex and rotate the trunk, relax during inspiration to accommodate
abdominal viscera, contract during micturiction, defecation and vomiting.
• Rectus abdominis - flexes the trunk and stabilises the pelvis

Question 29 Which one of the above descriptions of the arcuate line is correct?

Options for Questions 29-29

Insertion of the inguinal ligament on the pubic


A Lateral margins of the rectus abdominis muscle B
bone
Point where the aponeurosis of the internal
Point where the aponeurosis of the external
C oblique muscle passes anterior to the rectus D
oblique passes posterior to the rectus abdominis
abdominis
Attachment of the ileo-pectineal ligament on the
E
superior pubic ramus

A(Correct answ er: C)

Explanation
RECTUS SHEATH
Above costal margin
• Anterior wall: Aponeurosis of external oblique
• Posterior wall: thoracic wall.
Between costal margin and anterior superior iliac spine
• Anterior wall: Aponeurosis of external and internal oblique
• Posterior wall: Aponeurosis of internal oblique and transversus abdominis
• Note that the aponeurosis of the internal oblique splits to enclose the rectus abdominis
Between anterior superior iliac spine and pubis
• Anterior wall: Aponeuroses of external, internal oblique and transversus
• Posterior wall: Transversalis fascia
• Arcuate line : Site where the aponeuroses of the posterior wall pass anterior to the rectus at the level of the anterior
superior iliac spine. The inferior epigastric artery enters the rectus sheath at this point and lies posterior to the rectus
abdominis. Pyramidalis lies within the rectus sheath.

This Copy is for Dr. Mohamed ElHodiby


Question 30 The main action of the psoas major muscle

Options for Questions 30-30

A Medially rotate the hip joint B Laterally rotate the hip joint
C Flex the hip joint D Adduct the hip joint
E Adduct the hip joint

A(Correct answ er: C)

Explanation
PSOAS MAJOR
• Origin: Roots of transverse processes, sides of vertebral bodies and inter-vertebral discs T12 - L5
• Insertion: Lesser trochanter of the femur
• Nerve: Lumbar plexus
• Action: Flexes thigh on trunk. If thigh if flexed, flexes trunk on thigh.

ILIACUS
• Origin: Iliac fossa
• Insertion: Lesser trochanter of the femur
• Nerve: Femoral nerve
• Action: Flexes thigh on trunk. If thigh is flexed, flexes trunk on thigh

This Copy is for Dr. Mohamed ElHodiby


Question 1 The first wave of trophoblast invasion occurs at this gestation

Options for Questions 1-1

A 4-6 weeks B 8-12 weeks


C 12-16 weeks D 16-18 weeks
E 22-24 weeks

A(Correct answ er: B)

Explanation
DEVELOPMENT OF THE VILLOUS TREE
• Primary villous stems become infiltrated by cytotrophoblasts between days 13-21 post-ovulation
• Villous stems are subsequently infiltrated by extra-embryonic mysenchyme which differentiates into fetal blood
vessels
• The distal parts of the villous stems are not vascularised. Here, cytotrophoblasts proliferate and spread laterally to
form a cytotrophoblastic shell, splitting the syncytiotrophoblast into a definitive syncytiotrophoblast on the fetal side
and the peripheral syncytium on the decidual side which degenerates and is replaced by fibrinoid material (Nitabuch's
layer)
• Sprouts extend from primary villous stems, initially made up of syncytiotrophoblast and then infiltrated by
cytotrophoblast and mesenchyme - these are primary stem villi and the placenta is a true villous structure by day 21
of gestation. These villi grow and divide into secondary, tertiary and terminal villi
• The villi oriented towards the uterine cavity degenerate between day 21 and the 4th month to form the chorion laeve.
The overlying decidua degenerates and the chorion laeve comes in contact with the deciduas of the opposite uterine
wall
• The rest of the villi form the chorion frondosum which develops into the definitive placenta
• Division and modification of the villous tree continues until term. First trimester villi are larger, have a complete layer
of cytotrophoblasts and have a loose mysenchymal core which is vascularised towards the end of the first trimester
• At term, the villi are smaller, cytotrophoblasts are few in number, the syncytiotrophoblast is irregularly thinned. Fetal
vessels are sinusoidal and occupy most of the villous core and lie close to the syncytiotrophoblast, forming
vasculusyncytial membranes which maximise materno-fetal transfer.
• Sometimes, the syncytiotrophoblast nuclei appear in clusters called syncytial knots - more common in placentas from
IUGR / pre-eclamptic pregnancies
• Maternal blood is separated from fetal blood by the syncytiotrophoblast and the fetal capillary endothelium

Question 2 The fetal component of the placental bed

Options for Questions 2-2

A Extra-villous trophoblasts B Villous and extra-villous trophoblasts


C Endovascular trophoblasts D Endovascular and extra-villous trophoblasts
E Villous and endovascular trophoblasts

A(Correct answ er: A)

Explanation
THE PLACENTAL BED
• Refers to the decidua and myometrium directly underlying the placenta
• The fetal component is made up of extra-villous trophoblast
• The maternal component is made up of decidualised endometrial stromal cells, macrophages and granular
lymphocytes. Residual endometrial glands are also present.

This Copy is for Dr. Mohamed ElHodiby


Question 3 Amniotic fluid

Options for Questions 3-3

Has a lower concentration of alpha-fetoprotein


A B Has a higher osmolarity than fetal plasma
than fetal blood
Creatinine concentration decreases with
C Has a higher osmolarity than maternal plasma D
increasing gestation age
E Does not contain lipids

A(Correct answ er: A)

Explanation
AMNIOTIC FLUID
• Initially produced by primitive cells around the amniotic vesicle
• Later formed from transudate from fetal skin and umbilical cord and diffusion across the amniotic membrane
• Fetal skin becomes keratinised in the second trimester and amniotic fluid is mainly formed from fetal urine
and lung secretions. The term fetus passes 500-700ml urine per day
• Fetal swallowing is a major route of amniotic fluid re-circulation and begins at 12 weeks. At term. ~500ml
amniotic fluid is exchanged / 24h
• Amniotic fluid volume = 50ml at 12 weeks, 150ml at 16 weeks gestation and ~1000ml at term. Peak volume
is at 32-36 weeks
• Osmolarity: 275 mOsmol/l (lower than maternal or fetal), decreases as pregnancy progresses
• Cells: at term, contains fetal epithelial cells, amniocytes and dermal fibroblasts. Epithelial cells and
amniocytes grow poorly in culture. Glial cells present if neural tube defect
• Protein: concentration increases with gestation but plateaus after 30 weeks. Mainly albumin and globulins.
Also contains AFP (1/10TH concentration in fetal blood - rises until 12 weeks then declines). Virtually no
fibrinogen.
• Urea, creatinine and urate concentration increases with gestation
• Amino acids: concentration similar to that in maternal plasma
• Lipids: mainly free fatty acids. Also contains phospholipids, cholesterol and lecithin (secreted by lungs during
maturation)
• Carbohydrates: mainly glucose; concentration ~ half that of maternal serum
• PO2 = 2-15mmHg while PCO2 = 50-60mmHg
• pH = 7.0 (acidic relative to fetal blood)
• Bilirubin concentration falls in the third trimester (except in haemolytic disease)

Question 4 Which one of the above is not a source of amniotic fluid during the third trimester?

Options for Questions 4-4

A Transudate from fetal skin B Transudate from umbilical cord


C Diffusion across amniotic membrane D Fetal urine
E Fetal lung secretions

A(Correct answ er: A)

Explanation
AMNIOTIC FLUID
• Initially produced by primitive cells around the amniotic vesicle
• Later formed from transudate from fetal skin and umbilical cord and diffusion across the amniotic membrane
• Fetal skin becomes keratinised in the second trimester and amniotic fluid is mainly formed from fetal urine
and lung secretions. The term fetus passes 500-700ml urine per day
• Fetal swallowing is a major route of amniotic fluid re-circulation and begins at 12 weeks. At term. ~500ml
amniotic fluid is exchanged / 24h

This Copy is for Dr. Mohamed ElHodiby


• Amniotic fluid volume = 50ml at 12 weeks, 150ml at 16 weeks gestation and ~1000ml at term. Peak volume
is at 32-36 weeks
• Osmolarity: 275 mOsmol/l (lower than maternal or fetal), decreases as pregnancy progresses
• Cells: at term, contains fetal epithelial cells, amniocytes and dermal fibroblasts. Epithelial cells and
amniocytes grow poorly in culture. Glial cells present if neural tube defect
• Protein: concentration increases with gestation but plateaus after 30 weeks. Mainly albumin and globulins.
Also contains AFP (1/10TH concentration in fetal blood - rises until 12 weeks then declines). Virtually no
fibrinogen.
• Urea, creatinine and urate concentration increases with gestation
• Amino acids: concentration similar to that in maternal plasma
• Lipids: mainly free fatty acids. Also contains phospholipids, cholesterol and lecithin (secreted by lungs during
maturation)
• Carbohydrates: mainly glucose; concentration ~ half that of maternal serum
• PO2 = 2-15mmHg while PCO2 = 50-60mmHg
• pH = 7.0 (acidic relative to fetal blood)
• Bilirubin concentration falls in the third trimester (except in haemolytic disease)

Question 5 The mechanism of transfer of carbondioxide across the placenta

Question 6 The mechanism of transfer of oxygen across the placenta

Options for Questions 5-6

A Simple diffusion B Facilitated diffusion


C Active transport D Secondary active transport
E Bulk transfer

A(Correct answ er: A)

A(Correct answ er: A)

Explanation
CARBOHYDRATES & AMINO ACIDS
• Glucose transport across the microvillous and basal plasma membranes is by facilitated diffusion.
• This is dependent on glucose concentration gradient and the activity of transport proteins within the placenta. In
theory, facilitated diffusion can be saturated.
• In practice, the glucose transfer capacity (especially of the microvillous plasma membrane) is so large that it would
not approach saturation under in-vivo conditions
• Fetal amino acid concentrations are generally higher than maternal levels
• Transfer of amino acids from the mother to the fetus is therefore against a concentration gradient and energy (ATP) is
required for this process
• This energy is provided by coupling the transfer of amino acids (up a concentration gradient) to the transfer of sodium
ions (down a concentration gradient) - secondary active transport. These are called Na+-dependent transporters. The
sodium concentration gradient is maintained by Na+K+ATPase.
• Other amino acid transport systems are, however, Na+-independent.

Question 7 Which one of the above is not readily transferred across the placenta?

Options for Questions 7-7

A Amino acids B Glucose


C Thyroxine D IgG
E Ca2+

This Copy is for Dr. Mohamed ElHodiby


A(Correct answ er: C)

Explanation
Molecule Placental transfer

Testosterone Minimal transfer - androgens aromatised by placenta. Very high maternal androgen
concentration may virilise female fetus

Ca2+, Mg2+ Active transfer against concentration gradient

PTH, Calcitonin Not transferred

Vitamin D Good transfer

IgA Minimal passive transfer

IgG Good active and active transfer from 7 weeks gestation

IgM No transfer

Glucose Facilitated diffusion - excellent transfer

Amino acids Active transport - excellent transfer

Free fatty acids Very limited transfer - essential fatty acids only

Ketone bodies Excellent transfer - diffusion

Insulin,
No transfer
glucagon
Thyroid
Poor transfer - diffusion
hormone

TRH Excellent transfer

Iodine and
Excellent transfer
thioamides
Cortisol &
Excellent transfer
aldosterone

ACTH No transfer

Question 8 Identical twins always have this type of placentation

Options for Questions 8-8

A Monochorionic monoamniotic B Dichorionic diamniotic


C Dichorionic monoamniotic D Monochorionic diamniotic
E None of the above

A(Correct answ er: E)

Explanation
MULTIPLE PREGNANCY?Twins
UK data 2007: ~ 1 in 65 (1.5%) pregnancies were twins
Prevalence varies world-wide, being lowest in Japan and highest in Nigeria
Incidence of monozygotic twins relatively constant world-wide at ~ 3.5 per 1,000 births

This Copy is for Dr. Mohamed ElHodiby


Incidence of dizygotic twins varies widely
Dizygotic twins
• Fertilisation of two oocytes by different sperm
• Dizygotic twins have no more resemblance than brothers / sisters of different ages
• Both zygotes implant independently in the uterus and there are two separate placentas, amniotic and chorionic sacs
• The placentas and chorionic sacs may come into close approximation and fuse.
• Dizygotic twins cannot be monochorionic and cannot be identical. They are always dichorionic and diamniotic
Monozygotic twins
• A single ovum is fertilised and splits into two at different stages of development
• Earliest separation occurs at the 2 cell stage producing two zygotes which enter the uterus independently. Each
embryo would have its own placenta and chorionic sac, forming dichorionic diamniotic twins
• In the majority of cases, splitting occurs in the early blastocyst stage. The inner cell mass splits into two, producing
two embryos with a common placenta and a common chorionic cavity but separate amniotic cavities. These form
monochorionic diamniotic twins
• Rarely, splitting occurs at the stage of the bilaminar germ disc, resulting in two embryos with a common placenta,
common chorionic cavity and common amniotic cavity. These form monochorionic monoamniotic twins
• Splitting at later stages of development may result in incomplete splitting of the axial area of the germ disc, resulting
in conjoint twins

Question 9 Dizygotic twins always have this type of placentation

Options for Questions 9-9

A Monochorionic monoamniotic B Dichorionic diamniotic


C Dichorionic monoamniotic D Monochorionic diamniotic
E None of the above

A(Correct answ er: B)

Explanation
MULTIPLE PREGNANCY?Twins
UK data 2007: ~ 1 in 65 (1.5%) pregnancies were twins
Prevalence varies world-wide, being lowest in Japan and highest in Nigeria
Incidence of monozygotic twins relatively constant world-wide at ~ 3.5 per 1,000 births
Incidence of dizygotic twins varies widely
Dizygotic twins
• Fertilisation of two oocytes by different sperm
• Dizygotic twins have no more resemblance than brothers / sisters of different ages
• Both zygotes implant independently in the uterus and there are two separate placentas, amniotic and chorionic sacs
• The placentas and chorionic sacs may come into close approximation and fuse.
• Dizygotic twins cannot be monochorionic and cannot be identical. They are always dichorionic and diamniotic
Monozygotic twins
• A single ovum is fertilised and splits into two at different stages of development
• Earliest separation occurs at the 2 cell stage producing two zygotes which enter the uterus independently. Each
embryo would have its own placenta and chorionic sac, forming dichorionic diamniotic twins
• In the majority of cases, splitting occurs in the early blastocyst stage. The inner cell mass splits into two, producing
two embryos with a common placenta and a common chorionic cavity but separate amniotic cavities. These form
monochorionic diamniotic twins
• Rarely, splitting occurs at the stage of the bilaminar germ disc, resulting in two embryos with a common placenta,
common chorionic cavity and common amniotic cavity. These form monochorionic monoamniotic twins
• Splitting at later stages of development may result in incomplete splitting of the axial area of the germ disc, resulting
in conjoint twins

Question 10 With respect to the complications of twin pregnancies

This Copy is for Dr. Mohamed ElHodiby


Options for Questions 10-10

In a monochorionic twin pregnancy, if one fetus


Cord entanglement does not occur in dizygotic
A B dies, there is a 50-75% risk of the other fetus also
twins
dying
Dichorionic twin pregnancies have a higher fetal The risk of neuro-developmental delay is higher in
C D
loss rate than monochorionic twin pregnancies dichorionic twins than in monochorionic twins
In the twin placenta, artery-to-vein anastomoses
are associated with a lower risk of twin-to-twin
E
transfusion syndrome than artery-to-vein
anastomoses

A(Correct answ er: A)

Explanation
Monochorionic twins
• Both fetuses are dependent on a single shared placental mass
• One in 3 twin pregnancies have a monochorionic placenta
• Twin pregnancies are associated with an increased risk of maternal pregnancy symptoms, pre-term delivery, fetal
growth restriction, pre-eclampsia, and PPH. These risks are further increased in monochorionic twin pregnancies
• Additional problems associated with monochorionic twins include:
1) Twin-to-twin transfusion syndrome
• This is a complication of disproportionate blood supply to the fetuses in a monochorionic twin pregnancy, resulting in
one twin (the donor) being smaller with oligohydramnios and the recipient being bigger with polyhydramnios
• Associated with significant morbidity and morbidity with severe TTTS having a perinatal mortality rate of 60-100%
• Complicates 10-15% of monochorionic twin pregnancies but is less frequent in mono-amniotic compared to di-
amniotic twins
• Results from blood flow across the vascular anastomoses that connect the two umbilical circulations. Pregnancies
complicated by TTTS are more likely to have unidirectional artery-to-vein anastomoses and less likely to have bi-
directional artery-to-artery anastomoses.
• Artery-to-artery anastomoses can be identified antenatally using colour Doppler. Their absence is associated with an
increased risk of TTTS (61% Vs 15%)
2) Consequences to the co-twin of fetal death. Risk of death of surviving twin is ~12% and risk of neurological
abnormality ~18%?
3) Management of discordant fetal malformation?
4) Risk of cord entanglement in mono-amniotic twins
As a consequence, monochorionic twin pregnancies have a higher fetal loss rate compared to dichorionic twins,
mainly due to second trimester loss. The risk of fetal loss after 24 weeks is also increased. There may also be an
increased risk of neuro-developmental delay

This Copy is for Dr. Mohamed ElHodiby

Вам также может понравиться